The central climate fallacy is that the unknowns are known

By Christopher Monckton of Brenchley

What is science for? Its end and object is to stretch out a fumbling hand for the truth by a humble and eternally-unsatisfied attempt to constrain uncertainties.

The scientific method is hungry curiosity, followed by acute observation, followed by careful measurement, followed by the meticulous application of pre-existing theory to the results, followed by the detailed drafting and reviewed publication of a hypothesis, followed by other scientists’ attempts to overturn the hypothesis, which is either discarded or slowly accorded credence to the extent that it has survived the process of error elimination.

In the physical sciences, absolute proof – which mathematicians call demonstration – is very seldom available. In mathematics, the art that is the language and the queen of the sciences, demonstration is more often available, particularly in number theory, the branch of discrete mathematics that concerns itself chiefly with the integers. Die ganze Zahl schuf der liebe Gott, said Kronecker. Alles übrige ist Menschenwerk.

One of the reasons why the Thermageddonite superstition has been so successful until recently is that just about the only formal demonstration that nine-tenths of the population recall – and even then hazily – is that in the Euclidean and hyperbolic planes the square (or semicircle) on the hypotenuse of a right triangle is equal to the sum of the squares (or semicircles) on the other two sides.

Now, the problem with the theorem of Pythagoras is that it is unquestionably true. For those who agree with the philosopher Schopenhauer that Euclid’s demonstration is “a triumph of perversity”, here is the admirably clear proof by dissection attributed to the fifth-century Hindu mathematican Aryabhatta.

clip_image002

However one looks at the theorem, the wretched thing is objectively true. There are hundreds of distinct demonstrations of it. It is true whether or not you or I or anyone else wants it to be true or believes it to be true or knows it to be true. In the language of theology, it is intrinsice honestum. It is right in itself.

I submit that it is the theorem of Pythagoras that has allowed the Thermageddonites to succeed when, scientifically speaking, they should simply have been laughed at.

For the truth of that venerable theorem has misled generations who have been given some acquaintance with the sciences but little or none with the philosophy of science into believing that every scientific finding is definitive, unalterable, unchallengeable and unquestionable.

Yes, liberating greenhouse gases by combustion will cause some warming – all other things being equal. Yes, we are liberating greenhouse gases by combustion. These things have been established beyond reasonable doubt. But what has not been established is the crucial quantitative question how much warming we may cause.

That simple fact has been relentlessly and artfully concealed from the population, who have been sedulously deluded into believing that “the science is certain” and that anyone who questions how much warming we may cause is by implication also challenging the well-established experimental results showing that greenhouse gases cause warming.

It is worth briefly considering the how much question – or, as the climate scientists call it, the question of climate sensitivity.

In response to my recent posting pointing out that on the RSS satellite record there has been no global warming for 17 years 10 months, a commenter, one Stacey, asked:

“Please could you … demonstrate the actual climate sensitivity in respect of the last 30 years due the increase in CO2 in the atmosphere? My guess is that it’s zero. :-)”

“Stacey” is adopting a sound, scientific approach, beginning with that vital first step: curiosity. Socrates used to say that the first step on the road to knowledge was an awareness of one’s own ignorance. And what is curiosity, the wellspring of the sciences, if not an awareness of one’s own ignorance coupled with a determination to do something about it?

“Stacey” offers the hypothesis, that climate sensitivity to CO2 over the past 30 years has been zero, and asks whether she is right.

So let us examine the hypothesis. We begin with the observation that in the climate there appear to be quasi-periodicities of about 60 years in global temperature – roughly 30 years’ tendency to warm followed by 30 years’ tendency to cool – and that these cycles are tied to, and may be caused by, the great ocean oscillations, of which the most influential seems to be the Pacific Decadal Oscillation.

Therefore, in order to cancel these naturally-occurring cycles, it is necessary either to take periods that are multiples of 60 years in length or to take periods that straddle a phase-transition from one 30-year phase of the PDO to another.

Usefully, the most recent phase-transition in the PDO was in about the year 2000, so that the past 30 years will have been about equally influenced first by a warming phase of the PDO and then by a cooling phase. That removes the most obvious potential natural distortion of our attempt to determine climate sensitivity from what has actually occurred.

Next, measurement. How much global warming was there in the past 30 years? For this, we shall take the mean of the global mean surface temperature anomalies on all five principal datasets – GISS, HadCRUT4, NCDC, RSS, and UAH.

clip_image004

We now make the assumption, for the sake of beginning somewhere, that all of the global warming over this 30-year period was attributable to us.

As the CO2 record on the chart shows, CO2 concentration rose by 54 ppmv over the 30-year period. We shall assume that all that increase in concentration was anthropogenic. If so, official theory suggests it caused 0.78 Watts per square meter of radiative forcing.

We shall also assume that over the period CO2 represented 70% of all anthropogenic forcings, so that the total anthropogenic forcing was 1.11 Watts per square meter. This assumption is derived from the IPCC’s Fifth Assessment Report.

We shall further assume that none of the warming was committed but unrealized warming from before 1984, or that, if there was some, it was approximately balanced by uncommitted warming generated over the 30-year period but not yet apparent in the temperature record.

Now we can divide the measured temperature change of 0.48 Cº by the total anthropogenic forcing of 1.11 Watts per square meter to obtain the transient-sensitivity parameter λt where t = 30 years, which is 0.43 Cº per Watt per square meter.

However, the instantaneous, Planck or zero-feedback sensitivity parameter λ0, which is the mean of the first derivatives across all latitudes of the fundamental equation of radiative transfer at the characteristic-emission altitude, where incoming and outgoing radiative fluxes balance, is just 0.31 Cº per Watt per square meter (IPCC, 2007, p. 631 fn.).

Next, some handy equations, that you will not find in most textbooks, will help us to determine the system gain Gt, the closed-loop feedback gain gt, and the feedback-sum ft directly from the Planck sensitivity parameter λ0 and the transient-sensitivity parameter λt:

clip_image006

The transient system gain Gt, which is the factor by which the direct warming from anthropogenic forcings is multiplied to allow for the action of feedbacks over the 30-year period, is simply 0.43 / 0.31, or 1.39, showing that short-acting feedbacks have increased the direct warming by almost two-fifths.

The loop gain gt, which is the product of the sum of the short-acting feedbacks and the Planck parameter, is 1 – 0.31/0.43, or 0.28.

Then the feedback sum ft, the sum of all short-acting feedbacks over the period, is 1/0.31 – 1/0.43, or 0.9 Watts per square meter per Celsius degree of warming over the period.

The IPCC’s current central estimate is that at equilibrium, when all short-acting and long-acting feedbacks have acted, the feedback sum is just 1.5 Watts per square meter per Cº (IPCC, 2013, table 9.43, red dot at top right). This is a substantial reduction from the 2.1 Watts per square meter per Cº (blue dot at top right) central estimate implicit in IPCC (2007).

From that current estimate of the equilibrium feedback-sum f, which is the sum of all feedbacks acting over the thousands of years till the climate has established a new equilibrium following perturbation by a forcing, the equilibrium system gain G may be determined, using the Bode system-gain equation:

clip_image008

From the Bode equation, the equilibrium system gain G is [1 – 0.31(1.5)]–1, or 1.87. In short, the IPCC’s current implicit central estimate is that feedbacks increase the direct forcing-driven warming by seven-eighths.

clip_image010

The central estimate of Charney sensitivity implicit in the IPCC’s current central estimate of the feedback sum is given by

clip_image012

Thus, the IPCC’s current central estimate of Charney sensitivity to a doubling of CO2 concentration ought to be (5.35 ln 2)(0.31)(1.87), or 2.1 Cº. So the 3.2 Cº that is the current central estimate of the CMIP5 model ensemble is too big by at least half.

The eventual warming as a result of the anthropogenic forcings of the past 30 years will be (1.11)(0.31)(1.87) = 0.64 Cº, of which 0.48 Cº, or three-quarters, has already occurred, leaving just 0.16 Cº warming committed but unrealized over the period. Yet the IPCC says there is about four times that amount of committed but unrealized warming in the pipeline.

Now, all that is set out above is mainstream climate science. It is possible that we caused some of the warming of the past 30 years. It is possible, though by no means certain, that we caused at least half of it.

Why, then, can we not be more precise in deriving a climate-sensitivity estimate from the temperature change and CO2 concentration change we have seen over the past 20 years?

The answer – though you will very seldom hear it from the Church of Thermageddon – is that there are far too many unknowns. The biggest outright lie in the IPCC’s 2013, 2007, and 2001 reports is the notion that we can determine with any confidence the fraction of global warming over recent decades that is attributable to us.

In 2001 the IPCC said it was 66% confident we had caused most of the warming since 1950; in 2001, 90% confident; in 2013, 95-99% confident. All of these confidence values are direct lies. For there is no dataset from whose values any such supposed confidence levels can be determined by any recognizable statistical process.

It is a measure of the contempt in which today’s scientific elite holds the rest of the population that the IPCC, as the elite’s soi-disant spokesman on climate change, could have made and persisted in and doubled down upon these particular lies. It knows it can get away with them, for scientists have abdicated, forfeiting their high priesthood to far-left politicians and profiteering environmental protection rackets.

We do not know enough about the climate to say in what direction, let alone by how much, the climate would be changing in the absence of any anthropogenic influence. We do not know the forcing from CO2 or any other greenhouse gas to a sufficient precision. The official value of the CO2 forcing was cut by a massive 15% in 2001.

We do not know the value of the Planck parameter to any great precision. We thought we did: then the actual mean surface temperature of the Moon was measured, and was found to be 40% adrift from the official value that had been determined on the basis of the lunar sensitivity parameter, and had been confidently published on official government websites.

We do not know what fraction of manmade warming is attributable to each of the greenhouse gases. We do not know the magnitude or even the sign of the forcing from our emissions of soot to the atmosphere.

We certainly do not know the values of the individual temperature feedbacks to anything like the narrow error bars falsely claimed by the IPCC. We do not even know the sign of the cloud feedback, and the IPCC has recently been forced to reduce the value of that feedback drastically.

Then there are the major non-radiative transports: evaporation and convection up, advection across, and precipitation and subsidence down.

We cannot predict the behavior of the oceans – indeed, we cannot even measure changes in their heat content with sufficient resolution to give a meaningful result.

We cannot predict el Niño and la Niña events. Look how many enviro-left news media predicted a record-busting event this year. They predicted it because they wanted it.

They needed it, desperately, to bring the Great Pause to an end and reduce the humiliation that the less dishonest among them are beginning to feel now that the laymen like me at whom they have viciously snapped and sneered for so long are – so far, at any rate – closer to the mark than they and their expensive but useless models.

For it would take only a very small reduction in each of the assumed values on the basis of which the IPCC makes its predictions to reach a climate sensitivity far less than the models’ currently-claimed central estimate of 3.2 Cº per CO2 doubling. And there are powerful theoretical reasons why some of those values must have been greatly overstated, though there is no space to consider them here.

My own best estimate, for what little it is worth, is that a doubling of CO2 concentration would warm the world by about 1 Cº, if that. The IPCC is heading – albeit far too slowly – towards the same answer. It has cut the CO2 forcing; it has accepted that the next-biggest anthropogenic forcing, from methane, has been vastly overestimated; it has slashed the feedback-sum and consequently the equilibrium system gain; it has all but halved its near-term predictions of global warming.

However, the honest answer to Stacey’s question about what climate sensitivity is indicated by the temperature record of the past 30 years is, “We do not know”.

But you will never hear that honesty on the lips of the Thermageddonites, as they furtively and profitably cement in place the last of the thousand interlinked supranational bureaucracies that are the silent sinews of what – if they get their way in Paris in December next year – will be an all-powerful world government founded upon the greatest lie ever told, the lie of certainty in a cloud of unknowing, elected by none, loved by few, feared by all, and answerable only to itself.

0 0 votes
Article Rating
263 Comments
Oldest
Newest Most Voted
Inline Feedbacks
View all comments
August 3, 2014 11:34 am

The problem with most data sets is that they not properly balanced NH/SH. We should not carry on with them until they are properly balanced.
Here are the results of a properly balanced sample of weather stations:
http://blogs.24.com/henryp/files/2013/02/henryspooltableNEWc.pdf
I have three different data sets, namely one for the speed of maxima, one for means and one for minima. All three data sets show deceleration of warming, to the point where we are now actually globally cooling [from 2000], with high correlation. The maths is as simple as what you learned in high school, when somebody throws a ball. You get a curve when you set the speed of the ball in m/s out against time. That is when you see acceleration and deceleration in the curve.
In the case of my last data set, for minimum temperatures, which is supposed to show chaos, due to alleged man made warming, I found the following final results:
over the past 40 years, from 1974 until 2014 the speed of warming was 0.004 K/annum
over the past 34 years, from 1980 until 2014 the speed of warming was 0.007 K/annum
over the past 24 years, from 1990 until 2014 the speed of warming was 0.004 K/annum
over the past 14 years, from 2000 until 2014 the speed of warming was -0.009 K/annum
Setting the speed of warming in K/annum out against time, you find the deceleration in K/annum2
Admittedly, I only have 4 points to find the deceleration. But it is enough. I always used 4 points in photometry, AAS, etc. As long as the curve/relationship is perfectly defined within the range.
See graph at the bottom of the last table
The curve I found shows Rsquared = 1. That means that the warming over time is perfectly defined by the reported quadratic function.
At any point in the past 40 years I can tell you exactly what the speed of warming was [as far as minima is concerned]
Somebody else who duplicates my results, should find the same function or something very close. Similar to throwing the dice and finding that the average of all throws is 3.5.
In my case, I just happened to find the right final number. God, or nature, if you please, has thrown us a ball. Man made warming (AGW) is, or must be, exactly 0.000K/annum. Everything is going down so naturally. There simply is no room for any AGW in the equation.
Unless somebody here has any ideas how we could put it in, so that it comes out 100% parabolic?

H Grouse
August 3, 2014 11:57 am

“Yes, liberating greenhouse gases by combustion will cause some warming – all other things being equal. Yes, we are liberating greenhouse gases by combustion. These things have been established beyond reasonable doubt”
Then you need to explain why in the past 17 years an 10 months the increase of 35 ppm (10%) has had no effect.
..

Leonard Weinstein
August 3, 2014 12:02 pm

Your analysis assumed that drift up over the last 150 or so years, and especially rising more recently, superimposed over those 60 year cycles, was possibly caused by human activity. Since the longer period cycles over the last 10,000 years, including the approximately 1,000 year cycles over the last 2,000 years, resulted in about as large variation up in previous times (and were not possibly human caused), even the assumption that recent increases are different is without any separate supporting logic. I realize you were not calling this a true indicator of cause and effect, but only representing that even if it were true, the IPCC did bad maths. However I think that point needs to be made here.

milodonharlani
August 3, 2014 12:04 pm

Team CACA makes the same anti-scientific mistake as the Intelligent Design movement. ID proponents look at one of the bacterial flagella & toss up their hands, asking, “What else could explain this structure than an Intelligent Designer (presumably set on afflicting humanity with more effective pathogens)?” The “climate science” Team similarly asks, “What else could have cause the warming observed (or adjusted) or “climate change” since 1950 (or whenever) but man-made greenhouse gases?”
In both cases, the advocates of phony “science” abandon all the curiosity at the root of real science. Instead of searching for valid explanations via the scientific method, ie making guesses & testing them against reality, they’re content, indeed happy, not only to look no further themselves, but to keep others from doing so by asking the questions they are too scared & comfortable to ask.

milodonharlani
August 3, 2014 12:13 pm

H Grouse says:
August 3, 2014 at 11:57 am
It probably has had an effect, but so minor that natural cycles swamp it out. At 365 ppm, the world would probably be slightly cooler than at 400 ppm, as now. And less green. Such a small increase is negligible, as indeed is 100 ppm, when considered against the 40,000 ppm of water vapor in the tropics.

August 3, 2014 12:14 pm

milodonharlani, I find Team CACA to be closer to astrologists (the ones who claim it is real).
They do have a set methodology.
They do have formal reporting procedures.
They do make predictions about the future.
But the predictions are either:
A) Too vague to be meaningful
B) Untestable
C) Changed after the fact (we were talking about air temperature but we meant ocean T; we said war would destroy a great nation – did you think that was the other guy?)
It looks like science and it acts like science.
But it isn’t falsifiable.
Let’s hope it doesn’t last as long as astrology

D. Cohen
August 3, 2014 12:23 pm

Don’t forget that we also do not know whether several degrees C of warming would, on balance, be good or bad for humanity. Remember, most of the predicted warming takes place near the poles, the daily highs are not expected to change much, only the daily lows increased to get an overall average daily temperature. Hence longer growing seasons, more food — and as the ice retreats, more land to colonize as well as more ocean to fish. How bad can that be? Dig deep into the assumptions behind any flavor of environmentalism and you find that change of any sort is automatically assumed to be bad. This plays to the buried fears of late-middle-aged individuals with lots of spare money to contribute to political causes and crusades (and who are just beginning to recognize and dread their own all-too-rapidly approaching old age and death) but has little else to recommend it.

August 3, 2014 12:27 pm

My dear Monckton, you take the prize oh yes you do. I am what might be called a die-hard empiricist, and empiricism is not the favorite approach of the climate scientists. They love theory and this love leads them through many a tangled thicket in their quest for the truth. You are a good example of one of these questors, albeit one that I nearly always find myself in agreement with which is not usually the case with these pestiferous theorists who are always basing their turgid math on doubtful assumptions.
Concerning climate sensitivity, this is theoretically and variously determined and can never be affirmed by any data, by which I mean that there is no way to determine incontrovertibly how much of a trend is assignable to CO2 and how much to natural causes.
This is my approach: the late warming trend of circa 1977-97 is best explained by increased insolation. For 37 or so years prior there was no warming and for nearly 18 years since there was no warming. So where is the so called climate sensitivity? I would say that it is still in the tangled thickets of theory and there it will stay.
But what a worker you are (you are a demon for postings) and how admirable is your clarity of expression.

dmacleo
August 3, 2014 12:29 pm

We now make the assumption, for the sake of beginning somewhere, that all of the global warming over this 30-year period was attributable to us.

but if we were NOT responsible for all of it how would that affect the rest of the computations?
Its always seemed to me (just an opinion) that so many of the issues MAY stem directly from this.
By assuming this is is correct, for the reasons you stated, are we possibly also enabling bad data?

However, the honest answer to Stacey’s question about what climate sensitivity is indicated by the temperature record of the past 30 years is, “We do not know”.

I think this succinctly wraps it all up. I think way to many are afraid to just say they do not know.
thanks.
[1. You must use “angled brackets” – NOT rectangle “[]” brackets around your html coding.
[2. By convention, on this site, only the moderators use [] brackets. They indicate editing.
[3. It is best to “test” html entries on the “Test” thread. See the site, top-level header for the link. .mod]

August 3, 2014 12:30 pm

D.Cohen
http://wattsupwiththat.com/2014/08/03/the-central-climate-fallacy-is-that-the-unknowns-are-known/#comment-1700656
henry says
My results suggest there is no global warming now [anymore], there is no discernible man made warming. There is only global cooling coming up ahead.
http://wattsupwiththat.com/2014/08/03/the-central-climate-fallacy-is-that-the-unknowns-are-known/#comment-1700623
t really was very cold in 1940′s….The Dust Bowl drought 1932-1939 was one of the worst environmental disasters of the Twentieth Century anywhere in the world. Three million people left their farms on the Great Plains during the drought and half a million migrated to other states, almost all to the West. http://www.ldeo.columbia.edu/r
Danger from global cooling is documented and provable.

August 3, 2014 12:32 pm

D. Cohen says at August 3, 2014 at 12:23 pm

Don’t forget that we also do not know whether several degrees C of warming would, on balance, be good or bad for humanity.

Very good point. And many of the supposed negative impacts of several degrees C of warming are actually going to happen anyway – it just affects the timing.
Thus the expected bad is inevitable anyway and getting the good, first, may well be a net gain.
An example would be the collapse of the West Antarctic Ice Sheet. It will happen at some point this millennium (probably) with or without CO2 emissions. Even if CO2 speeds up the disaster… the gain from increased wealth may well make the downside irrelevant.
And if we are poorer the inevitable collapse may be worse.

Monckton of Brenchley
August 3, 2014 12:36 pm

I have some sympathy with Henry P’s analysis, but one should be careful not to make precise predictions on the basis of past trends on notoriously stochastic data. There may be cooling to come, as the solar physicists think, but global temperature does not change easily, so the best projection is for a small change in either direction over the coming century. A large change in either direction is unlikely.
.
Mr Grouse asks me to explain why there has been no global warming for 17 years 10 months. He fails to cite in full the passage in the head posting that leads him to ask the question. I had said that CO2 causes warming, all other things being equal. But it is not known whether all other things are equal, which is why the head posting makes the point that exaggerated claims of high climate sensitivity are not yet justifiable.
Mr Weinstein says I should not have assumed that recent warming was largely manmade. I made no such assumption: I said it was a possibility. There are too many unknowns to justify the absurd expenditure that the extremists in the governing class are making.

dp
August 3, 2014 12:38 pm

Thank you, Lord Monckton, for providing the supporting analysis for my assertion here: http://wattsupwiththat.com/2014/07/27/the-ddp-conference/#comment-1699585
Why this important point remains well below the radar I don’t know.

Sasha
August 3, 2014 12:39 pm

Dear Stacey,
The answer to your question is a figure so low as to be statistically insignificant, which means you are correct – for all practical purposes it is zero.

dp
August 3, 2014 12:39 pm

For blockquoting don’t use square brackets around the tag.

August 3, 2014 12:39 pm

In 2001 the IPCC said it was 66% confident we had caused most of the warming since 1950; in 2001, 90% confident; in 2013, 95-99% confident. All of these confidence values are direct lies. For there is no dataset from whose values any such supposed confidence levels can be determined by any recognizable statistical process. ~ Lord Monckton

Agreed. All of those confidence levels are indeed odious, harmful lies.
The real question is how can we believe anything that the IPCC or the government controlled data sets tell us? If they would lie about the confidence levels — manipulate the public by these lies — why would they not manipulate all manner of other so-called facts and data?
If one looks at one of the long term temperature measuring stations like one that Lord Monckton has mentioned before in central England, one sees that we have had a fairly steady and small rise in temperature since the end of the Little Ice Age. How can we honestly measure any supposed temperature increase due to slightly higher CO2 amounts in the atmosphere?

dmacleo
August 3, 2014 12:43 pm

ok this one uses {} and not [] now?
thanks
[1. You must use “angled brackets” – NOT rectangle “[]” brackets around your html coding.
[2. By convention, on this site, only the moderators use [] brackets. They indicate editing.
[3. It is best to “test” html entries on the “Test” thread. See the site, top-level header for the link. .mod]

milodonharlani
August 3, 2014 12:46 pm

markstoval says:
August 3, 2014 at 12:39 pm
IMO just as bad as the totally bogus confidence levels is the fact that the Team steadfastly refuses to offer an estimate of what they mean by “primarily” (or whatever unquantified term) man-made, let alone show how they derive such a result.
I’m 95% confident that the human contribution (if any) to warming (if any) since 1950 is less than half, by methods at least as valid as those of IPeCaC, unless you count bogus adjustments to the “record” & maladjusted UHI effects as anthropogenic climate change.

August 3, 2014 12:51 pm

dmacleo
It’s with blockquote in the middle.
Or with b in the middle for bold
Or a href=”link here” for quoting

John W. Garrett
August 3, 2014 12:56 pm

Bravo and thank you.

John West
August 3, 2014 12:57 pm

@ dmacleo
Less Than and Greater Than
see:
http://home.comcast.net/~ewerme/wuwt/index.html
Scroll down to: Formatting in comments.

dmacleo
August 3, 2014 1:00 pm

I got it now guys, I don’t know how/why I forgot how to do this as I have done it many times here in the past.
think maybe dealing with some php stuff today stuck in my head.

Steve in SC
August 3, 2014 1:04 pm

“Then there are the major non-radiative transports: evaporation and convection up, advection across, and precipitation and subsidence down.”
That, sir, is the proverbial elephant in the proverbial room. Heat transfer has routinely been neglected by alarmist and skeptic alike.

August 3, 2014 1:05 pm

Irrespective of all of the above, for me the question is “Why did the earth not fry when CO2 was at several thousand ppm?”
Let the Thermageddonites answer that one.
(The spellchecker doesn’t like “Thermageddonites” for some reason.)

August 3, 2014 1:09 pm

Another Unknown:
I’ve speculated that the impact of UHIs is underestimated.
If a large UHI is in the right (or wrong) place it could disturb the prevailing winds around it.
That could affect the sea currents below.
That could affect the heat distribution of the whole planet.
That could affect the re-emission of heat out of the planet.
Unknown. SciFi

August 3, 2014 1:16 pm

@my dear lord
I was not making any exact predictions
in fact, I just added the forward 5 year only for illustration……
I AM saying we are globally cooling now, and there is no such thing as a “pause”
In nature there no “pause”. We are on a natural curve, so it is either cooling or it is warming.
You must change that mantra and go with what the data are saying, now…….
You also simply ignore the fact that recording- and calibration procedures have greatly improved during the past 5 decades. e.g. recording is now done automatically with multiple probes. Indeed, I have challenged [anyone] to show me a re-calibration certificate of a thermometer dated before 1945
Perhaps you have one of those certificates that I do not know of?
So, really, let us agree to use no data collected before 1964. Where would that leave us? [I think] exactly where I showed you:
http://blogs.24.com/henryp/files/2013/02/henryspooltableNEWc.pdf
3 x rsquared greater than 0.96, means it [i.e. the correlation] will pass any statistical test for significance.
finally, I am at loss with my last equation, for minima, which [apparently] leaves no room for any AGW, unless you have an idea how to put it in whilst still maintaining 100% correlation?

Follow the Money
August 3, 2014 1:18 pm

The IPCC will never accept a 1C doubling sensitivity. It is too low for setting the prices of carbon offsets. More, the IPCC is an arm of the UN which takes a direct 2% skim off it carbon development mechanism (CDM) game, basically a phony “authentication” of emissions “reductions.”
You can bother with all the science you want. It’s about the money.

commieBob
August 3, 2014 1:20 pm

But there are also unknown unknowns, the ones we don’t know we don’t know.
http://en.wikipedia.org/wiki/There_are_known_knowns

Donald Rumsfeld got a lot of derision for the above quote. In time however most sensible folks came around to the realization that he was right. The things we don’t know that we don’t know are the ones we can’t prepare for and, therefore, come around to bite us in the rear.

E.M.Smith
Editor
August 3, 2014 1:22 pm

M’Lord,
I’d assert that the central climate fallacy is that an average of temperatures has any meaning at all, and the secondary fallacy is that temperatures say much of use about HEAT.
http://chiefio.wordpress.com/2011/07/01/intrinsic-extrinsic-intensive-extensive/
Since temperature is an intrinsic ( intensive ) property, an average of it is void of meaning.
Period. Full stop. End of it all.
Yet folks bang on endlessly about this data set (that is an average of temperatures) or that data set (that is an average of temperatures) or the Global AVERAGE Temperature; all the while blissfully ignorant of the fact that they are entirely void of meaning at the most basic level of physics.
That, then, leads to confounding this ‘temperature’ average with some notion of what is happening with HEAT. Repeat after me: “Temperature IS NOT HEAT.” Period. Full stop.
Now I do it too. Just as you do. Just as we all do. Because it is so easy to do, and everyone else is doing it… but it is, at the very core, a fallacy.
Recently someone said that the tops of clouds were cold so were not important in heat transport / radiation to space. Completely confounding temperature (cold) with heat ( IR to space) transfer. The reality is that the calories / lb of water that condensed in the cold altitude were in fact dumped to the near zero K of space, though at a low temperature.
Well, that’s my rant… Meanwhile, back at your posting:
The usual elegant turn of phrase. The expected sharp whit and clear mind. Yet…
Frankly, having tutored many a dolt though math in High School, I must assert that most of them don’t have a clue what they studied and past oh so long ago. Ask them who Pythagoras was or how to demonstrate the thesis and you will get blank stares, even from the bright ones. (Maybe things are different in the UK. Mum was from England, and she was a bit brighter than average on this side of the pond…) So I wonder how much it really taints their vision. Frankly, I think they largely just thinks “I’m a bit slow, and those folks are smart, so they must be right.” Ignoring the fact that all people are prone to error and all of us are a bit stupid at times.
Finally, on the question of the 30 year vs 17 (14, 15, 16, …) year halt of warming:
There are more than just 60 year cycles. The lunar tidal ocean mixing cycle has an 18 year component (Saros) and an every third time (so about 54 year) return of the same bit of ocean / land under it cycle:
http://chiefio.wordpress.com/2013/01/24/why-weather-has-a-60-year-lunar-beat/
and there are longer cycles out to 1500, 5000, and about 23000 years as various lunar wanderings stir the ocean differently. To only allow for the 60 (ish) year range is a bit limiting…
We get ENSO / El Nino steps up during long warming cycles. Then we get ENSO / La Nina steps down during long cooling phases. Right now, the warming has ended and we are in transition. The next phase is a long series of La Nina steps down. As that 1500 ish (or 1800 / 1200 year nodes with average about 1500 years) cycle swaps phase.
It is, really, all ‘wheels within wheels” and to only account for one of them is a, while an improvement over most, not quite complete.
My final bone of contention is just this: You average a bunch of data sets as though they were of similar merit. I know, I know… it’s a nice ploy to avoid being accused of playing favorites or of cherry picking… but really: NCDC, Hadley, and GISS are all largely just different perversions layered on top of the actual data. GHCN lays beneath them all. It is the only real “data set”. The rest are Data Food Products made via dodgy manipulation of GHCN. So by using them (and worse, averaging the three of them together and thus giving their manipulation more weight / merit) you validate them. They are fundamentally broken and are ‘economical with the truth’ of global temperature data, IMHO.
OK, enough of a rant. With all that said: Your posting is a pleasant read, and has a stronger awareness of the needs of the political / position game than I will ever have. I suspect you are well aware of everything I’ve said, but have balanced it against the needs of reality…. And it was fun to read….

Matthew R Marler
August 3, 2014 1:26 pm

Christopher Monckton of Brenchley: Mr Weinstein says I should not have assumed that recent warming was largely manmade. I made no such assumption: I said it was a possibility.
Please forgive my nitpicking a rather nice essay and comments, but you did indeed “make the assumption”, as you clearly stated, for the purposes of deriving its logical and mathematical consequences (We now make the assumption, for the sake of beginning somewhere, that all of the global warming over this 30-year period was attributable to us. ) Having written the proposition as an assumption, you did not write it as an “assertion” (literally a proposition asserted to be true), nor did you write any of the conclusions as “assertions”. The one unambiguous assertion is: However, the honest answer to Stacey’s question about what climate sensitivity is indicated by the temperature record of the past 30 years is, “We do not know”.

August 3, 2014 1:26 pm

These are not errors and mistakes, the claims of certainty by the warmistas. This is intentional obfuscation. They are not innocent misguided lambs, they are wolves.

Aphan
August 3, 2014 1:34 pm

H. Grouse…Science comes with qualifiers. Statements need to be examined in light of those qualifiers. The one you missed? “All things being equal…”

David S
August 3, 2014 1:42 pm

Unfortunately we no longer live in a society which operates on the basis of fact and reason. We live in a society that operates more like George Orwell’s 1984. It is a society where 2+2=5 because the government says so. If you don’t agree you will be tortured until you do agree. The government routinely changes history to support their agenda. They can make the record colder in the past as well as warmer in the present to exaggerate the warming trend. If you don’t agree with the claims of the alarmists you are written off as a quack. The torture hasn’t started yet, but some of the alarmists have suggested that those who don’t subscribe to CAGW be sent to re-education camps. The torture may start soon enough.

Justthinkin
August 3, 2014 1:53 pm

Great couple of articles the last few days,Lord Monckton. One silly(?) question though. How do you get lamdas,infinties,etc. to show up when typing.I understand the formulas and such,however learned them 35 years ago with pen and paper.I never did catch onto computer lingo.
And commieBob…BINGO. That is why,IMHO,curiosty must be the driving factor in science,however,it must be for the truth,not some pre-concieved idea or scam.We may never find it,but if we stop looking for it,we are pooched.

stan stendera
August 3, 2014 1:55 pm

This may be M’Lord’s best post ever. The quality of the comments proves the point.

August 3, 2014 2:03 pm

stan stendera says at August 3, 2014 at 1:55 pm…
Speak openly, please.
The comments are varied, true, and…
It is my failing that half the comments are mis-formatted…
But my intention was to be helpful.

John West
August 3, 2014 2:05 pm

“My own best estimate, for what little it is worth, is that a doubling of CO2 concentration would warm the world by about 1 Cº, if that.”
I think that’s a pretty darn good guess, but let’s not forget that while it is generally accepted that TCR is less than ECS that is not necessarily the case. Feedbacks over the long haul could still turn out to be net negative such that even if TCR ends up being greater than 1 K, ECS could still end up being less than 1 K.

August 3, 2014 2:08 pm

H Grouse says:
August 3, 2014 at 11:57 am
Then you need to explain why in the past 17 years an 10 months the increase of 35 ppm (10%) has had no effect.
—————
H Grouse, the simple explanation is, if I may, is that the author’s claim that an increase in CO2 ppm, be it 35 ppm or greater, ….. “will cause some warming” is based solely on the mathematical calculations via use of the physical IR absorption/emission properties of the CO2 molecule … and thus the mathematically calculated “warming” for a 35 ppm increase in CO2 is an insignificantly small number and is therefore non-measurable by use of present day instruments.
And if said calculated increase is non-measurable then it is permissible to claim it “had no effect”.
And if the truth be told, one can not actually measure the “warming” effect of the current 400 ppm of CO2 via use of present day instruments.
And neither is anyone capable of actually measuring the “warming” effect of a daytime increase of 400 ppm of H2O vapor (humidity) via use of present day instruments.
Said “warming” effect can be calculated ….. but it can not be measured via a thermometer.

JJM Gommers
August 3, 2014 2:11 pm

Patience, the Paris conference at the end of 2015 will be an unexpected cracker.

August 3, 2014 2:21 pm

The following is an honest and I summary of the state of climate science as expressed by three serious scientists :-
“Our knowledge of the causes of climatic change is still highly imperfect. It is not a field in which many people can dwell comfortably for a long time because it is almost entirely speculative…. No completely acceptable explanation of climatic change has ever been presented, and it is also clear that no one process acting alone can explain all scales of climatic change….. We are dealing with an immensely complex series of of interrelated systems: the solar system, the oceans and the land. It is thus unlikely that any simple hypothesis or model of climatic change will have very wide applicability…… It is clearly impossible at the present state of knowledge to make any safe prognosis of the climatic developments of the future. Because of the complexity of the atmospheric system and the large number of possible causes, it is difficult to assess and quantify the role that man has played, though certain mechanisms of man-induced climatic changes on a global ( as opposed to a micro-climatic scale) can be recognised.
Quoted from Anderson, Parker and Goudie ; Global Environments through the Quarternary.- Oxford University Press, 2007)

JohnWho
August 3, 2014 2:26 pm

Justthinkin says:
August 3, 2014 at 1:53 pm
Great couple of articles the last few days,Lord Monckton. One silly(?) question though. How do you get lamdas,infinties,etc. to show up when typing.

See if you get some help at the WUWT Test area here:
http://wattsupwiththat.com/test-2/

FreeHat
August 3, 2014 2:30 pm

Corallary of the h1 could be ‘There are known knowns and then there are our knowns. We hope everyone listens to our version’.

latecommer2014
August 3, 2014 2:37 pm

You stated “……for scientists have abdicated, forfeiting their high priesthood to far left politicians and profiteering environmental protection rackets” and I agree. I hope that others feel as I do, that the title of “scientist” is a privilege , no an honor, and we need to remember those who prostituted that honor for money and perverted fame. I for one have a list of these political whores, and will never, never consider them as scientists, and thus diminish the work I have done to proudly claim that title. I hope for an accounting at some future date, but until then I will not address them or consider them scientist .
Thank you for you lucid post.

JohnWho
August 3, 2014 2:42 pm

I’m sure many here may recall this:
“I know you think you understand what you thought I said,
but I’m not sure you realize that what you heard is not what I meant.”
Perhaps, in the “Climate Change” discussion, the Alarmist/Warmists could use:
“We know you think you understand what we think we know,
but we are not sure you realize that what we think we know is not what we know.”
Or something like that.

August 3, 2014 2:42 pm

milodonharlani says:
August 3, 2014 at 12:13 pm
Such a small increase is negligible, as indeed is 100 ppm, when considered against the 40,000 ppm of water vapor in the tropics.
——————
Right you are, milodonharlani .
Likewise, the “warming” effect of the current 400 ppm of CO2 is negligible when considered against the average 15,000 to 30,000 ppm of H2O vapor (humidity) that is per se resident in the near-surface atmosphere of the temperate zones.

richard verney
August 3, 2014 2:53 pm

Lord Monckton
For an article that is all about proof, I find the statement “Yes, liberating greenhouse gases by combustion will cause some warming – all other things being equal.” to be very strange.
First it is necessary to define what you mean, and the terms being used. Second, is it even possible for all things to remain equal given that you are consuming something by burning it, in the course of which oxygen from the atmosphere is being absorbed, resulting in the release of some oxygenated gas and water and heat? Third, where is the proof as to the correctness of that statement, or is the reader to take it as true on trust just because the theorem of Pythagoras is intrinsice honestum. It is right in itself.
As you know, when man first started burning CO2 in large quantities, say as from the late 1940s, it appears that the globe cooled, not warmed.
As you know, these past 17 years 10 months during which time about 30 to 33% of all manmade CO2 emissions has taken place, it appears that the globe has not warmed at all.
Both of these facts suggest that may be the statement “…liberating greenhouse gases by combustion will cause some warming – all other things being equal…” may not be correct
Of course, you may wish to argue that all other things were not equal in the two examples that I cite. If so, please identify those things and please detail how those things played out, not only during the periods of the two examples cited by me, but during the last 1200 or so years.
But I suspect that you (and indeed anyone) will be unable to detail how all other things may have changed over that period. That being the case, we simply do not know, for example, whether the reason why there has been no observed warming these past 17 years 10 months is because all other things have not remained equal (and have instead managed to cancel out the warming that would otherwise have taken place), or whether all other things have in fact remained equal, and the reason why there has been no warming is simply because ‘liberating greenhouse gases by combustion does not cause some warming’
PS. I agree with the title of the article.
PPS. I am not saying that liberating greenhouse gases by combustion does not cause some warming, merely I am questioning the proof behind the correctness of such assertion.

Paul
August 3, 2014 2:57 pm

Simple analytics and curve fitting aside, fact is that the bulk of scientific studies (not just modeling, but analysis of past climates) suggest climate sensitivity in the range of 2-5 C:
http://www.skepticalscience.com/climate-sensitivity.htm
Yes, there is a lot of uncertainty, and Monckton is entitled to his opinion of 1 C or less (based on some oversimplified math), but it is a gamble at this point, and I don’t think it’s prudent to wishfully hope for the low end.

H Grouse
August 3, 2014 2:57 pm

Samuel C Cogar says:
August 3, 2014 at 2:08 pm
” therefore non-measurable by use of present day instruments.”

Wait a minute (or 17+ years) , how about these “instruments?”
http://www.woodfortrees.org/plot/uah/from:1997/plot/uah/from:1997/trend

August 3, 2014 2:58 pm

But there are also unknown unknowns, the ones we don’t know we don’t know.
==========
the history of science shows that with every answer, we get two new questions. eventually, as you learn more about a subject, you discover how little you actually know.
climate science has so few questions because they have discovered so few answers. in time, as answers are discovered, there will be many questions raised.

August 3, 2014 2:59 pm

Strange how that atmospheric CO2 works. Since the first half of the 80s, it caused two substantial upward shifts in mean global temperature (about +0.2 degrees on each occasion), both occurring abruptly within the time span of a year and both times just at the transition between strong, solitary El Niños and the directly succeeding strong La Niñas – the world simply not cooling adequately with the cold ENSO events. Otherwise, nothing. Major action in two specific years, 1988 and 1998. No action whatsoever in the other 30.
http://i1172.photobucket.com/albums/r565/Keyell/NINOvsglOIv2_zps5be10b66.png
http://i1172.photobucket.com/albums/r565/Keyell/GWexplained_zps566ab681.png
Going back to 1970, there are three years of major upward shifts. And 40 years of nothing.
Now how is that CO2 climate sensitivity supposed to work again? Like that?

August 3, 2014 3:00 pm

I want to reiterate Lord Monckton’s final point that all this lucrative deceit is in pursuit of world government in the sense of political coordination, not some sort of world legislature. Invisible for the most part since we do not tend to get invited to the relevant meetings. I do love that Lord Monckton does tend to show up and occasionally sky dive in.
The reason education in the common law oriented Anglosphere countries all over the world is now geared to what I call (based on the Russian term behind it) the obuchenie mindset is so that the facts will no longer be allowed to disrupt the desired belief system. Constructivism in math and science that ignited the math and reading wars was never about how to teach the subjects as we were told. It is because an analytical mind full of facts is hard to get to act on the basis of these cultivated false beliefs.
UNESCO simply shorthands all this now as media education so that it can usefully pitch how to control what gets disseminated now as knowledge. Unless we begin to recognize where the actual playing field is–education and its focus on altering values, attitudes and beliefs in order to guide future perception and behavior–the Statists will win.
And then we all lose.

milodonharlani
August 3, 2014 3:03 pm

Paul says:
August 3, 2014 at 2:57 pm
There is no actual evidence for an equilibrium climate sensitivity higher than about one degree C for a doubling of CO2 from 280 to 560 ppm. The only way to imagine higher than that is to make assumptions about positive feedbacks which not only are not in evidence but which the preponderance of (IMO, all) available evidence is to the contrary.
Linking to SS doesn’t cut it. Make your own argument if you can, relying upon the lies there if you think they have some validity.

August 3, 2014 3:06 pm

Paul,
Look at the sidebars at the right where the various climate blogs are listed. Note that Skeptical Science is in a category by itself: unreliable. Now pause just a minute and reflect on the sort of impression that you make here by citing Skeptical Science as a reference. Now I will drive my point home: if you do not wish to appear foolish here, then do not cite Skeptical Science.

H Grouse
August 3, 2014 3:06 pm

Monckton of Brenchley says:
August 3, 2014 at 12:36 pm
“But it is not known whether all other things are equal,”

This is an effective disclaimer, which also applies to anything further that you have written with regard to sensitivity. For all you know, it could still be 3.5 degrees C for a doubling all other things being equal. …
Being non-committal is safe right?

mellyrn
August 3, 2014 3:07 pm

My own best estimate, for what little it is worth, is that a doubling of CO2 concentration would warm the world by about 1 Cº, if that.
If I dump my cup of coffee into the ocean, that will raise sea level. It is, after all, adding fluid to a fluid body. It certainly wouldn’t lower sea level. And it would not be a meaningful increase.
I don’t dispute CO2’s IR absorption. But my curiosity makes me wonder, if a doubling (400 ppm to 800 ppm?) would warm the world by even 0.5C, why would doubling it more than three times over (400 ppm to 4400 ppm) chill the world by 10C — from the 25C of Earth-normal conditions to the 15C (if not less) of the Andean-Saharan ice age?

August 3, 2014 3:11 pm

Very nice analysis. A continual problem skeptics have is explaining why we are skeptics. That includes explaining that yes, we do believe CO2 is a greenhouse gas and yes, the world has warmed. This essay explains very succinctly why skeptics are skeptical.

August 3, 2014 3:14 pm

analysis of past climates) suggest climate sensitivity in the range of 2-5 C:
==================
“suggests” is different than “shows”. Paleo data shows that warming causes CO2.

Dr. Deanster
August 3, 2014 3:29 pm

It would seem to me, that the Good Lord has made a mistake in his choice of time frames for data. For the Demonstration, he chose a “30 year” period, which represents only half of the PDO cycle. A more accurate assessment would have been to start his data at 1943-2003 …. that would be a full downward swing and upward swing of the PDO cycle.
While, IMO, that would have been a more accurate assessment of the temperature flux, it by no means is the truth, given that the data have been manipulated to the point of unrecognizable. But that being as it may be ….. and excluding any influence of the sun … just the PDO …. and using Woodfortrees as the metic, we see a rise in temperature from 1943-2003 of 0.35C.
Simple Math …. 0.35/60 * 100 years/century gives a wopping 0.58C per century. Yes, I know, I don’t quantify it to any CO2 ppm, because in the face of all feedbacks, that is irrelevant. But, if we are to assume that CO2 is the culprit, that is it’s maximum effect. …. 🙂

August 3, 2014 3:30 pm

Climate prediction on the Great Pause needs to take several thousand years of climate
evolution into account. To conclude that from an up-movement of temps over 150
years only that further global warming will occur is outright ridiculous. There are 5 major
climate drivers, demonstrated for over 20,000 years, and those drivers demand the
continuation of the Great Pause. This is not speculation but the result of solid calculations.

H Grouse
August 3, 2014 3:30 pm

Monckton forgot to mention that there is another case of the Pythagorean theorem
..
http://upload.wikimedia.org/math/5/1/f/51fb8faba1f42a03afa323aa13f61b94.png
Eculid’s 5th postulate has THREE cases.

milodonharlani
August 3, 2014 3:31 pm

mellyrn says:
August 3, 2014 at 3:07 pm
The Ordovician glaciation occurred when the sun was about 4% weaker than now is the excuse CACA spewers like to use, along with maybe there was a spike down in CO2 which proxies aren’t fine enough to record.
When that glaciation happened, CO2 concentration was in the thousands of ppm. It presumably fell as a result of colder temperatures, but remained higher than now, then rebounded, only to drop later with the spread of green plants onto the land.
The subsequent glaciation in the Carboniferous did apparently produce CO2 levels close to those of our present glaciation. Major Icehouse worlds appear to occur at about a 150 million year frequency. Shaviv, Svensmark & other cosmoclimatologists attribute this to the solar system’s passage through spiral arms of the galaxy.
Clearly, there is little correlation between CO2 changes (in the 100s to 1000s of ppm at least) & climate change, except that a colder world leads to lower levels & warmer to higher, so causation is reversed from that claimed by CACA spewers. There might however be a slight positive feedback effect from increased CO2 in a warmer world, & the reverse.
BTW, 25 C is not normal, but near the usual maximum on our homeostatic planet, at least for the past 540 million years. The present 14-15 C remains decidedly on the cold side, however. Very roughly, under Icehouse conditions, the range is about 10 to 17 C & under Hothouse, about 18 to 25 C. The range is often shown as 12 to 22 C, with occasional excursions above & below.

James Bull
August 3, 2014 3:34 pm

I saw a great T shirt the other day which had the following on it
“Science seeks to answer the unknown…. Today’s experiment is poking things with a stick”
Someone has certainly poked the warmistas with the stick of truth as they do seem to be getting angrier.
James Bull

son of mulder
August 3, 2014 3:40 pm

From http://www.diplomatie.gouv.fr/en/french-foreign-policy-1/sustainable-development-1097/21st-conference-of-the-parties-on/article/france-confirmed-as-host-of-2015
“France is already totally galvanized to ensure that a binding, fair, global climate agreement is reached in Paris in 2015, in order to limit global warming to 2ºC.”
If this is the answer, what is the question?
If you need clues why not read this drivel http://www.euractiv.com/climate-change/french-socialists-want-climate-c-news-531463

Monckton of Brenchley
August 3, 2014 3:42 pm

In answer to those who say climate sensitivity is 2-5 K, there is a growing body of literature – including a couple of papers by me – that find climate sensitivity to be in the region of 1 K.
And in answer to those who say the math in the head posting is over-simplified, it is based on an irreducible simple climate model that has the overriding merit of clarity. Its
predictions have so far been far closer to the truth than those of the general-circulation models.

Monckton of Brenchley
August 3, 2014 3:49 pm

The slayers seem to be out in farce again. Mr Verney asks what proof exists that greenhouse gases cause warming. I refer him to any elementary textbook of heat transfer.

August 3, 2014 3:59 pm

I see Moncton has finally decided to notice the blindingly obvious 60 year temperature cycle. It is about time that he and rest of the climate science community began to take notice of the equally obvious 970 year cycle which the 60 year cycle modulates.Any calculation of CS using the last 100 – 150 years of data that does not also include the underlying temperature trends relating to this natural quasi-millennial cycle in solar activity is really a waste of time. For estimates of the timing and amplitude of the coming cooling based on the 60 and 970+/- quasi-periodicities and using the 10Be and neutron count data as the most useful measure of solar activity. see:
http://climatesense-norpag.blogspot.com/2014/07/climate-forecasting-methods-and-cooling.html
Using these cycles there is no compelling empirical reason to assign any of the 20th century warming to human activity.
In 2013 in AR5 – WG1 the IPCC actually recognizes this.It says (Section 9.7.3.3)
“The assessed literature suggests that the range of climate sensitivities and transient responses covered by CMIP3/5 cannot be narrowed significantly by constraining the models with observations of the mean climate and variability, consistent with the difficulty of constraining the cloud feedbacks from observations ”
In plain English, this means that the IPCC contributors have no idea what the climate sensitivity is. Therefore, there is no credible basis for the WG 2 and 3 reports, and the Government policy makers have no empirical scientific basis for the entire UNFCCC process and their economically destructive climate and energy policies.
The whole idea of a climate sensitivity to CO2 (i.e., that we could dial up a chosen temperature by setting CO2 levels at some calculated level) is simply bizarre because the response of the temperature to Anthropogenic CO2 is simply not a constant, and will vary depending, as it does, on the state of the system as a whole at the time of the CO2 introduction.
Of course the IPCC SPM -an entirely political document – written as propaganda for the policies of Western governments – conveniently pays no attention to its own WG1 report in this matter.

Konrad
August 3, 2014 4:14 pm

Steve in SC says:
August 3, 2014 at 1:04 pm
“Then there are the major non-radiative transports: evaporation and convection up, advection across, and precipitation and subsidence down.”
That, sir, is the proverbial elephant in the proverbial room. Heat transfer has routinely been neglected by alarmist and skeptic alike.
——————————————————–
Steve, I would further this by saying it is a very, very old elephant.
When Callendar tried to revive the global warming scare in 1938, Sir George Simpson of the Royal Meteorological society pointed out –
“..but he would like to mention a few points which Mr. Callendar might wish to reconsider. In the first place he thought it was not sufficiently realised by non-meteorologists who came for the first time to help the Society in its study, that it was impossible to solve the problem of the temperature distribution in the atmosphere by working out the radiation. The atmosphere was not in a state of radiative equilibrium, and it also received heat by transfer from one part to another. In the second place, one had to remember that the temperature distribution in the atmosphere was determined almost entirely by the movement of the air up and down. This forced the atmosphere into a temperature distribution which was quite out of balance with the radiation. One could not, therefore, calculate the effect of changing any one factor in the atmosphere..”
It was true in 1938, it is still true today.

Hoser
August 3, 2014 4:23 pm

LMoB,
I was going to say don’t forget the unknown knowns, that is the things we ‘know’ that just aren’t so. But I think you make clear they are lying deliberately and therefore my point is moot.

August 3, 2014 4:36 pm

Reblogged this on sainsfilteknologi and commented:
Climate Fallacy

Konrad
August 3, 2014 5:10 pm

Monckton of Brenchley says:
August 3, 2014 at 3:42 pm
“And in answer to those who say the math in the head posting is over-simplified, it is based on an irreducible simple climate model that has the overriding merit of clarity. Its predictions have so far been far closer to the truth than those of the general-circulation models”
—————————————————————————————–
Viscount Monckton,
the problem is not that the math is over-simplified, but rather it is based on an utterly incorrect assumption. You have made the same mistake as the climastrologists in assuming that the surface of the planet is a “near blackbody” and would have an average surface temperature of around -18C in the absence of atmospheric cooling and DWLWIR. This is not a minor error, it is an error so critical that it invalidates not just AGW but the entire radiative GHE hypothesis. In this situation “all other things being equal” does not even rate as a fig leaf.
71% of our planet is covered in liquid water. Liquid water is not a “near blackbody” it is instead a “selective surface”. Selective surfaces typically have absorptivity and emissivity <1. Selective surfaces may have unequal absorptivity and emissivity. Selective surfaces may have transparency to various frequencies and a slow speed of internal non-radiative transport. Liquid water meets all of these conditions.
To demonstrate why understanding surface properties is critical to climate, examining the case of a simple selective surface will illustrate the point. Some polished aluminium alloys can have emissivity near 0.02 and absorptivity near 0.2. A polished aluminium planet without atmosphere exposed to around 240 w/m2 would have an equilibrium temperature of around 180C. Adding a strongly radiative atmosphere like ours would cool such a planet's surface. You can check this by placing a sheet of polished aluminium outdoors on insulating foam and measure its average temperature over diurnal cycle as our radiative atmosphere cools it. It will get nowhere close to 180C Tmax, let alone Tav.
Viscount Monckton, I urge you to examine the assumptions underlying your math. At the very foundation you will find the -18C assumption for the surface in absence of atmosphere and DWLWIR. My empirical experiments indicate that this may be in error by as much as 98C for the oceans. This may seem an incredible error, but you would be unwise to dismiss it without having replicated my empirical experiments or those from Texas A&M in 1965. Remember the blackbody calcs for the lunar regolith? Empirical results from Diviner mission showed those also to be in error by around 90C.
“All other things being equal” can never cover an error this large for 71% of our planets surface.
PS. Just because I am disproving the entire radiative GHE hypothesis, does not mean I can be safely accused of being a “slayer” who doesn't understand radiative physics. I have given the build instructions for the working empirical version of the two shell radiative model many times. It works fine with matt black plates separated by vacuum. I do not claim error with basic radiative physics, just its misapplication to atmospheric modelling. If you can challenge my claims without the “slayer” ad hom, please do so. But a challenge without empirical evidence that water is a “near blackbody” not a “selective surface” is no challenge at all 😉

NikFromNYC
August 3, 2014 5:11 pm

Paul, assuming you are a reasonable player in a real debate, consider that it took string theory physicist Lubos Motl a single post to reveal a pattern of deception and bias on the web site you link to, including this criticism of its high sensitivity presentation:
“Climate sensitivity is low: That’s a typical headline of some of my talks. Cook says that it’s 3 °C because of many reasons. The fact is that the direct calculation gives 1.2 °C and all balanced analyses of the Earth’s history, including very old geological data, suggest that this is about right, i.e. the net feedbacks are small, with an unknown sign. All papers or claims going to 3 °C or higher are fabricated and cherry-pick something to “hype” this number that almost certainly can’t reach 3 °C. The promoted positive feedbacks may be viewed as a quantification of the hype, exaggeration, and fraud: 70 percent of the IPCC figure for the climate sensitivity is fabricated because a higher value is favored by the “big picture” of the political process.”
http://motls.blogspot.com/2010/03/john-cook-skeptical-science.html
Another hundred skeptical arguments being called “myths” are skewered in that post. Though many are superficial it is the overall pattern of the serious rebuttals that makes your post hit a nerve here. Many of us became skeptics in part due to the unfair abuse we were subjected to on that site in a way that raised red flags that reason was not invited there whatsoever.
Evangelical “climate justice” Christian Cook pretends to reasonably survey the climate sensitivity literature while actually presenting a purely biased one, leaving out any sense of doubt on hundreds of pages of his site, indeed as Monckton here suggests reflects climate “science” in general. Cook omits papers that in fact simply measure sensitivity and find it to be low such as this recent paper:
http://www.worldscientific.com/doi/abs/10.1142/S0217979214500957
“The anthropogenic CO2 additional warming extrapolated in 2100 is found lower than 0.1°C in the absence of feedbacks.”
Cook only offers presents two options: models or the distant past, omitting the recent past, as a way to estimate sensitivity! Yet models have not been updated to match the recent past and in the distant past temperature leads CO2 spikes instead of follows them so sensitivity to it based on cause and effect is a false calculation when all it may be is a positive feedback rather than a driver.

phlogiston
August 3, 2014 5:17 pm

Lord Brenchley is right to point to the asinine arrogance of the warmists in considering the climate unknowns to be known. The often repeated chorus of “we cant find any other explanation for apparent climate warming so CO2 it must be” will hang like an intellectual albatross round their necks for all time. Its rather like glancing out of one’s kitchen window briefly then concluding “no elephants in my back yard – so elephants do not exist”.
So why and how does/can climate change if not by CO2 alone? How can I even entertain such a blasphemous thought? My own pet theory and I’m sticking to it – for now – is chaotic intermittence in deep ocean vertical mixing over log-log fractal timescales.
You can talk all you like about CO2. Or soot or methane or cows farting or CFCs. Or about the sun or magnetic fields. You can look for astrophysical cycles, clouds, radiation upward or downward or thermal bean-counting in the atmosphere.This is all fine and is your first amendment right in the USA an in some other countries also.
However if you want to understand climate and why the climate changes by itself without the need of external forcing, the primary place to look is in the ocean, not just the surface but all the way down to the bottom. The two key facts are (a) the ocean consists of two distinct circulation systems, the surface and deep circulations and (b) the deep water is much colder than the upper water. This strong temperature stratification means that any deep vertical mixing always moves heat down. What if the amount and rate of deep mixing varied globally on certain timescales? This would be reflected in climatic warming or cooling, relating to decreases or increases respectively in vertical mixing. Bear in mind the amount of heat in the oceans totally dwarfs that involved in all atmospheric procesess. The ocean is the dog, the atmosphere is the tail. Deep ocean mixing is a single parameter that has the power to regulate climate globally all by itself. And finally, it is known from experimental nonlinear-chaotic fliud mixing/circulation systems that turbulent mixing can be intermittent. Why should global deep ocean mixing be different? Where else need one look for the etiology of climate change?

Matthew R Marler
August 3, 2014 5:18 pm

Paul, your skeptical science link says this right at the top: Net positive feedback is confirmed by many different lines of evidence.
That statement is literally true, but the full case of the matter is that net positive feedback is also “disconfirmed by many different lines of evidence”. Whether the net feedback is + or – is one of the major known unknowns, and the topic of review articles in the major journals, such as Science and Nature. Biases like that are among the reasons that SkS is disparaged by Anthony Watt.
Here is a better link for the topic of climate sensitivity. http://www.climatedialogue.org/
Note well that the article by Nic Lewis shows how the higher estimates depend on Bayesian techniques that assign unrealistically (i.e. totally non-supportable) high amounts of prior probability to the region 4-8, 4-12, etc; and on other unrealistic assumptions. The more focused on actual data the methods are, the lower are the resultant estimates. The statistician F. J. Samaniego has shown in his book “A comparison of frequentist and Bayesian methods of estimation” that Bayesian estimates are better than frequentist estimates only if the prior distribution is sufficiently accurate. The uniform priors with 50% – 75% of probability above 4C are indefensible and do not meet his criterion.
There is at present no good evidence that the climate sensitivity to a doubling of CO2 concentration is likely above 2C, which SkS puts as its lower bound.

Steve Oregon
August 3, 2014 5:25 pm

CMoB said, “The answer – though you will very seldom hear it from the Church of Thermageddon – is that there are far too many unknowns. The biggest outright lie in the IPCC’s 2013, 2007, and 2001 reports is the notion that we can determine with any confidence the fraction of global warming over recent decades that is attributable to us.”
Hey I’m just a Mr. Nobody layperson but I say it is not possible for “them” to determine such a thing.
With only 0.117% of the greenhouse effect due to atmospheric CO2 from human activity it is preposterous to claim they can measure any human warming impact at all let alone what fraction of supposed warming is attributable to us.
You are right CMoB. It is the biggest outright lie.
The rest of the biggest lie is this:
http://earthobservatory.nasa.gov/Features/WaterVapor/water_vapor2.php
“Warming due to carbon dioxide emissions from fossil fuel combustion evaporates even more water, increasing the thickness of the blanket, which leads to more heating, which leads to more water vapor… The loop is called the water vapor feedback, and it has the potential to be a serious problem.”
They can’t possibly know that increased fossil fuel use caused the warming to begin with when only 0.117% of the greenhouse effect is due to atmospheric CO2 from human activity.
NASA goes on to say,
“Sherwood explains. “If you have enough of this positive feedback, then of course the whole climate system would be unstable.” Today’s climate, he quickly adds, is not unstable. “But as you pile on more and more of this sort of thing, you get closer and closer to an unstable situation. So if the climate is unstable, small differences in how strong these feedbacks are can become relatively important, more important than you might think.”
I can read that as an admission that the warming and instability is yet to occur. That a piling up can someday become more important.
The EPA seems to concur:
http://cdiac.ornl.gov/pns/current_ghg.html
“Concentrations of ozone and water vapor are spatially and temporally variable due to their short atmospheric lifetimes. A vertically and horizontally averaged water vapor concentration is about 5,000 ppm. Globally averaged water vapor concentration is difficult to measure precisely because it varies from one place to another and from one season to the next. This precludes a precise determination of changes in water vapor since pre-industrial time. However, a warmer atmosphere will likely contain more water vapor than at present.”
“A warming atmosphere will?”
IMO NASA and the EPA are hypothesizing about a future scenario.
Which begs the question. How has so much been so confidently attributed to that which has is yet to occur?
Man-made CO2 is only 3.225% of atmospheric CO2 96.775% is natural.
Total human greenhouse gas contributions add up to about 0.28% of the greenhouse effect.
0.117% of the greenhouse effect is due to atmospheric CO2 from human activity.
So there could not have been, or at least scientists cannot know that there has been any initial warming do solely to the increased fossil fuel use.
AGW is not possible because CO2 emissions did not cause warming or the evaporation of more water.
http://www.climate4you.com/GreenhouseGasses.htm

mellyrn
August 3, 2014 5:36 pm

The slayers seem to be out in farce again. Mr Verney asks what proof exists that greenhouse gases cause warming. I refer him to any elementary textbook of heat transfer.
In other words, you referred him to more theory, which allows you to skirt the real-world absence of warming following CO2 increases. Usually, we check our theories against the real world.
Before you get snarky again on me, though, consider this analogy: someone who understands gravity superbly well, but who has never — YET, a very important “yet” — encountered the concept of buoyancy. He is going to insist, based on magnificent, excellent and true theory, that my grandson’s balloon will fall down once it is released.
Buoyancy is, of course, a gravitational effect. It’s just that my hypothetical gravity expert has not — YET — reasoned his way to the buoyancy principle. If he clings to what he already knows, and refuses to look at balloons that fly up and away from careless little boys, he never will.
I say your theory is perfectly fine (for what my little opinion is worth). I also see perfectly well, thank you, that (apologies to Inigo Montoya) it does not seem to do what you think it does.
It doesn’t mean you’re wrong. It means there is something a damned sight more interesting going on here. CO2 “should” warm an atmosphere. Venus says it doesn’t, the Ordovician ice age says it doesn’t, the last 17.833 years say it doesn’t. Gravity (at first blush) says the balloon “should” fall down; the balloon floating away into the clouds says, go figure.
Where is your curiosity, Sv Monckton?

milodonharlani
August 3, 2014 5:43 pm

mellyrn says:
August 3, 2014 at 5:36 pm
The Ordovician ice age might have lasted longer & been colder with CO2 at 400 ppm instead of 4000 ppm or more. The glaciation was brief but fairly intense, thanks to so much land over the South Pole. What can be said is that CO2 is not the “major driver of climate change”. Its level is more of an effect than a cause.

mellyrn
August 3, 2014 5:51 pm

Konrad! That’s the sort of “damned sight more interesting” I could wish for! Thank you ever so.
(Should I roof my house with aluminum panels instead of galvanized steel ones, for a cooler home in the summer, then?)

August 3, 2014 5:52 pm

Monckton of Brenchley:
Are you asserting that when the change in the equilibrium surface temperature is divided by the change in the logarithm of the CO2 concentration the result is a constant? If so, please support this hypothesis..

milodonharlani
August 3, 2014 5:59 pm

Konrad says:
August 3, 2014 at 5:10 pm
Valuable comments, but please consider that oceans are not just water. Their surfaces are often turbulent or covered in ice. They contain many dissolved salts & minerals & perhaps most important of all, living things. Their currents & tides are affected by islands & continents. Their depths & optical properties vary. Nutrients upwell & surface salinity & temperature fluctuate. All these factors affect their absorption & reflection profiles as opposed to pure water. Not to mention the effect of latitude, ie angle of incidence of incoming radiation.

u.k.(us)
August 3, 2014 6:17 pm

@ mellyrn.
Here’s some background music, minus the balloons, full of misery.
http://www.youtube.com/watch?v=ODHhWcEdrvg

Konrad
August 3, 2014 6:17 pm

mellyrn says:
August 3, 2014 at 5:51 pm
————————————-
I would actually recommend a coating of white titanium oxide paint. This is also a “selective surface”, but has the unusual property of having an IR emissivity grater than its UV/SW absorptivity.
Have a look at photos of the old apollo service module. Those white ribbed panels are actually radiators for the cryogenic systems. In sunlight they preform better than matt black.

mellyrn
August 3, 2014 6:22 pm

The Ordovician ice age might have lasted longer & been colder with CO2 at 400 ppm instead of 4000 ppm or more.
Thank you, milodonharlani, but with Venus at 960,000 ppm CO2 and no hotter than simple proximity allows (at one atm pressure — apples to apples), I have to doubt it.
I can say that pouring a cup of water into the ocean will raise sea level — but a swimmer in Japan getting out of the water just as I do will more than negate it. CO2 apparently heats gas in a chamber in a lab. It doesn’t heat a planet, not even when it’s nearly pure (barring a weird, coincidentally-perfect albedo effect that is more effective when there is less of it). The theory is clearly missing something.
Clinging to the theory-as-it-is serves no one.

Konrad
August 3, 2014 6:43 pm

milodonharlani says:
August 3, 2014 at 5:59 pm
————————————-
You raise a number of valid points concerning the oceans.
“Their depths & optical properties vary.”
This is particularly important as it effects depth of UV/SW absorption. Transparency and depth of absorption is one of the critical selective surface effects that means the oceans can be heated far above theoretical blackbody temperature by the sun alone. This simple experiment you can build for yourself demonstrates this effect –
http://oi62.tinypic.com/zn7a4y.jpg
Both samples have equal UV/SW absorption and equal IR emissivity yet sample A reaches the higher temperature when exposed to sunlight. The only difference is the depth of UV/SW absorption.
This paper concerning ocean turbidity and temperature effects of varying depth of absorption was written by scientists who clearly understood selective surface effects (ie: not your average climastrologists) –
http://www.ldeo.columbia.edu/~csweeney/papers/SWpen_Sweeney.pdf
“Not to mention the effect of latitude, ie angle of incidence of incoming radiation.”
This does effect UV/SW absorption as can be seen by the sun glare of the oceans when viewed from space. But the interesting point here is that viewing angle and surface roughness of the oceans also effects apparent emissivity. This was the big “clue” that climastrologists missed. Materials that change emissivity with viewing angle and surface roughness could not possibly be a “near blackbody”.

milodonharlani
August 3, 2014 6:44 pm

mellyrn says:
August 3, 2014 at 6:22 pm
Venus lacks water vapor & its hotter upwelling photons (with different mix of energies than the LW IR from the surface of Earth) have to fight through a much thicker mass of molecules to get to the one atmosphere level. The example is IMO instructive, but can’t make more of it than justified, given all the differences between the two planets.
Mars after all also has a mainly CO2 atmosphere.
But I agree that lab measurements of CO2 IR absorption don’t translate directly into the real world, in which in some environments it might be that more CO2 has a cooling effect, as perhaps in the warmest parts of the moist tropics.

Alcheson
August 3, 2014 6:45 pm

Nice article. As you have correctly pointed out there are still several unknowns, especially when it comes to feedbacks. However, as our planet appear to have a very stable climate system, it much more likely than not, has net NEGATIVE feedbacks or it almost surely would have either frozen solid or become inhospitably hgot long ago. To make the initial assumptions that all of the important feedbacks were positive from the Get-Go,.. is pretty much proof that these people pushing this had an agenda from day one.

Mervyn
August 3, 2014 7:19 pm

Albert Einstein was right when he said, “We still do not know one thousandth of one percent of what nature has revealed to us.”
Sadly, those promoting the dangerous human-caused global warming supposition think Einstein was wrong. Even worse, individuals like President Obama and Prince Charles actually believe the science is settled when it comes to climate change. How can such ignorance be explained?

August 3, 2014 7:33 pm

mellyrn says, August 3, 2014 at 5:36 pm:
“I say your theory is perfectly fine (for what my little opinion is worth).”
It works fine inside a closed glass box in a laboratory experiment. It doesn’t work fine out in the real, open, large-scale surface/atmosphere system. Not even in theory. Because that’s a completely different situation. The two are not analogous. For instance, there is no rigid lid. What works on the blackboard doesn’t necessarily work in nature.

Robert of Texas
August 3, 2014 7:51 pm

I love your posts, they are a pleasure to read.
I am wondering if this particular sentence contains an error:
“In 2001 the IPCC said it was 66% confident we had caused most of the warming since 1950; in 2001, 90% confident; in 2013, 95-99% confident.”
Should the second part “in 2001, 90% confident;” be 2007 instead of 2001?
Again, thanks for the excellent post.

AlexS
August 3, 2014 8:41 pm

“Thermageddonites”
I like this.

sinewave
August 3, 2014 9:17 pm

Paul says:
August 3, 2014 at 2:57 pm
As others have pointed out SkS is hardly a credible source to reference, however, your link to it in the comments of a WUWT post probably increased traffic to that site by an order of magnitude today…

richard verney
August 3, 2014 11:13 pm

mellyrn says:
August 3, 2014 at 5:36 pm
//////////////////
Don’t bracket me as you did in your first paragraph. I am not alligned to that group.
I am a sceptic. That means that I am sceptical of each and every argument in facvour of AGW, and sceptical of each and every argument against AGW.. I don’t take anything for granted without proof. Obviously, I consider that some things are more likely correct than other things, but that is as far as I would go; without proof nothing should be assumed to be fact. According to the theory of the half life of facts, about 50% of things that we consider today to be fact, will in about 10 years time, not be considered as fact.
I fully accept the laboratory physical properties of CO2. The issue here is not its laboratory characteristics, but rather how those characteristics play out in the real world environment of planet Earth. And that was what my post was getting at.
Increasing CO2 has one of three possibilities. It may warm the planet, it may have such an insignificant effect that it is so small that we cannot measure what it does. It may cool the planet. We have yet to work out which of these eventualities occurs in real world conditions.
What one can say is that there are a lot of problems with the AGW theory, notably, it appears that CO2 lags temperature on every time scale, thereby appearing to be a response and not a driver. There is no first order correlation between CO2 levels and temperatures in any data set (admittedly the temperature data sets are very poor and not fit for purpose). CO2 cannot sometimes heat the atmoshere and not heat the oceans, and at other times heat the oceans and not the atmosphere; whatever its properties are, in the real world environment, these properties do not switch like that. There has never been runnaway warming when in the past there have been high levels of CO2 and/or high temperatures (notwithstanding that when there were high temperatures there was almost certainly high levels of water vapour given the much warmer ocean conditions which according to GHW theory should have led to further warming).
I agree with you, that a proper evaluation of Venus also suggests that there is a problem with CO2 GH warming because the surface of Venus does not appear to be warmed by reflected Solar from the surface of the planet being ‘trapped’ and then re-radiated downwards by all the GHGs in the atmosphere because little if any Solar actually reaches the surface to be reflected from it – the vast majority of incoming Solar is simply reflected at or towards the TOA. The isothermal charcteristics where there is no significant diurnal difference (notwithstanding that night on Venus is some 122 Earth days which is a long time to go without ‘daylight’), nor eqitorial/polar difference (unlike that seen on planet Earth) suggest that some other operation is at play in producing the high temperatures seen on Venus. CO2 does not appear to be adding anything to the the apples to apples comparison that you give. Of course the atmosphere on Venus has a lot of thermal inertia, which may partly explain matters, but so do the oceans on planet Earth, and one can see a diurnal response in the oceans even though ‘nightime’ on Earth is only 12 hours, not about the 122 Earth days as seen on Venus. The thermal inertia of the Venusian atmosphere is therefore unlikely to explain the isothermal characteristics of that planet.
As Konrad often comments the oceans are a real problem for AGW, because of their selcective surface nature, one aspect of which is their absorption characterics of LWIR. Given that DWLWIR is omnidirectional about 80% of all DWLWIR is fuly absorbed in just 3 microns of the ocean, and so far no one is able to explain how the energy absorbed in the top few micron layer can be diluted and disipated to depth (and thereby spread over a large volume) at a speed quicker than the theoretical energy absorbed within the first few micron layer would power evaporation. IF DWLWIR has sensible energy in the real world environs of planet Earth it would likely result in copious quantities of evaporation of the oceans (which would tend to boil off from the top down), which copious quantities of evaporation are not detected. Solar does not present this problem since the energy from Solar is not absorbed within just a few microns but for the main part over a depth of at least 1 metre thereby greatly diluting the energy so that the oceans can be kept warm without boil off.
If DWLWIR has sensible energy capable of performing sensible work in the real world environs of planet Earth, it is astounding that there are no large scale projects seeking to harness that energy. Which energineer would opt for Solar as a power source, when according to K&T, DWLWIR has about 100% more power and is a constant available 24/7 come rain or shine? The appeal of DWLWIR as a constant power source would favour this, and yet there appears to be no significant research into harnessing it, thereby suggesting that it is viewed as nothing more than a signal incapable of performing sensible work in the real world environs in which it finds itself here at the surface (or near surface) of planet Earth .
My earlier comment was more directed at the iconsistency with the quoted sentence and the thrust of the article, namely that we do not know things, that we think that we know things which we do not know, and that we should not accept things as correct without proof that they are correct. I accpet the thrust of the article, but I consider the sentence “Yes, liberating greenhouse gases by combustion will cause some warming – all other things being equal.” to be inconsistent with the thrust of the article. The article would have read better without that sentence.
No one doubts that burning fossil fuels is releasing CO2 (whether that is the cause of the rise in the levels of CO2 in our atmosphere is moot, but a prima facie case is made out that it at least in part explains the increase seen since the late 1950s). The isue is the effect of this, and the article could simply concentrate on the evidence of response.
Personally (as I frequently comment), I do not consider that one can derive a figure for climate sensitivity from observational data until such time as absolutely everything there is to know about natural variation is known and fully understood; until such time as we are able to identify each and every constituent forciing that is encompassed within natural variation, and the upper and lower bounds of each and evbvery constituent forcing. The reason for this obvious, until we can say to what extent natural variation explains the temperature change (and we cannot do that until we can identify all the forcings and to what extent they are in play at any given time), we cannot seperate and identify the signal (if any) from CO2. Put simply we cannot say that CO2 has caused this change or that change until natural variation is removed from the equation.
Climate Sensitivity is not something capable of some theoretcial calculation devoid of observational basis since it is looking at the real world response to CO2 in our atmosphere, which real world response can only be ascertained bu actual observation. Presently discussions suggesting that we can assess Climate Sensitivity are a disengenuous exageration of our abilities and understndings, and, of course, are based upon data that is not fit for purpose. .

Monckton of Brenchley
August 3, 2014 11:21 pm

In answer to “Justthinkin”, special characters such as Greek letters and mathematical symbols are available in Word in two ways. The first and simplest is to hit control-I followed by S. That displays a table of symbols. The second is to use the clunky, ugly, error-prone equation writer, which will be found on the Insert tab. To toggle in and out of subscripts in ordinary text-writing mode, use control-O then F then B. For superscripts, control-OFP. The Word interface was designed by an idiot.
I agree with “latecommer2014” that some scientists have abused their white coats with leaky biros sticking out of the front pockets. It is time for the same law to apply to them as to everyone else.
“Kristian” is right that that the stochastic changes in global temperature do not sit well with the monotonic changes in CO2 concentration. Since absence of correlation necessarily implies absence of causation, we know that the stochasticity of the temperature changes is not caused by the monotonically-changing CO2 concentration. Therefore, the sudden temperature changes to which “Kristian” refers are caused by something else; and, until we have identified clearly what that something else is, and quantified it, and subtracted it from the temperature trend, determining the fraction of temperature change attributable to CO2 concentration change is impossible.
Mr “Grouse” has choisen to take a qualifier from one part of what I wrote and apply it inappropriately to another. On a separate point, he says there are “three cases” (whatever that may mean) of Euclid’s fifth or parallel postulate, which, though it be equivalent to the Pythagorean theorem, applies unmodified only in the Euclidean plane. It is necessary to modify that postulate to apply it in the elliptic and the hyperbolic planes.
“Mellyrn”, yet another slayer, continues to sow confusion, this time by suggesting on the basis of paleoclimate evidence that CO2 concentration increase causes cooling, not warming. In the Neoproterozoic era 750 million years ago, there was at least 30% CO2 concentration in the atmosphere, yet glaciers came and went, twice, at the Equator. Such evidence does not tell us CO2 has no warming influence; but it tells us that, all other things being equal, it is likely to be small.
“Dr Deanster” has failed to read the head posting carefully. To cancel the cyclical temperature effects of the PDO, one may take either periods that are multiples of 60 years in length or periods centered about a phase-transition from the warming to the cooling phase of the PDO or vice-versa.
Mr Seifert makes the important point that one should keep the influence of Man’s small perturbation in atmospheric composition in proportion to the larger influence of major climate drivers such as the sun, the oceans, clouds, and non-radiative transports.
Dr Page says I have “finally decided to notice” the 60-year cycle of the PDO. I have, however, referred to it frequently in previous postings going back many years, have frequently discussed in detail its influence on the global temperature record, and have also discussed it with the authors of the leading scientific papers on the subject.
I am grateful to “Konrad” for having unearthed Sir George Simpson’s quotation. He was right in 1938 and remains right today. However, he is incorrect about the elementary physics that determines the effective radiating temperature of the Earth. If the Earth’s albedo were kept at its present value of 0.3, an Earth without an atmosphere (or with an atmosphere but without greenhouse gases) would have an effective radiating temperature of 255 Kelvin. Particularly with respect to the long-wave radiation with which we are chiefly concerned in studying the greenhouse effect, the Earth is a blackbody or very nearly so. The emissivity of nearly all of the Earth’s major surfaces, including that of the ocean, is sufficiently close to unity that very little error arises by assuming that it is unity.
“Phlogiston” says one need look only to the oceans to explain climate change. One should certainly not ignore them: there are, for instance, some 3.5 million subsea volcanoes, none of which is routinely monitored for its heat output. However, there are other major influences other than greenhouse gases: the sun, the clouds, and the non-radiative transports, for instance.
“Steve Oregon” is incorrect to say that man-made CO2 is little more than 3% of atmospheric CO2. It is more like 40%. He is right, though, that the climate is stable: it has shown itself near-thermostatic for at least 810,000 years. Therefore, there is no imminent danger of instability arising from our having altered the composition of the atmosphere by 1 part in 2500, which we may achieve by the end of this century if CO2 concentration continues to rise on the business-as-usual scenario.
“Mellyrn”, a serially tedious slayer, says that my referring yet another tedious slayer to an elementary textbook on heat transfer I am referring him to what he calls “mere theory”. However, the relevant theory, like all theories in physics, is of course rooted in observation followed by experiment, as the head posting explains, and there are plenty of experiments demonstrating that there is such a thing as a greenhouse effect, though quantifying its application in the real climate is difficult – which, after all, is what the head posting was about.
“Mellyrn” also continues to be wrong about Venus. I have already given the relevant value for the effective temperature of Venus, and it is plain that that temperature is considerably below the temperature at a pressure-altitude equivalent to that of Earth, demonstrating yet again the reality of the greenhouse effect. Moderators, please refer this particular commenter to Anthony, who has a strict policy of not giving space to slayers who question the basic science of the greenhouse effect.
Mr Oldberg asks whether I am asserting that when the change in equilibrium surface temperature change is divided by the change in the logarithm of the CO2 concentration change the resulting climate-sensitivity parameter is constant. Sigh. Since surface temperature change at equilibrium is an unknown variable, and since future CO2 concentration change is also a variable, the climate-sensitivity parameter that is the ratio of the two can scarcely be constant, now, can it? The head posting made it plain that it is not possible to determine climate sensitivity on the information now to hand: therefore, it is not possible to determine the equilibrium climate-sensitivity parameter; therefore, I cannot have given the slightest ground for anyone to suspect that I regarded it as constant.
Robert of Texas has indeed noticed an error in the head posting: “in 2001, 90% confident” should read “in 2007, 90% confident”. The IPCC’s “90% confidence” that recent warming was mostly manmade was reached by show of hands among scientifically-illiterate government representatives. China had (correctly) wanted no such estimate; most of the rest of the world wanted 95%, as that would keep the gravy-train rolling for another few years; so 90% was settled upon. All such estimates are, statistically speaking, nonsense.

August 4, 2014 12:00 am

richard verney says:
I am a sceptic.
I think that is correct. Richard Verney has always appeared to me to be a very reasonable scientific skeptic and a good analyst. I would not classify him as any kind of Slayer. Just MHO.

richard verney
August 4, 2014 12:44 am

Monckton of Brenchley says:
August 3, 2014 at 3:49 pm
The sl**ers seem to be out in farce again. Mr Verney asks what proof exists that greenhouse gases cause warming. I refer him to any elementary textbook of heat transfer.
////////////////////////
Lord Monckton
I have noticed a distict change in your style these past 3 or 4 years, one of which is a tendency towards ad homs, and another is adopting a supercilious attitude towards others. If you are going to level accustaions at others or otherwise disparage them, make sure you have evidence supporting your position; I am not aligned to that groug and I have read elementary textbooks on heat transfer (many of which do not discuss GHGs). However, I do not take something as fact simply because I read it in a book (or online), without appropriate supporting proof.
For one who prides themselves on logic, your article is a logical fail to the extent that the sentence that I quoted is one of the many things that we think that we know, but may in reality be a thing that is incorrect; arising from an over extrapolation of other facts that we know. Over extrapolation of facts and data, unfortunately, is not an uncommon failing, especially in climate science. Your article (correctly) argues that we should not simply assume but instead we should look for actual proof, and yet the assertion in the sentence that I quoted is a matter on which there is no proof, and one on which no one has offered any credible proof; whilst the laboratory characteristics of CO2 have been observed and tested and evaluated, it is not known how the release of CO2 plays out in the real world environs of the atmosphere of planet Earth at about 280ppm and more.
The sentence that I quoted is in itself logically inconsistent, since whenever you effect change, by the very nature of change, things do not remain equal; although I understand what you meant by your sloppy language, but nonetheless the sentence possesses an inherent inconsistency such that it is a logical fail.
Of course you could have sought to set out the proof backing up your assertion. You could have sought to demonstate the entire temperature profile of the Holocene and the warming caused by variations in CO2 in the light of all other things, but you did not. I would suggest that the reason that you did not do this is because you are unable to do so.
I would go on to suggest that the reason for that is that we simply do not know what other things are, nor to what extent each of these other things is in play at any given moment of time, and we are therefore unable to establish whether in the real world environs of the atmosphere of planet Earth, CO2 has any measurable input on temperature. Given that it appears that CO2, on all time scales, lags temperature, it does not look that promising that the release of CO2 drives warming as you assert. i am not going as far as saying that the lag proves that it does not, since the data sets are unfit for purpose and do not lead to informed knowledge or understanding.
i would suggest that your article would have read better without that sentence, which in practice is otiose. Unfortunately, the more one repeats an unknown ‘fact’ as if it is a true fact, the more entrenched and the more difficult it becomes to see the light of truth.
You state (correctly in my opinion) “However, the honest answer to Stacey’s question about what climate sensitivity is indicated by the temperature record of the past 30 years is, “We do not know”.” I would suggest the reason you came to such conclusion is because we do not know whether “…liberating greenhouse gases by combustion…cause some warming – all other things being equal.” The former (correct) assertion, reveals the problem with the latter (unproven) assertion.
You state “Then there are the major non-radiative transports: evaporation and convection up, advection across, and precipitation and subsidence down.” and this may lie at the crux of the issue since it may well be the case, in say 30 or 40 years time, when we have a better state of knowledge and understanding of things, that it is shown that the climate is governed almost exclusively by non-radiative transports and that radiative transports play an all but insignificant role.
I do not make any prediction of the future climate, but even die hard warmists such as Julia Slingo are suggesting that there may be no resumption to warming before 2030. If that is how matters pan out, by that time about 80% of all manmade CO2 emissions will have been released during which period no warming will have taken place, such that I predict that in 2030 not only will there be a significant reassessment of Climate Sensitivity 9suggesting a low sensitivituy), but also a reassessment of the very foundations on which GHG theory is based. One reason behind a low sensitivity may be that there are fundamental flaws with the very theory itself.

rogerknights
August 4, 2014 12:55 am

Mervyn says:
August 3, 2014 at 7:19 pm
. . . individuals like President Obama and Prince Charles actually believe the science is settled when it comes to climate change. How can such ignorance be explained?

They probably were subjected to indoctrination sessions with seemingly know-it-all warmist scientists and activists. They didn’t exercise due diligence and give the other side a hearing. They thought that SkS’s list of straw man refutations was conclusive. I suspect this is what happened to the bigshots now sounding off as part of the Risky Business campaign.

richardscourtney
August 4, 2014 1:16 am

rogerknights:
At August 4, 2014 at 12:55 am you say of President Obama and Prince Charles

They probably were subjected to indoctrination sessions with seemingly know-it-all warmist scientists and activists. They didn’t exercise due diligence and give the other side a hearing. They thought that SkS’s list of straw man refutations was conclusive. I suspect this is what happened to the bigshots now sounding off as part of the Risky Business campaign.

Please remember that it is not their job to “exercise due diligence”.
Politicians are required to assess and to balance information of every kind from every possible source: that is their job when they are elected. And they cannot know everything about everything.
Information is supplied to elected politicians by their personal advisers and their civil services. When their information suppliers become biased then the information they get is biased with probable result that their decisions are flawed, and there are many examples of this (e.g. WMD in Iraq).
Good politicians always remember the possibility that their information may be wrong and/or incomplete so they always “consider that ye may be wrong”. Those who forget to “consider that ye may be wrong” get decapitated either literally or metaphorically.
Richard

thingadonta
August 4, 2014 2:07 am

“In 2001 the IPCC said it was 66% confident we had caused most of the warming since 1950; in 2001, 90% confident; in 2013, 95-99% confident. All of these confidence values are direct lies”
I think you will find that in bureaucratic speak, ‘confidence values’ really mean how much the organisation wants to believe it, how much they need to believe it, how long they have believed it, and how much good it gives to the organisation to continue to believe it, not how confident they should be in something based on actual data. It’s a form of projection.
What they are essentially saying is they are 95-99% sure the organisation would like this to be true. Otherwise what the hell have they been doing all this time?. A bureaucrat has to look back and see progress, which often means more surety in the absence of anything tangible.
One could write a book on how bureaucracies often become more sure of themselves over time. It doesn’t have anything to do with data or reality, but how organisations often evolve to be more sure of themselves. Which is partly why we invented democracy, destroying surety which isn’t justified.

Steve C
August 4, 2014 2:17 am

Lord Christopher, regarding “the thousand interlinked supranational bureaucracies” you mention. Do you have any thoughts on how these entities might be rendered subject to our control, or, ideally, removed?

phlogiston
August 4, 2014 2:23 am

@Monkton of Brenchley
Thanks for your feedback. Yes I do go too far – for rhetoric effect – to say climate is determined only in the ocean. Of course there are other factors. However deep vertical mixing in the oceans, by various mechanisms and integrated globally is an important part of the climate equation which should not be overlooked, as it routinely is.
if we consider ocean (bilayer) circulation to be a nonlinear chaotic process characterised intermittently by turbulence, then like many known nonlinear systems it could be periodically forced by external factors such as solar derived variation in heat input or variation in cloud cover. While there are several plausible astrophysical climate forcings, the difficulty in finding convincing correlations with climate could be because these cyclical inputs are not acting directly on climate but indirectly by for instance weak and complex periodic forcing of oceanic oscillations.

Stacey
August 4, 2014 2:30 am

Christopher
Thank you for the response and by the way the last time I looked I was a man 🙂

August 4, 2014 2:40 am

Richard Verney:
You are right. A laboratory vessel is not the earth’s atmosphere and cannot duplicate natural processes yet most of those who call themselves skeptics accept the flawed assumption that the results of the vessel can be straightforwardly applied to the real thing. Hence the idea that the absorbency spectrum of CO2 will somehow assert an effect on climate, sooner or later.
Monckton is one such skeptic. His comment about Pythagorian absoluteness shows that he understands

Konrad
August 4, 2014 2:53 am

Monckton of Brenchley says:
August 3, 2014 at 11:21 pm
——————————————-
Viscount Monckton,
firstly, thank you for taking the time to write responses to so many of the commentators on this thread.
However, again I would urge you to have a care when responding to those who challenge the idea of a NET radiative GHE and calling them “slayers”. This is little better than referring to sceptics as “holocaust deniers”. There are few “slayers” left, as most at WUWT were “false flag” and little energy remains for these games in the dying days of the hoax. All that we are left with are a few sad “sleepers” hoping to keep sceptics on the lukewarmer path so as to engineer a “less than we thought” soft landing.
As to your 255K claims, I say crap! I work in engineering, not science. I am an empiricist, not a mathematician. Maths is not physics. Math can model physics, but it can also model the non-physical. And this is just what you (and 97% of climastrologists ) have done. I am an empiricist. You are a mathematician. But this is a question of physics. Therefore you can never defeat me. Ever.
Have you personally measured the emissivity of water with background IR minimised? No, you haven’t, but I have –
http://i61.tinypic.com/24ozslk.jpg
– Guess what? With background dropped to -40C, apparent emissivity of water at 40C drops to below <0.8. ( you ain't in the running Christopher, nowhere close )
But is the IR emissivity of water being lower than UV/SW absorptivity all of the picture? No. As I have empirically demonstrated many times –
http://oi61.tinypic.com/or5rv9.jpg
http://oi62.tinypic.com/zn7a4y.jpg
– depth of UV/SW absorption matters. It matters a lot.
Viscount Monckton, If you wish to defeat me, you need to be a better empiricist than me. And that would be challenging the barriers of physical impossibility.
Just how good do you think you are, mathematician?

August 4, 2014 3:04 am

Fumble fingers me back to finish my comment.
Monckton shows his awareness of the pitfalls of treating theory as absolutes, as in his very apt Pythagorean allusion. Yet he is not yet at the point where he is ready to question the theoretical application of the absorbency spectrum of CO2 to the earth’s atmosphere. In other words, he hangs on the lip of his understanding.
I think eventually he will remember that Arrhenius was not Pythagoras and come to see the fruitlessness of pitting theory against observations in the case of CO2.

August 4, 2014 3:15 am

One thing we do know that CO2 causes: http://chiefio.wordpress.com/2014/08/04/what-a-coincidence/
USHCN adjustments.

August 4, 2014 3:21 am

mpainter says:
August 4, 2014 at 3:04 am
I have come to a similar conclusion. Sensitivity CO2 = 0.00 deg C/doubling is my best current estimate.

August 4, 2014 3:27 am

richardscourtney says:
August 4, 2014 at 1:16 am
Some think the WMDs in Iraq were shipped to Syria. So the information could be good on Monday and useless by Friday. The question then is: did those making the statements know they were no longer true?

August 4, 2014 3:33 am

richard verney says:
August 4, 2014 at 12:44 am

Monckton of Brenchley says:
August 3, 2014 at 3:49 pm
The sl**ers seem to be out in farce again. Mr Verney asks what proof exists that greenhouse gases cause warming. I refer him to any elementary textbook of heat transfer.
////////////////////////

… I would go on to suggest that the reason for that is that we simply do not know what other things are, nor to what extent each of these other things is in play at any given moment of time, and we are therefore unable to establish whether in the real world environs of the atmosphere of planet Earth, CO2 has any measurable input on temperature. Given that it appears that CO2, on all time scales, lags temperature, it does not look that promising that the release of CO2 drives warming as you assert. i am not going as far as saying that the lag proves that it does not, since the data sets are unfit for purpose and do not lead to informed knowledge or understanding.

You state “Then there are the major non-radiative transports: evaporation and convection up, advection across, and precipitation and subsidence down.” and this may lie at the crux of the issue since it may well be the case, in say 30 or 40 years time, when we have a better state of knowledge and understanding of things, that it is shown that the climate is governed almost exclusively by non-radiative transports and that radiative transports play an all but insignificant role.
I do not make any prediction of the future climate, but even die hard warmists such as Julia Slingo are suggesting that there may be no resumption to warming before 2030. If that is how matters pan out, by that time about 80% of all manmade CO2 emissions will have been released during which period no warming will have taken place, such that I predict that in 2030 not only will there be a significant reassessment of Climate Sensitivity suggesting a low sensitivity), but also a reassessment of the very foundations on which GHG theory is based. One reason behind a low sensitivity may be that there are fundamental flaws with the very theory itself.

I was taught approximately 40 years ago that “… there are the major non-radiative transports: evaporation and convection up, advection across, and precipitation and subsidence down.” In fact, I was taught in college that these transports plus the density of the atmosphere caused by gravity were the “main drivers” of our climate. I am saddened to discover that some here would call anyone who is skeptical of today’s dominate theory of climate a “Sl**yer” and asked that they be banned or closely watched. If (a big if I hope) we reach the point where Richard Verney’s very scientific skepticism is seen as disruptive to this site — then I see no reason for the site to pretend to be scientific.
I echo a point made today. The fact we know that CO2 has an absorption ban and can demonstrate that in a high school level experiment does not in any way tell us what CO2 does in our Earth’s atmosphere on NET. What is the net effect of going from 350 ppm to 400 ppm? I don’t know but all evidence suggests that whatever the effect is it is darned small. Vanishingly small.
Note: I held my beliefs decades before this very site made me aware that there was a group of people who are called “sl**yers”. I don’t think any post that mentions that name should be automatically tossed into the moderation bin. I hope Lord Monckton was exempt from that whey he wrote the word without the stars in it.

August 4, 2014 3:34 am

“Yes, liberating greenhouse gases by combustion will cause some warming – all other things being equal. Yes, we are liberating greenhouse gases by combustion. These things have been established beyond reasonable doubt”
So why the ZERO temperature rise in 17+ years despite significant CO2 rises??? This is the stark evidence of reality that rejects this “must warm” conjecture. It really is about time this luke-warmist *assumption* was dropped, as that’s all it is, an assumption. It is wholly unscientific.
CO2 absorbs IR in 2 main and 1 narrow band, and once it has absorbed that, it does what any other gas does, as it has to obey the laws of physics, it spontaneously re-emits that energy on collision with cooler molecules as it rises through the atmosphere under convection currents. It cannot ‘trap’/’hold’/’store’ that energy, i.e. it cannot ‘decide’ to retain it. CO2 also has zero reaction to all other IR bands, even if the CO2 concentration were 100%.

bobl
August 4, 2014 3:40 am

Lord Monckton,
A point or two…
Like the IPCC you fail to consider energy conversions, energies absorbed in conversion of heat and light to motion ( especially wind and rain ) , chemical bonds (as in photosynthesis), lightning, sound, water waves, entropy through melted ice, weathered stone, every grain of sand moved by a thermally driven wind, even vitamin D in our skin produced by the interactions of cholesterol and sunlight.
Sometimes these are neglected because the energy conversion losses are considered to be small, but I ask you to consider that a 2m swell contains 36 KW per square meter of wavefront, almost 30 times the total incident sunlight at midday and fully 60,000 times the supposed imbalance of energy of 0.6W per square meter driving global warming. Unless one knows the fraction of water waves/motion that is thermally driven, I don’t see how any conclusion can be drawn about energy balance and therefore sensitivity. It not just about temperature, it’s about energy. These are potentially huge unknowns that you don’t mention.
Secondly, I would respectfully ask you to consider the following.
It is known that the absorbtion bands at which CO2 operates are 85 odd percent saturated, IE that they are 85% opaque, and though disputed, scientists tell us that the earths surface is about 33 degrees above that for a theoretical blackbody at 1 AU.
From this we can make a relation, if we ascribe all the 33 degree warming to CO2 (and of course ignore the gas law and half of physics in doing so) We can say that ( assuming a linear relationship – I think worst case ) CO2 would cause 33/85 = 0.38 degrees of warming per %energy intercepted, and further that no more than 15 x (33 / 85) or 5.8 degrees could be expected even if we had a CO2 atmosphere. If we allow for real physics and therefore greenhouse gasses to only provide say 10 degrees of the 33 degrees then that projection is reduced to say 2 degrees of warming, for the last 15% of energy available to cause warming. Given the number of doublings required to reach that sort of saturation, one must conclude that sensitivity to C02 has to be much less than 1 degree per doubling.
While this has a lot of flaws ( for example energy to temperature is NOT a simple linear relationship , it would be expected that more warming would occur at low saturations and less at high) I think however that this simple analysis OVERSTATES sensitivity to energy at this point. It’s a bounds test I use. A sensitivity higher than about 0.5 degrees per doubling is inconsistent with the effect of greenhouse gasses overall in insulating our world. This simple boundary test also establishes a hard limit to CO2 warming or no more than 5.8 degrees – ever.
Could I ask you to dwell on this a bit.
Thank you
Bob.

Chris Wright
August 4, 2014 3:41 am

“Yes, liberating greenhouse gases by combustion will cause some warming – all other things being equal. Yes, we are liberating greenhouse gases by combustion. These things have been established beyond reasonable doubt. But what has not been established is the crucial quantitative question how much warming we may cause…..”
Christopher, you also stated that you believe that a CO2 doubling would give one degree of warming.
All the evidence e.g. the ice cores strongly suggest that CO2 has zero effect on the climate. Yes, I’m quite sure it has a warming effect in the laboratory of about one degree. I’m quite sure that the basic physics that predicts one degree is correct. But the climate system is completely different. So, what empirical evidence do you have that a doubling will give one degree of warming in the climate system? What historical data shows that a step change in CO2 was followed by a corresponding step change in temperature as predicted by AGW?
We know that a warmer world probably always has more atmospheric CO2, because the oceans emit more as they warm up. As far as I’m aware, no data with sufficient time resolution has ever shown that the CO2 change occurred before the temperature change.
With this in mind, my default belief (I hate the word ‘belief’, as it smacks of religion, but it’ll have to do) is that the sensitivity is effectively zero. But I’m completely open to any actual proof for a higher sensitivity.
I think the explanation for the modern warming is extremely simple:
The modern warming was caused by exactly the same thing that caused the warming during the MWP. And the Roman period. And the 1500 BC warming.
And like those warmings, the modern one had essentially nothing to do with CO2. By the way, of those warmings, quite likely the modern one is the smallest, despite the extra CO2.
If you think the sensitivity is one degree in the climate system (as opposed to the basic physics), please show me the empirical evidence to support that.
Many thanks for all your work!
Chris

MattN
August 4, 2014 3:52 am

Its been well over 30 years now since we first attempted to quantify climate sensitivity. Billions and Billions of dollars later, untold amounts of computing power used, literally tens of millions of man-hours consumed, we don’t have any more of a clue what the climate’s sensitivity to a doubling of CO2 is than we did when we started.
We got to the moon in less time once we got serious about it.

August 4, 2014 3:56 am

Sirs (Monckton & Watts), to outright reject evidence and argument against your “must warm” conjecture because it doesn’t fit with your belief is behaviour as unacceptable as Mann’s and Schmidt’s etc. The automatic labelling and dislike of anyone you even hint at considering ‘slayers’ really needs to stop. To shut down any thread of argument is standing in the position of ‘argument from authority’, something coming from the thermageddonites you have so long fought against. It also says you have the answer to the exclusion of all other, which I’m sure you would acknowledge simply isn’t the case.

richardscourtney
August 4, 2014 3:56 am

M Simon:
As I see it, your reply at August 4, 2014 at 3:27 am adds emphasis to my point at August 4, 2014 at 1:16 am.
Richard

August 4, 2014 3:59 am

Konrad says:
Viscount Monckton, If you wish to defeat me, you need to be a better empiricist than me. And that would be challenging the barriers of physical impossibility.
Now that’s confidence! I don’t know if it’s justified. But it made me LOL. ☺

August 4, 2014 4:03 am

H Grouse says:
August 3, 2014 at 2:57 pm
Wait a minute (or 17+ years) , how about these “instruments?”
—————
What instruments? Are you losing it ….. or already lost it?
I did not see any instruments on that graph that you provided a link to ….. nor did I see any mention of any instrument on said.
HA, in 17 years you should be able to learn how to use a dictionary …. iffen you get started today.

Bloke down the pub
August 4, 2014 4:09 am

‘We shall further assume that none of the warming was committed but unrealized warming from before 1984, or that, if there was some, it was approximately balanced by uncommitted warming generated over the 30-year period but not yet apparent in the temperature record.’
M’Lud, shouldn’t that be committed warming?

Monckton of Brenchley
August 4, 2014 4:30 am

In answer to ilma630, the freedom to comment here is a privilege, not a right. If anyone wants to comment here, he or she must do so in accordance with the rules, which include a prohibition on “slayers” (that is what they call themselves), who try to deny, in the teeth of the evidence, that there is any such thing as the greenhouse effect.
That vexatious, futile and time-wasting denial of what theory, observation and measurement amply confirm plays (and may well be intended to play) into the hands of those who brand climate skeptics as “deniers” of true science.
Maundering on about the supposed non-existence of the greenhouse effect is also off topic, and gums up threads such as this, to the exasperation of all who join the conversation here.
The slayers have their own website where they can invent and debate mad theories all they want: but, if they want to be taken seriously here, they must first convince a reputable scientific publication, by way of peer review, that what they maintain has some shred of scientific plausibility. As things stand, the notion that there is no such thing as a greenhouse effect was adumbrated by Joseph Fourier two centuries ago; was confirmed by the measurements of Tyndall and of Stefan, and of countless confirmatory experiments since, including the attenuation over time of long-wave emissions from the Earth’s surface in CO2’s principal absorption bands; and the theoretical demonstration by Boltzmann of the fundamental equation of radiative transfer, first derived experimentally by Stefan.
The basic theory has been repeatedly demonstrated not only on Earth but in numerous celestial bodies, and has been quite thoroughly explained in quantum theory. The only area of legitimate discussion is the likely magnitude of the enhanced greenhouse effect under today’s conditions, which was the subject of the head posting.
Yes, the Thermageddonites accuse climate skeptics of denying basic science: but they do this because, thanks to the poisonous and deliberately disruptive efforts of the “slayers”, we are bracketed and branded with their invincible ignorance. So, if you want to debate the imagined absence of the greenhouse effect, just try to get your notions into a scientific journal, and, when the journal rejects your notions, go and moan about it at the “slayers'” website.

hunter
August 4, 2014 4:33 am

I believe that history will show two central fallacies sustaining the failed paradigm of dangerous climate change. The first will be the gullibility of people to confuse basic science with ideas about how those ideas behave specifically. The second will be the endless appetite of far too many people to believe scary end of the world stories.

Monckton of Brenchley
August 4, 2014 4:38 am

In answer to Chris Wright, in the absence of temperature feedbacks there is unlikely to be much warming above 1 Celsius degree per doubling of CO2. But, though there are so many other processes influencing the climate that one cannot altogether rule out a zero sensitivity, the irreducibly simple climate model I have developed (it is now out for review) indicates that 1 Celsius degree per CO2 doubling is likely to be nearer the mark. There are so many unconstrainable uncertainties that it is possible we shall never know precisely what the warming effect of a change in CO2 concentration is: however, by the end of this century it will be evident to all that sensitivities as high as the CMIP5 models’ 3.2 C per doubling are extravagantly implausible.
From the policy perspective, a zero warming is the same as 1 C-per-doubling warming: both are non-problems, and neither requires any mitigative action. For this and other reasons, I am disinclined to argue about whether equilibrium sensitivity is 0, 1 or 2 degrees. All these are much of a muchness. Above 2 degrees, the climatic effects might become noticeable, but even then they would be unlikely to do much more harm than good.

Dan
August 4, 2014 4:40 am

“there has been no global warming for 17 years 10 months”
Your graph clearly shows 0.48 degrees warming over the 30 year period.
This therefore suggests that the global warming over the period 1984 to 1997, was 0.48 degrees, or 3.7 degrees per century.
Now that is some very significant warming.
Do you have any information on the breakdown of the causes for such a rapid temporary increase in global temperatures?

Hoser
August 4, 2014 5:00 am

Rereading the title, I realize you were talking about something we need to keep in mind.
http://wattsupwiththat.com/2013/12/27/many-climate-reconstructions-incorrectly-attributed-to-temperature-change/#comment-1515224
“And there are unknown knowns, the things we think we know, but actually don’t…. Aren’t we discussing a great collective lie, and a great crumbling example of an unknown known?”
It’s an important extension of Rumsfelds list (from the same post):

“… there are known knowns; there are things we know that we know. There are known unknowns; that is to say, there are things that we now know we don’t know. But there are also unknown unknowns – there are things we do not know we don’t know.”

Monckton of Brenchley
August 4, 2014 5:56 am

“Bobl” says I ignore several matters, including the ocean waves, which are caused not by global warming but by the wind and the tides. The principal climatically-interesting non-radiative transports are evaporation and convection upward, advection sideways, and precipitation and subsidence downward. Other influences no doubt exist, but they are ill constrained. There is far too much guesswork in the climate game, and there are far too few facts.

August 4, 2014 5:57 am

Monckton the main point of my 3:59pm comment was to try to get you and everyone else to widen your perspectives and include consideration of the quasi- millennial cycle in your calculations- see the resulting cooling forecasts at :
http://climatesense-norpag.blogspot.com/2014/07/climate-forecasting-methods-and-cooling.html

H Grouse
August 4, 2014 5:59 am

Samuel C Cogar says:
August 4, 2014 at 4:03 am
“What instruments?”
..
Spencer at UAH uses a different set of satellites than RSS

Monckton of Brenchley
August 4, 2014 6:04 am

“Dan” appears confused. A warming of 0.48 C over 30 years is equivalent to 1.61 C over 100 years, as shown on the graph, not the 3.7 C he suggests. One cannot simply subtract the 17 years and 10 months of no warming at the end of the 30-year period and thus obtain a trend of 3.7 C. The actual trend on the period from July 1984 to October 1996 is equivalent to 1.67 C/century equivalent, and the effect of the long period without warming is to bring that trend slowly down to its present 1.61 C/century equivalent over the past 30 years.

Monckton of Brenchley
August 4, 2014 6:18 am

Mr Painter, yet another “slayer”, says I am not ready to question the application of the greenhouse-gas theory to the atmosphere. But the head posting did exactly that. It did not, of course, question the theory, for that is well established. It questioned instead the magnitude of the warming that might be caused by the enhanced greenhouse effect. I have studied the theory, have familiarized myself with the fundamental equation of radiative transfer, established that its first derivative under present conditions, which determines the zero-feedbacks warming (all other things being equal) is broadly correct, and also verified the emission temperature not only for Earth but for several other planetary bodies.
I can find no significant error in the theory, and the ramblings of the “slayers” have been relentlessly unimpressive and unscientific. The only scientist I came across who was a “slayer” believed that he had demonstrated the absence of back-radiation using a piece of equipment that had not been designed for that purpose and was not suitable for it. When I pointed this out, and referred him to the manufacturers for further information (and they confirmed what I had told him), he ceased to be a “slayer” because he was left with no evidence whatsoever that they were right. That was an honest scientist.
Where I find a significant error is in the assumption that temperature feedbacks in the climate are strongly net-positive. There is evidence for that proposition, but it is not very good evidence. The evidence against a strongly net-positive feedback sum, both empirically and in theory, is strong (see, for instance, Lindzen & Choi, 2011; Spencer & Braswell, 2011; Choi et al., 2014). It is necessary only to remove the notion of a strong net-positive feedback sum to remove all menace from the enhanced greenhouse effect. Whether the warming at a doubling of CO2 is zero (if there is no greenhouse effect), or 1 C (if there is a greenhouse effect but no net-positive feedback sum), then there is no climate problem.
Unless I am shown reviewed evidence that there is no such thing as a greenhouse effect, I shall continue to rely upon its existence in my own calculations. And I shall continue to support Anthony in his desire to keep the wretched “slayers” off the pages here. They are a vexatious and unwelcome distraction.

Monckton of Brenchley
August 4, 2014 6:27 am

“Konrad” waves his arms, but fails to provide a quantitative value for his proposed amendment to the 255 K emission temperature for Earth that theory would lead us to expect. What proposed alternative value for mean terrestrial emissivity in the near infrared would he propose, and how has he determined it? And in which reviewed publication has his work appeared? We are not told. Do the measurement, do the math, draft the hypothesis in properly quantitative terms (math being the language of physics, like it or not), publish in a reviewed journal, and then come back and tell us all the new value for the Earth’s emission temperature.
Otherwise, one should be as skeptical of “Konrad’s” claim as of all other such claims. If a new hypothesis is incapable of withstanding scientific scrutiny, then why should I or anyone give it credence? Of course it is difficult to get any questioning of the official story-line into journals such as Nature, for it is not in Nature’s nature to do science any more, just as Science no longer does science. But there are other journals. If “Konrad” is as smart as he says he is, he will no doubt refer me to the publication in which he has properly set out his theory, and has identified a new and different value for the Earth’s emission temperature. If there is no such paper, then his remarks here amount to no more than bluff and bombast. There is plenty of that in the climate science debate, but I am not impressed by it, and nor should anyone else be.

JohnWho
August 4, 2014 6:40 am

E.M.Smith says:
August 3, 2014 at 1:22 pm
M’Lord,
I’d assert that the central climate fallacy is that an average of temperatures has any meaning at all, and the secondary fallacy is that temperatures say much of use about HEAT.

I suppose that while “the central climate fallacy is that the unknowns are known” may be correct, it is important to note that there are other climate fallacies that may also be central to the issue.

bobl
August 4, 2014 6:55 am

Lord Monckton
You say:
including the ocean waves, which are caused not by global warming but by the wind and the tides.
But what causes the wind that causes the waves?
Energy can’t simultaneously exist as both thermal energy and motion, either waves due to thermally induced wind or currents due to thermal differences in the ocean, these energies are removed from the thermal/radiant energy budget, so presumably increases in retained energy, increases all these little losses, since the losses of energy through transformations are unchacterised, ignored and (in the case of ocean motion – )potentially very large they are a completely unquantified and possibly non trivial loss with a magnitude quite probably much greater that Trenbeth’s energy imbalance – no imbalance, no global warming.
I quite agree that the main paths are as you say, but as an Engineer I always acknowledge the loss term. For example one can’t characterise a pendulum without friction even though the friction is very small, the clock eventually winds down – there are ALWAYS losses, and to completely characterise a system you must know the magnitude of the losses. Don’t make the same mistake as Trenbeth, acknowledge the “completely unknown” energy transformation losses.
Yes, much tidal energy is gravitational, but some unknown portion is due to temperature, I stress the word – unknown. Let’s say it’s only 1 in 1000, well that still represents 40W / square meter. Conversely of the gravitationally and momentum driven winds and waves, how much does friction add to the atmospheric temperature – another unknown? These are not small numbers.
Your post is about the unknowns, the uncertainties – the ignored system losses (and gains through friction) are I think, the most blatant unknowns of them all.

H Grouse
August 4, 2014 6:58 am

JohnWho says:
August 4, 2014 at 6:40 am
“I’d assert that the central climate fallacy is that an average of temperatures has any meaning at all”
If I have three one liter containers of water, with one at 10 degrees C, one at 20 degrees C and the third at 30 degrees C, what do you think the measured temperature will be if I take all three an put them into a 3 liter container and mix them?
..
I’m a betting man, and will bet on 20 degrees C.
What is your bet?

Dan
August 4, 2014 7:01 am

““Dan” appears confused.”
Not so, but thanks for taking the time to provide a response.
“One cannot simply subtract the 17 years and 10 months of no warming at the end of the 30-year period and thus obtain a trend of 3.7 C.”
But to not do so would highlight that there must have been warming during the 17 years and 10 month period, as indeed your graph shows.

Tim
August 4, 2014 7:18 am

Lord Monckton; your last paragraph says more to me than the rest of the article. My hope is that science blogs may soon turn their attention more to the political elephant in the room.The real cause behind all the arguments about the effects.

MikeB
August 4, 2014 7:22 am

Bobl (August 4, 2014 at 6:55 am)
There are no losses. Not in the long term.
Energy cannot be created or destroyed. This is the 1st Law of Thermodynamics. Energy may temporarily be in different forms, winds tides etc. but that is a distraction. It will likely end up as heat, just like the friction in your pendulum clock which eventually appears as heat; the energy is not ‘lost’.
It is not necessary to consider all these intermediate forms of energy (which would be prohibitive anyway) to understand the climate system in macroscopic terms. It is sufficient to know only the incoming and outgoing energy at the surface to explain the effects of greenhouse gases.
The first law of thermodynamics makes it easy for us : energy cannot be created or destroyed.

bobl
August 4, 2014 7:27 am

Lord Monckton
Let me also address the following comment
“Other influences no doubt exist, but they are ill constrained. There is far too much guesswork in the climate game, and there are far too few facts.”
Yes, I agree, but it’s perfectly acceptable to lump these terms into a single , non radiant loss term of unknown magnitude, the gains from friction due to gravitationally, and momentum driven motion can be similarly dealt with as a lumped (and completely unknown) non radiant heat gain term.
My point being that without these terms the model is incomplete, and we can’t possibly know what bias to warming or cooling is ultimately at play, it’s just as you say, but worse. Energy leaks into and out from the climate, and there is no reason at all to expect that radiant energy in = radiant energy out. Only that Total energy in = Total energy out, big terms are missing.

August 4, 2014 7:33 am

richard verney says:
August 3, 2014 at 11:13 pm
Increasing CO2 has one of three possibilities. It may warm the planet, it may have such an insignificant effect that it is so small that we cannot measure what it does. It may cool the planet. We have yet to work out which of these eventualities occurs in real world conditions.
—————-
Well now, sorry bout that, cause all 3 have been “worked out” and all 3 “do occur” in real world conditions.
============
I agree with you, that a proper evaluation of Venus also suggests that there is a problem with CO2 GH warming because the surface of Venus does not appear to be warmed by reflected Solar from the ……………
—————-
It is utterly silly for anyone, man or beast, to even think about comparing the planet Venus to the planet Earth relative to surface temperatures and/or atmospheric temperature & composition.
Mimicry is OK until one is told differently, but to do so after being told would be as idiotic as would be the comparing of “mountain oysters” to “jet fuel”.
And following are 5 facts about the planet Venus that substantiates my above sarcasm, … with #2 & #3 probably being the most important ones.
1. The atmosphere of Venus is 96% carbon dioxide (CO2).
2. The distance of Venus from the Sun is 68 million miles (1/3 rd closer than Earth)
3. It takes 242 Earth days for Venus to rotate on its axis
4. The clouds surrounding Venus are comprised mostly of sulfuric acid.
5. Venus has an atmospheric pressure (1,352 psi) that is 92 times that of planet Earth
Now, given Earth’s current atmospheric status, ….. just how frigging “HOT” do you suppose it would get in say Miami, FL, iffen the residents there had to withstand …. 100+ continuous 24 hour periods (2,400 hours) of bright Sunshine? Or, continuous bright Sunshine from mid-May to the end of August?
The temperature of Venus is determined by its close proximity to the Sun and the mass density (1,352 psi) of its atmosphere …. and not to the composition (CO2) of its atmosphere.

JohnWho
August 4, 2014 7:40 am

@ H Grouse says:
August 4, 2014 at 6:58 am
The assertion quoted was from
E.M.Smith at August 3, 2014 at 1:22 pm
However, I’ll assume you have made proper temperature measurements, so your result should be correct.
At this time, none of us can say that about the temperature measurements being made of the planet’s lower atmospheric temperature or upper atmospheric temperature. The satellite measurements do not encompass the entire upper atmosphere – I believe they don’t get the polar areas.
Using our best monitoring capability, even though we do not get a proper measurement of the entire area, arguably if properly “adjusted”, perhaps we could at least get trends. Of course, the Alarmist/Warmist would prefer to claim that this is one of our “knowns”.

August 4, 2014 7:49 am

H Grouse says:
August 4, 2014 at 6:58 am

how much water do you have?
1 + 1 + 1 = 3 lt correct
How many degrees do you have?
10 + 20 + 30 = 60 IN correct
How much heat energy (joules per second) do you inject into each container to reach each temperature? Well, it depends upon the starting temperature of each container.

August 4, 2014 8:22 am

Sorry post needs a few corrections, Here it is again,:
Much of this discussion re CS, radiative balance, ocean processes etc is irrelevant and indeed muddies the waters if your objective is to make forecasts of future climate. To do the latter all that is necessary is to simply look at the quasi periodic – quasi repetitive patterns in the temperature data itself and then project these forward for periods suitable to the wavelength of the pattern concerned. Thus we have a reasonable idea as to where we are relative to the Milankovitch cycles, and to the 970 and 60 year periodicities in the temperature data. The latter two can be reasonably assigned to solar activity variability but it is not necessary to understand the processes involved in order to make likely successful predictions.
It seems to me that if your interest is investigating the role of anthropogenic CO2, which is the stated mission of the IPCC and its modelers – you really need to understand that this is not possible unless you have a good idea of the timing and amplitude of the natural variations prior to dissecting out the human contribution. This, so far, the modelers have completely failed to do and so built their models on the highly implausible assumption that anthropogenic CO2 is the main climate driver. Obviously what they put into the models at one end came out at the other.
The way ahead is to continue gathering,for the last 3-5000 years in particular, as much quality temperature proxy data as possible and so refine on a regional basis our understanding of the exact timing and amplitude of the natural quasi-periodicities.
For a complete discussion of this see
http://climatesense-norpag.blogspot.com/2014/07/climate-forecasting-methods-and-cooling.html

August 4, 2014 8:43 am

Monckton of Brenchley:
Sir, a slayer I am not, not by your earlier definition as one who denies the greenhouse effect. I consider the greenhouse effect as something as obvious to anyone sensible. No need to apologize for mis-labeling me as one of those whom you regard as “a vexatious and unwelcome distraction”.
My comment above was fragmented at 2:40am & 3:04 am. Perhaps it was that which led you to misapprehend its meaning while you were in the midst of slaying the slayers. If you read it again, more carefully, I think that you will take away a different meaning and remove me from the bill of mortality, or so I should hope. I am not comfortable with such a pejorative (and untrue) tag on me.

more soylent green!
August 4, 2014 9:00 am

Yes, liberating greenhouse gases by combustion will cause some warming – all other things being equal.

I think one of the key take-aways here is the hightlighted phrase all other things being equal. The climate system isn’t a test tube and it reacts to the anthropogenic emissions. The warmists believe the natural reaction is to amplify the warming — an assumption not well supported by real-world observations. The actual response seems to be the opposite.

Bart
August 4, 2014 9:02 am

H Grouse says:
August 4, 2014 at 6:58 am
“I’m a betting man, and will bet on 20 degrees C.”
Now, assume you are mixing different fluids with different heat capacities. Do all regions of the Earth have the same effective heat capacity? Does the Amazon have the same heat capacity as the Sahara?

August 4, 2014 9:20 am

H Grouse says:
August 4, 2014 at 5:59 am
Spencer at UAH uses a different set of satellites than RSS
—————–
YUP, …… but just what EXACTLY are those satellites measuring? To wit:
—————————–
Satellite temperature measurements
The temperature of the atmosphere at various altitudes as well as sea and land surface temperatures can be inferred from satellite measurements. Weather satellites do not measure temperature directly but measure radiances in various wavelength bands.
Satellite datasets show that over the past four decades the troposphere has warmed and the stratosphere has cooled. Both of these trends are consistent with the influence of increasing atmospheric concentrations of greenhouse gases.

http://en.wikipedia.org/wiki/Satellite_temperature_measurements
Graph: Surface and Satellite Temperatures
http://en.wikipedia.org/wiki/Satellite_temperature_measurements#mediaviewer/File:Satellite_Temperatures.png
—————————-
TA DAH, ….. can be inferred, …..are consistent with, ….. WHOOOPEEE.
And Grouse, the above said greenhouse gases that have the same absorption/emission wavelengths ….. are what?
YUP, you guessed correctly, ……. H2O vapor and CO2. See reference graph below:
Graph: Radiation Transmitted by the Atmospherecomment image
And Grouse, the thermal “heat” energy in question can be transferred to the CO2 and H2O vapor via ….. conduction …… from any other gas molecule and/or particulate that is in the atmosphere.

Matthew R Marler
August 4, 2014 10:15 am

Teerry Oldburg: Monckton of Brenchley:
Are you asserting that when the change in the equilibrium surface temperature is divided by the change in the logarithm of the CO2 concentration the result is a constant? If so, please support this hypothesis..

What he has written is that, if it is a constant (as assumed by IPCC among others), then it is a small constant (smaller than alramists have been warning us of.)

Matthew R Marler
August 4, 2014 10:16 am

oops. that was for “Terry Oldberg.”

Matthew R Marler
August 4, 2014 10:26 am

Monckton of Brenchley: Since absence of correlation necessarily implies absence of causation, we know that the stochasticity of the temperature changes is not caused by the monotonically-changing CO2 concentration.
oops. You have forgotten that with (deterministic) high dimensional non-linear dissipative systems (such as the Earth climate system) the steady input can produce apparent step-changes in output at irregular intervals. A brief reference is the text book Modern Thermodynamics by Kondepudi and Prigogine, the later chapters.

August 4, 2014 10:35 am

richard verney says, August 3, 2014 at 11:13 pm:
“IF DWLWIR has sensible energy in the real world environs of planet Earth it would likely result in copious quantities of evaporation of the oceans (which would tend to boil off from the top down), which copious quantities of evaporation are not detected. Solar does not present this problem since the energy from Solar is not absorbed within just a few microns but for the main part over a depth of at least 1 metre thereby greatly diluting the energy so that the oceans can be kept warm without boil off.”
Yup. Because DWLWIR is not ‘heat’. The SW from the Sun is heat and so is a separate, thermodynamically working flow of energy. DWLWIR isn’t. The solar heat can directly heat things, that is, make them warmer. It can prompt evaporation. The DWLWIR can’t. Because it is only the one part of a continuous, integrated radiative exchange between surface and atmosphere, netting out to an UPWARD heat flux, the only real, detectable transfer of energy between the two systems.
Still the climate establishment really seems to crave for the DWLWIR to be heat (without coming out and stating it in plain words, of course), just like the solar flux. They after all ADD them together as separate energy inputs to the surface, as if they were equals (meaning, heat) in those Earth energy budget diagrams, and in all the talk about the DWLWIR warming the top sea skin layer a little bit to make the gradient through the surface smaller and hence facilitate an increase in OHC.
“If DWLWIR has sensible energy capable of performing sensible work in the real world environs of planet Earth, it is astounding that there are no large scale projects seeking to harness that energy. Which energineer would opt for Solar as a power source, when according to K&T, DWLWIR has about 100% more power and is a constant available 24/7 come rain or shine? The appeal of DWLWIR as a constant power source would favour this, and yet there appears to be no significant research into harnessing it, thereby suggesting that it is viewed as nothing more than a signal incapable of performing sensible work in the real world environs in which it finds itself here at the surface (or near surface) of planet Earth.”
Exactly right. A good way of showing why the solar flux and the calculated ‘back radiation’ can not be considered equivalent fluxes of energy. And why it’s clear they know it. Because they’re not equivalent fluxes of energy. People should stop pretending they are! You can harness the energy of the one. You can’t possibly harness (or even detect) the energy of the other.

H Grouse
August 4, 2014 10:38 am

Samuel C Cogar says:
August 4, 2014 at 9:20 am
….
:”satellites do not measure temperature directly”

Grab a mercury thermometer.
The device does not measure temperature.
There is some liquid mercury in the bulb, with a stem and markings. The Mercury inside the device expands and contracts, yet it does not measure temperature directly With the correct positioning of the markings on the stem via calibration, temperature can be inferred from thermometer readings.
Not really all that different than the instruments aboard a satellite????
..

Matthew R Marler
August 4, 2014 10:46 am

Monckton of Brenchley: From the policy perspective, a zero warming is the same as 1 C-per-doubling warming: both are non-problems, and neither requires any mitigative action. For this and other reasons, I am disinclined to argue about whether equilibrium sensitivity is 0, 1 or 2 degrees. All these are much of a muchness. Above 2 degrees, the climatic effects might become noticeable, but even then they would be unlikely to do much more harm than good.
Well said.
From WUWT’s new CO2 page, particularly the Mauna Loa graph, I estimated the rate of change of atmospheric concentration to be 0.4% per year. At that rate, the next doubling of CO2 concentration will take 174 years (obviously [?] I do not actually claim 3 sig figs of accuracy from a 1 sig fig at most input), which makes the 0, 1, or 2 equilibrium change even less of a problem, and not an urgent problem at all. Other rates of CO2 increase have been calculated: do you have a favorite.
I have read all of your responses to comments, as well as your head post. I thank you again for your writing, your persistence, and what looks to me like nearly unfailing good judgment. Please keep up the good work.

August 4, 2014 10:51 am

Matthew R Marler says, August 4, 2014 at 10:26 am:
“oops. You have forgotten that with (deterministic) high dimensional non-linear dissipative systems (such as the Earth climate system) the steady input can produce apparent step-changes in output at irregular intervals. A brief reference is the text book Modern Thermodynamics by Kondepudi and Prigogine, the later chapters.”
The point here is that those sudden upward shifts that I pointed to (the very same shifts that Bob Tisdale has now been pointing out and explained for the last five years, but no one seems to listen or care), are not ‘stochastic’ at all. They are very distinct, time-specific and process-related. The process in question? ENSO.
The ocean cycle caused the modern global warming between the 70s and the 00s. Nothing else. The ENTIRE rise in global temps since 1970 is contained within three abrupt and significant steps each established within the course of a year or less: in 1979, in 1988 and 1998. All the steps lift global temperatures 0.15-0.2 degrees up and away from the otherwise tight correlation to NINO3.4. But it ONLY happens on those three occasions. Not at any other time.
And the fully natural processes behind the steps are all easy to track in the data.
http://okulaer.wordpress.com/2014/06/17/how-the-world-really-warmed-between-the-70s-and-the-00s-part-i/
http://okulaer.wordpress.com/2014/06/23/how-the-world-really-warmed-part-ii-step-1/
http://okulaer.wordpress.com/2014/07/11/how-the-world-really-warmed-part-iii-steps-2-3/

Matthew R Marler
August 4, 2014 11:04 am

Richard Verney: IF DWLWIR has sensible energy in the real world environs of planet Earth it would likely result in copious quantities of evaporation of the oceans (which would tend to boil off from the top down), which copious quantities of evaporation are not detected. Solar does not present this problem since the energy from Solar is not absorbed within just a few microns but for the main part over a depth of at least 1 metre thereby greatly diluting the energy so that the oceans can be kept warm without boil off.
If DWLWIR has sensible energy capable of performing sensible work in the real world environs of planet Earth, it is astounding that there are no large scale projects seeking to harness that energy. Which energineer would opt for Solar as a power source, when according to K&T, DWLWIR has about 100% more power and is a constant available 24/7 come rain or shine? The appeal of DWLWIR as a constant power source would favour this, and yet there appears to be no significant research into harnessing it, thereby suggesting that it is viewed as nothing more than a signal incapable of performing sensible work in the real world environs in which it finds itself here at the surface (or near surface) of planet Earth .

I think it is a serious research question: where the DWLWIR impinges on the non-dry surfaces of the Earth, how much evaporation does it cause? There isn’t enough to “boil off” the oceans, but there is no reason to believe, or to assert, that there is no evaporation caused by the DWLWIR. Related to that is a question that I have often asked: If atmospheric CO2 concentration increases, how much will ocean evaporation increase? My conjecture is that more energy will go into evaporation than into heating, but it is nearly pure conjecture.
This again: thereby suggesting that it is viewed as nothing more than a signal incapable of performing sensible work in the real world environs in which it finds itself here at the surface (or near surface) of planet Earth .
That doesn’t make sense: if DWLWIR isn’t energy carrying, then it can’t result from electrons changing from higher energy orbits declining into lower energy orbits. You are better off saying that an important energy transfer process has been too little studied.

milodonharlani
August 4, 2014 11:05 am

Matthew R Marler says:
August 4, 2014 at 10:46 am
Average over the past 30 years has been about 1.78 ppm/year growth in Mauna Loa record (range of 0.48 to 2.66 in 1984-2013). With increasing reliance on natural gas, this level may drop, & even more so if modern nuclear power technologies are adopted & conservation embraced. But even at 2.0 ppm/yr, it will take 80 years to reach 560 ppm, a doubling from assumed 1850 level.

Matthew R Marler
August 4, 2014 11:11 am

Kristian: The point here is that those sudden upward shifts that I pointed to (the very same shifts that Bob Tisdale has now been pointing out and explained for the last five years, but no one seems to listen or care), are not ‘stochastic’ at all. They are very distinct, time-specific and process-related. The process in question? ENSO.
That may be true, but it is not what I consider “known”. The possibility that stepwise increments of mean global temperature associate with ENSO are caused by gradually accumulated CO2 can not be ruled out on present evidence because there is no complete and thorough account of all the energy transfers related to ENSO. Neither is CO2 accumulation the only possible explanation, pace IPCC. It simply isn’t resolvable on present evidence how much of the increase is due to increased CO2. But I think that Lord Monckton is correct that if the standard theory is basically true to a first order at least, then the effect of CO2 can’t be as great as alarmists have been claiming.

Steve Oregon
August 4, 2014 11:57 am

Monckton corrected me with
“Steve Oregon” is incorrect to say that man-made CO2 is little more than 3% of atmospheric CO2. It is more like 40%.”
I’m not sure what to make of that. Seems inconsistent with much.
http://notrickszone.com/2013/03/02/most-of-the-rise-in-co2-likely-comes-from-natural-sources/
“The natural CO2 flux to and from oceans and land plants amounts to approximately 210 gigatons of carbon annually. Man currently causes about 8 gigatons of carbon to be injected into the atmosphere, about 4% of the natural annual flux.”
Jeff Id “The atmosphere contains 720 billion tons of CO2 and humans contribute only 6 GT additional load on this balance.”
I had misstated what I had intended to say.
The figures were % of total man-made contributions to the greenhouse effect as it relates to CO2.
From here http://www.geocraft.com/WVFossils/greenhouse_data.html
My point on August 3, 2014 at 5:25 pm was to emphasize how miniscule the total human CO2 contribution to the greenhouse effect is in order to make by greater point.
That being with only 0.117% of the greenhouse effect due to atmospheric CO2 from human activity it is preposterous for the Climate team to claim they can measure any human warming impact at all let alone what fraction of supposed warming is attributable to us and our CO2 emissions.
Worse yet this miniscule contribution to the greenhouse effect by man cannot possibly
be the catalyst that triggers the atmospheric water vapor increases which AGW/global warming requires. There is no means to even measure such an effect or impact.
http://earthobservatory.nasa.gov/Features/WaterVapor/water_vapor2.php
“Warming due to carbon dioxide emissions from fossil fuel combustion evaporates even more water, increasing the thickness of the blanket, which leads to more heating, which leads to more water vapor… The loop is called the water vapor feedback, and it has the potential to be a serious problem.”
Anthropogenic (man-made) Contribution to the “Greenhouse
Effect,” expressed as % of Total (water vapor INCLUDED)
Based on concentrations (ppb)
adjusted for heat
retention characteristics % of Greenhouse Effect % Natural % Man-made
Carbon Dioxide (CO2) 3.618% 3.502% 0.117%
Water vapor 95.000% 94.999% 0.001%
Bottom line is there is no measured human CO2 catalyst that has ever been measured.
It is nothing but a conjecturous con job. (Is that a word)

Monckton of Brenchley
August 4, 2014 12:38 pm

Mr Marler, who has written very kindly about this posting, asks at what rate I think CO2 will rise. Though I agree with him that 1.004^174 = 2, I suspect that CO2 concentration will continue to rise quite rapidly as the third world – notably China, India and eventually even poor old Africa – industrializes its way out of poverty and thus towards population stability (we may be at peak children worldwide already).
So I’d expect to see the gentle but persistent acceleration in the curve of CO2 concentration to continue until mid-century, so that by 2100 there will be 550-700 ppmv in the atmosphere. In other words, not even double today’s concentration. My best estimate, based on an irreducibly simple climate model, is that the warming by then will be about 1 Celsius degree compared with today. In 2008, in Physics and Society, I predicted that doubling CO2 would increase global temperature by less than 1 Celsius degree. My current researches indicate that that is still correct, but the uncertainties are little better constrained than they were in 2008, so I wouldn’t call it settled science.

milodonharlani
August 4, 2014 12:43 pm

Steve Oregon says:
August 4, 2014 at 11:57 am
Conjectural con job.

Monckton of Brenchley
August 4, 2014 12:50 pm

“Dan” continues to be confused. All datasets other than RSS (which shows no warming for the past 17 years 10 months) show some warming since 17 years 10 months ago. But the trends as displayed on each of the graphs are correct, and there is no justification for saying that there has been any supradecadal warming at a rate equivalent to 3.7 Celsius degrees per century. The only period of 10 years or more in the entire instrumental record that exhibited a warming rate as great as that was the four decades 1694-1733 in the Central England record, showing a warming rate equivalent to 4.33 Celsius degrees per century – and that was before the Industrial Revolution began, so it cannot have been anything to do with us.
The main point remains the main point: the world has barely warmed in the past few decades, and the discrepancy between the absurd predictions of the IPCC and the models on the one hand and real-world observations and measurements on the other grows wider by the month. It is this widening discrepancy, rather than arguments over a few hundredths of a degree more or less, that demonstrates the falsity of the argument that one should rely on the models as the basis for shutting down the West to “save the planet” from a threat that is non-existent.
If dangerous global warming were a scientific theory, it would have been rejected by now as the nonsense it is. But it is not science. It is politics – and remarkably unpleasant, totalitarian politics at that.

Bart
August 4, 2014 1:00 pm

milodonharlani says:
August 4, 2014 at 12:43 pm
Steve Oregon says:
August 4, 2014 at 11:57 am
Steve Oregon should be made aware that the debate about attribution is widely considered a fringe topic, with most people even on the skeptic side accepting that human inputs have accumulated to the point where they represent some 30% of the observed atmospheric concentration. In many quarters, it is a quick ticket to oblivion to gainsay what is considered by many to be a self-evident truth.
Those people accepting human attribution are, however, wrong. It is very clear, if one actually takes the time to study the data, and is qualified to do so, that human inputs have little effect on the atmospheric concentration. But, I do not expect it will be broadly accepted before the primary hypothesis, that atmospheric CO2 is driving surface temperatures, is well and truly buried for good.
At that point, the lonely band of clear-eyed thinkers, who can read the tea leaves and see the reality for what it is, may gain a wedge to force a rigorous accounting by pointing out that, if the soi-disant experts got the primary hypothesis dead wrong, what else did they get wrong?

August 4, 2014 1:21 pm

Matthew R Marler says, August 4, 2014 at 11:11 am:
“That may be true, but it is not what I consider “known”. The possibility that stepwise increments of mean global temperature associate with ENSO are caused by gradually accumulated CO2 can not be ruled out on present evidence because there is no complete and thorough account of all the energy transfers related to ENSO.”
I’m sorry, but then you are not looking at the data. And I’m not just talking about the global data. You need to look at the different regions of the world. And the timing of these steps. Where and when did they take place? You can track it all. How it developed. No problem. But you need to look. Actually look.
This is not considered “known” only because no one wants to acknowledge it. People are too busy ignoring what the data is clearly saying.
We know. Because the data shows.
There is no need to ‘account for all of the energy transfers related to ENSO’. You just look at the data and see that global temps mimic the NINO3.4 in impressive fashion, with an average lag of a couple of months, more during large events, less in between. And that global temperatures show no upward (or downward, for that matter) deviation from any decadal trends in NINO3.4 since the mid 60s. EXCEPT during those three distinct and sudden steps, in 1979, in 1988 and in 1998.
ALL we need to explain are the three global steps. All the other 45-50 years are satisfactorily explained by that tight and consistent (and evidently causal) NINO3.4 (tropical East Pacific) lead/lag correlation.
Well, the three steps are also easily explained. But naturally you need to look beyond the NINO3.4 region. Just take a look at the available data. It’s still all initiated in the Pacific, large-scale coupled oceanic/atmospheric processes, just like the classical ENSO events themselves, only outside the NINO3.4 region. The ENSO process operates across the Pacific basin. And beyond, through so-called atmospheric bridges. It is in no way restricted to the NINO3.4 region, as most people (still) seem to think. ENSO isn’t the same as the NINO3.4 index.

Matthew R Marler
August 4, 2014 2:14 pm

Kristian: I’m sorry, but then you are not looking at the data. And I’m not just talking about the global data. You need to look at the different regions of the world. And the timing of these steps. Where and when did they take place? You can track it all. How it developed. No problem. But you need to look. Actually look.
This is not considered “known” only because no one wants to acknowledge it. People are too busy ignoring what the data is clearly saying.
We know. Because the data shows.

You missed my point, so I shall repeat it: the dynamical systems theory shows that you can have step changes and regional changes and regional step changes with continuous input; consequently, the step changes in the global and regional indices do not imply that the changes can not be related to continuous CO2 increases. Examples are provided in the thermodynamics text by Kondepudi and Prigogine, experimental and computational examples.

Matthew R Marler
August 4, 2014 2:20 pm

Monckton of Brenchley: So I’d expect to see the gentle but persistent acceleration in the curve of CO2 concentration to continue until mid-century, so that by 2100 there will be 550-700 ppmv in the atmosphere.
Thank you for your reply. Padilla et al some years back modeled a doubling by 70 years, but as you noted, a model requires some specific value for its conclusions to be derived. I think that’s kind of a low bound on the time to double. Your expectation is certainly defensible. I’m 67, so I don’t think I’ll ever know.

Steve Oregon
August 4, 2014 2:22 pm

Thank you Bart.
I guess I am dangerously close to becoming fringe. 🙂 But my arrival will not be without plenty of regular input from a number of science pals, known by many, who share my frustration.
I have found my recent curiosity and search for what is truly the known human contribution to the atmospheric greenhouse effect to be rather disturbing. Especially when trying to find what has actually been scientifically measured vs what is no more than embellished IPCC presumption.
Of course all of much smarter people here have nailed the IPCC’s theory with exceptional display of the “discrepancy between the absurd predictions of the IPCC and the models on the one hand and real-world observations and measurements on the other grows wider by the month” as Monckton says just above.
But I am nearly convinced that the most bare bones display of how little has been actually scientifically measured and what science is capable of measuring may be the genesis to finally suffocating the AGW farce.
If the human contribution to the overall atmospheric greenhouse effect is truly infinitesimal….
“Total human greenhouse gas contributions add up to about 0.28% of the greenhouse effect”.
http://www.geocraft.com/WVFossils/greenhouse_data.html
…..then it cannot have done or do what has been claimed it has or will.
And climate science is not capable of measuring the effects of this infinitesimal amount.
And no one anywhere has any scientific measurements of any human CO2 emission influences or impacts at all.
And the hypothesized notion that mankind’s minuscule contribution to the atmosphere has triggered an increase in atmospheric water vapor also has no scientific measurements at all, anywhere, to back up that thoroughly implausible assertion.
Measuring atmospheric human/CO2 emissions does nothing to measure the proportionate share it it represents or what that tiny share has, is or is capable doing.
Of course there have been many layers of additional assertions over the years but IMO AGW collapse will come from peeling the onion back to the core fallacious contention.
The impact of human CO2 is not measurable.
http://en.wikipedia.org/wiki/Infinitesimal
Infinitesimals have been used to express the idea of objects so small that there is no way to see them or to measure them.
In common speech, an infinitesimal object is an object which is smaller than any feasible measurement, but not zero in size; or, so small that it cannot be distinguished from zero by any available means. Hence, when used as an adjective, “infinitesimal” means “extremely small”. In order to give it a meaning it usually has to be compared to another infinitesimal object in the same context (as in a derivative).

August 4, 2014 3:08 pm

Matthew R Marler says, August 4, 2014 at 2:14 pm:
“You missed my point, so I shall repeat it: the dynamical systems theory shows that you can have step changes and regional changes and regional step changes with continuous input; consequently, the step changes in the global and regional indices do not imply that the changes can not be related to continuous CO2 increases. Examples are provided in the thermodynamics text by Kondepudi and Prigogine, experimental and computational examples.”
Yes, and you apparently continue to miss mine: The data from the real world precludes the gradual steady rise in CO2 since the 50s as the cause of those three distinct steps identified. Because it shows very clearly that they were all caused by internal processes initiated in the Pacific ocean, all directly associated with abrupt basin-wide climate regime shifts.
So, although I agree with you in theory, this is a perfect example of blinkered apologetic ad hoc hypothesis meets crystal-clear empirical evidence.

Bart
August 4, 2014 3:10 pm

Steve Oregon says:
August 4, 2014 at 2:22 pm
You’ve come to the right place, Steve. I am the resident, what I’m sure many here believe, crackpot explaining to everyone who will listen why the evidence indicates that humans are not driving CO2. You might be interested in the exchanges on this recent page. Every so often, this topic gets revived and I do battle with the resident champion of the human attribution camp, Ferdinand Englebeen. This post in particular gives a brief summation of my argument.

Dan
August 4, 2014 3:18 pm

““Dan” continues to be confused. All datasets other than RSS (which shows no warming for the past 17 years 10 months) show some warming since 17 years 10 months ago.”
Again not so. You seem to be struggling to understand my point.
You have chosen to put up a graph of the RSS data over a 30 year period. Which also has a linear fit over that whole period, equating to a 0.48 degree temperature rise.
It is you that stated “RSS (which shows no warming for the past 17 years 10 months)”.
So if you take your comment to be correct that RSS shows no warming over the past 17 years and 10 months, then all of that 0.48 degree warming would have occurred in the preceding time period of your chosen 30 year sample.
Indeed your article of 2 August 2014 is headlined “Global Temperature Update – No Global Warming for 17 years 10 months”, again referencing RSS data reiterates, your position on this matter, which you now seem to be backing away from.
There appears to be a lack of consistency between you various articles.

Monckton of Brenchley
August 4, 2014 3:38 pm

“Dan” is hopelessly confused. Over 30 years, RSS shows a trend. Over 17 years 10 months it shows no trend. But one cannot merely subtract the one trend from the other, as “Dan” has attempted to do. Any elementary statistics textbook will be of assistance.

milodonharlani
August 4, 2014 3:48 pm

Dan says:
August 4, 2014 at 3:18 pm
Not sure why this is a problem for you.
The global T trend for the past c. 3300 years (at least) is down. For the past c. 300 years it is up. For the past 30 years, it is slightly up. For the past going on 18 years, it is flat to down.

Steve Oregon
August 4, 2014 4:37 pm

My apologies for my persistence, but I’m simply seeking simplification.
My own shortest version of the central fallacy.
Scientists have never been able to measured the human role in the greenhouse effect.
Because the human contribution is infinitesimal the effect cannot be measured.
During all of my searches I’ve noticed that all of the government and alarmist sites avoid any mention of proportions. They speak about tonnes of CO2 etc. but not the EPA, NASA or any other I could find mention the percentages.
This allows the alarmists to cite the alarming tonnage of human CO2 being pumped into the
atmosphere and presumably absorbed by the oceans.
Whereas if they cited “human greenhouse gas contributions add up to about 0.28% of the greenhouse effect” who would care?
http://www.geocraft.com/WVFossils/greenhouse_data.html
Climate Science has been surmising what they cannot measure.
The entire heap they piled up to hide this central fallacy is meaningles
Again, because the human contribution is so small it is impossible to measure, or know, the impact of such a tiny influence.
If I pee in an Olympic size swimming pool I could only surmise that I am warming the water.
I can measure the pool temperature trend all I want and pile up a heap of surmising conclusions about buoyancy, swimmers, water level, pool life, swimwear, drain lines, lifeguards etc and still I will never know what my pee did.
AGW is Climate Surmising. Not science.
1sur·mise noun \sər-ˈmīz, ˈsər-ˌ\
Definition of SURMISE
: a thought or idea based on scanty evidence : conjecture

Steve Oregon
August 4, 2014 4:50 pm

Thursday, July 17, 2014
New paper finds only ~3.75% of atmospheric CO2 is man-made from burning of fossil fuels
http://hockeyschtick.blogspot.com/2014/07/new-paper-finds-only-375-of-atmospheric.html
That translates into an infinitesimal portion of the greenhouse effect.
“Total human greenhouse gas contributions add up to about 0.28% of the greenhouse effect”
That is so small it may as well be nothing.
http://www.geocraft.com/WVFossils/greenhouse_data.html
Forget GIGO
AGW is NINO
Nothing In Nothing Out

Werner Brozek
August 4, 2014 6:23 pm

Dan says:
August 4, 2014 at 3:18 pm
I hope this helps!
See all 4 lines in the graphic below. In particular, note that the green line goes up only 0.23 degrees in 12.2 years and not 0.48 degrees.
http://www.woodfortrees.org/plot/rss/from:1984.55/plot/rss/from:1984.55/to:1996.75/trend/plot/rss/from:1984.55/trend/plot/rss/from:1996.75/trend

ferdberple
August 4, 2014 6:32 pm

How can such ignorance be explained?
===============
“When I was a boy of 14, my father was so ignorant I could hardly stand to have the old man around. But when I got to be 21, I was astonished at how much the old man had learned in seven years.”
Mark Twain

Werner Brozek
August 4, 2014 6:39 pm

Steve Oregon says:
August 4, 2014 at 4:37 pm
Whereas if they cited “human greenhouse gas contributions add up to about 0.28% of the greenhouse effect” who would care?
And earlier:
Monckton corrected me with
“Steve Oregon” is incorrect to say that man-made CO2 is little more than 3% of atmospheric CO2. It is more like 40%.”
Carbon Dioxide (CO2) 3.618%

Many people assume that on an annual basis, humans contribute 3% of the CO2 and nature contributes 97%. Of course our annual contribution was much less than 3% in 1800. However the change from 1750 of a presumed 280 ppm to 400 ppm today is a 40% increase that many attribute to man. Now if we assume that CO2 contributes 3.6% of the greenhouse affect, and if we assume man added 40%, then man’s part would be 1.44% and not 0.28%.

ferdberple
August 4, 2014 6:50 pm

Given that it appears that CO2, on all time scales, lags temperature, it does not look that promising that the release of CO2 drives warming as you assert.
==============
this really is the only way I can see to resolved the debate. No amount of arguing over theory, no amount of model building, can ever resolve what will actually happen.
there is one and only one test that will tell us the effects of CO2. And that is to observe what happens when CO2 is released PRIOR to warming. Do we see any CONSISTENT evidence of warming following natural events that release CO2? Surely in the millions of years of paleo data, there must be multiple events that released large amounts of CO2 for reasons not related to temperature.
What do they tell us? Is there any evidence in the paleo records showing non temperature related CO2 increase driving temperature? Because quite frankly, we can all argue CO2 GHG theory until we are blue in the face and it will prove absolutely nothing.

TimNitOz
August 4, 2014 7:02 pm

I’m fairly sure I know nothing.
After following discussions over some topical issues, I noticed both sides referring to facts and truths. I’m sure you have all heard the expressions, “the fact of the mater is ….blah …. blah”, or, “the truth is … blah … blah”, the discussions often get heated and rarely do they find common ground.
I decided to research what a fact is, and basically it is something you believe to be true. So I researched truth, and found that philosophers have been discussing this for thousands of years, and there is no clear definitive answer. Most of it goes over my head, and they discuss such concepts of consensus, esoteric, observational, etc, etc.
This got me thinking. Why do so many people get so emotional about their truths, which seems to be their belief in what is true? My suggestion is that it really comes down to what each individual believes, and what that individual accepts to be truth to them. This is at the core of what a person is, and when challenged, it becomes personal, and then emotional, because the people involved see it as an attack of the very essence of who and what they are.
This led me to examine my own personal truths, and I had to ask my self, what am I basing my personal beliefs on? I came to realize that most of my beliefs were based on what I learnt, either through my family, my education, my social interaction, or personal research. I started asking myself many “What if …” questions. What if what I learnt wasn’t completely true, how could I sort out fact from fiction?
It became obvious to me I had to find a reliable truth filter, to properly assess my own truths, so I needed to distinguish between what I really know, from a belief. This is a continual work in progress, but at this stage (it took a while) I’ve discovered that my entire set of truths are beliefs, and I actually do “know”, very little, or maybe, nothing.
This realization bruised my ego somewhat, but I have developed an interesting theory that I would like to share with you. “The ultimate truth is to accept that we don’t know everything”. This is not to say that it is impossible to know everything, just at this stage, we don’t. I reckon this may be true for each individual and collectively for the human race today.
What a beautiful world it would be if we could discuss any subject with any person, and recognize that both parties are only exploring what their own belief is, and are testing this belief, to see if it is true or not, or maybe needs modification. It would take a huge amount of humility and love, but to me, it is worth striving for.

Dr. Strangelove
August 5, 2014 12:19 am

Matthew
“My conjecture is that more energy will go into evaporation than into heating, but it is nearly pure conjecture.”
We can calculate this. From the water cycle we know 434,000 cubic km. of water is evaporated from the oceans every year. This translates to 3.8 x 10^-5 kg per second per sq. m. rate of evaporation. Latent heat of evaporation of water = 2257 kJ/kg. We get 86 W/m^2 evaporation rate. The ave. longwave “backradiation” is over 300 W/m^2. Therefore less than 30% goes to evaporation.

Dr. Strangelove
August 5, 2014 12:35 am

BTW evaporation has a cooling effect but when the water vapor condenses, it’s an exothermic process. A heating effect. Notice the air feels warm before it rains. That’s high humidity and latent heat released in the clouds.

Konrad
August 5, 2014 3:34 am

dbstealey says:
August 4, 2014 at 3:59 am
———————————
The judges would also have accepted “overconfidence” 😉
However in terms of provocation the comment served its purpose…
Normally when I’m using the “smartest guy in the room” line I include the caveat –
“Serving suggestion only. Results may vary depending on telephone booth/broom closet size”
And the reason for such caveats? While I frequently play the fool, I am not (despite Christopher’s claims) an idiot. I work with people far smarter than me, people far, far smarter than climastrologists. Where did I learn to empirically double check such inane assumptions like “the oceans are near blackbody”? Unlike me Christopher assumed that the initial assumptions underlying his maths were correct. He did not personally check. And assume makes…?

Konrad
August 5, 2014 4:52 am

Monckton of Brenchley says:
August 4, 2014 at 6:27 am
——————————————-
Viscount Monckton,
I find your lack of faith in the scientific method….disturbing.
You shriek to present and future readers that my empirical experiments have not been pal reviewed, and that this justifies your epic failure to replicate or verify.
There are a few little problems with that. My selective surface experiments are just a replication of work from Texas A&M peer reviewed in 1965, and my 2011 experiments into incident LWIR on water that is fee to evaporatively cool were published in an engineering journal by proxy by an author who simply cut and pasted my work and experiment photos from Talkshop.
“Konrad waves his arms, but fails to provide a quantitative value for his proposed amendment to the 255 K emission temperature for Earth that theory would lead us to expect. What proposed alternative value for mean terrestrial emissivity in the near infrared would he propose, and how has he determined it?”
The relevant value is “well above 15C for 71% of the planets surface”, and you too could understand this if you replicated my experiments ( come on, I provide full build instruction…)
“And in which reviewed publication has his work appeared? We are not told.”
You what? I need pal review for replicable empirical experiment to be science? Do I need a pal reviewed consensus to be right? Is that the level you have descended to?
“Otherwise, one should be as skeptical of “Konrad’s” claim as of all other such claims. If a new hypothesis is incapable of withstanding scientific scrutiny, then why should I or anyone give it credence?”
To what actual scientific scrutiny did you subject my empirical claims? None. You made a call to authority argument. You cannot fight an empiricist and win, after all you are but a mathematician. Is it out of your depth on a wet pavement, or so far out of your depth the fish have lights on their noses?
“There is plenty of that in the climate science debate, but I am not impressed by it, and nor should anyone else be.”
You’re not impressed? Therefore no one else should be? You are the guy on permanent record who accepted the climastrologists 255K assumption. Call to authority arguments don’t work in these circumstances….
Ultimately if all you’ve got is “it’s not pal reviewed”, then you have nothing.
I checked Christopher, You assumed. What does assume make…?

August 5, 2014 4:57 am

Konrad:
How right you are – it is all in the assumptions and the data does not support the theory which all assume to be correct. Well, time to examine the assumptions. The poster M*nckt*n pretended to do just that but when commenters raised doubts about “climate sensitivity” this touched a nerve and he responded by labeling them as
“SL*y*rs. You were right to remind that fellow that empiricism trumps theoretical musings.

August 5, 2014 4:59 am

Matthew R Marler says, August 4, 2014 at 11:04 am:
“I think it is a serious research question: where the DWLWIR impinges on the non-dry surfaces of the Earth, how much evaporation does it cause?”
No need to waste money on such a project. The answer to your question is very simple: 0.
http://wattsupwiththat.com/2014/08/03/the-central-climate-fallacy-is-that-the-unknowns-are-known/#comment-1701396
“There isn’t enough to “boil off” the oceans, but there is no reason to believe, or to assert, that there is no evaporation caused by the DWLWIR.”
Yes there is. Because it couldn’t cause any evaporation. It is not a separate, thermodynamically working flow of energy.
“(…) if DWLWIR isn’t energy carrying, then it can’t result from electrons changing from higher energy orbits declining into lower energy orbits. You are better off saying that an important energy transfer process has been too little studied.”
DWLWIR is not an ‘energy transfer process’. It is one conceptual part of an integrated thermal radiative energy exchange through a continuous radiation field between two systems at different temperatures. The radiative heat going from surface UP, moving through that radiation field, is a real (as in detectable) ‘energy transfer process’. It is the net of the mathematically derived (potential) DWLWIR and UWLWIR ‘fluxes’. You cannot separate the one from the other within the exchange. The radiative heat is the only actual transfer of radiative energy between surface and atmosphere.
Again, this has been pointed out so many times to those of you who actually believe DWLWIR (‘atmospheric back radiation’) to be a real, separate working flow of energy to the surface (an extra (and equal) input of energy next to the solar heat flux): If this were really the case, then why aren’t we harnessing this energy flux? It is seemingly twice as intense as the solar flux, evened out globally and across the diurnal cycle. Why aren’t we seeing ‘back radiation’ power plants all over the world?

August 5, 2014 5:02 am

And for the benefit of others I will complete your last: “…an ass out of u and me”

Chris Wright
August 5, 2014 5:43 am

Christopher,
I agree, in a practical sense it probably doesn’t matter whether the sensitivity is zero, or one or two degrees. It wouldn’t be a problem, some warming is a benefit. History repeatedly shows that mankind prospered during the warm periods, and suffered during the cold periods.
But in another sense it is important, because it’s one of the great scientific questions, a bit like the age of the universe. Not necessarily of practical use, but nevertheless very important.
I’m certainly not an expert on climate science, but here are several reasons why I think the sensitivity is probably unmeasurably small:
1. Nearly half of the modern warming occurred before there was sufficient CO2, so clearly something else is at work.
2. As far as I’m aware, there is no empirical scientific data that shows a change in CO2 followed by a corresponding change in temperature. Surely, if the sensitivity is one degree, it should show up repeatedly in the ice core data? Of course, the CO2 follows the temperature, and not the other way around.
3. There have been four major warming periods over the last few thousand years: 1500BC, the Roman and Medieval periods and the modern warming. Based on ice core data, each warming period was a bit smaller than the previous one. Most likely they all had the same cause, whether it was solar or something else. But in the most recent warming, there should have been additional and fairly significant warming from the extra CO2. But there is no sign of it: the modern warming is no warmer than the previous three, and may well be slightly less warm. Surely, this strongly suggests that CO2 had essentially no warming effect at all.
4. AGW theory makes a number of specific forecasts: the equatorial hot spot, forecast warming, and the trend of Earth’s infra red emissions. The emissions are predicted by AGW to be getting smaller, but Lindzen showed that, not only is the size of the trend wrong, it has the wrong sign!
All these predictions based on AGW have turned out wrong. If science were working properly, AGW would have been scrapped years ago.
5. Much of climate science is corrupt almost beyond belief. If AGW were true, why is the most prominent paper supporting it (MBH98, the dreaded hockey stick) provably fraudulent?
And yet you do actually believe in a theory that looks to be completely discredited!
Finally, if the sensitivity is one degree, where is the empirical evidence to support that? Where are the examples of a step change in CO2 being followed by a corresponding step change in temperature?
Best regards,
Chris

Trick
August 5, 2014 6:12 am

Konrad 4:52am: “My selective surface experiments are just a replication of work from Texas A&M peer reviewed in 1965, and my 2011 experiments into incident LWIR on water that is fee to evaporatively cool were published in an engineering journal by proxy by an author who simply cut and pasted my work and experiment photos from Talkshop.”
Yet more handwaving “bluff and bombast” (top post Christopher term 6:27am comment). Citation and/or link please Konrad.
“The relevant value is “well above 15C for 71% of the planets surface”…”
Provably false by the bulk of papers in the field from reasoned analysis of both CERES, ERBE et. al. experiments, Earth weather station experiments, in situ ocean measurements. For the reasoned analysis featuring data from all the planets with thick atm.s proving Konrad claims are false bluff and bombast cite:
http://faculty.washington.edu/dcatling/Robinson2014_0.1bar_Tropopause.pdf
“To what actual scientific scrutiny did you subject my empirical claims?”
The 1st law of thermodynamics and Planck distribution et. al. basic intro. text physics including Fourier conduction, convective science of a bottom warmed fluid in a gravity field. ALL backed by extensive lab and field experiment accounted for with precise, calibrated equipment and informed, critical reasoned scrutiny.
“You’re not impressed?”
Not by Konrad. False prophet claims have always been with us. Reasoned analysis is a good defense.
Assume nothing, reasonably analyze & test claims. Konrad’s bluff & bombastic claims are found entirely lacking of reasoned analysis. Konrad’s central climate fallacy is unknowing, unreasoned analysis of known experiment.

August 5, 2014 6:16 am

H Grouse says:
August 4, 2014 at 10:38 am
Grab a mercury thermometer.
The device does not measure temperature.
Not really all that different than the instruments aboard a satellite????

——————-
You can keep on citing examples “until the cows come home” …. but they will not serve your purpose.
All thermocouples and thermometers are thermal “heat” energy measuring devices that rely on the “conduction” of said energy, …. directly from whatever molecular entity they are designed to measure, …… for their correct functioning regardless of what their “medium-of-exchange” is, ….. be it mercury, alcohol, water, a coiled spring, etc.
Satellites, IR cameras, rattlesnakes, insects, pythons, other pit vipers, etc. rely on the detection of specific wavelengths of “radiation” in the Infrared region of the electromagnetic spectrum …. for their correct functioning regardless of what the source of the IR is.
And like satellites, ….. insects and rattlesnakes can not measure temperature. Pit vipers have evolved the ability to “detect” a specific frequency of IR radiation which is emitted from their various “warm-blooded” prey animals.
An “air” thermometer is used to measure the temperature of ALL of the gas molecules in the near-surface air surrounding said thermometer, ….. and not just a specific gas molecule such as CO2 or whatever.
And don’t be forgettin, …. the majority of all increases in near-surface air temperatures are the direct result of “conduction” from the surface.

Trick
August 5, 2014 6:30 am

Kristian 4:59am asks: “…”DWLWIR (‘atmospheric back radiation’) to be a real, separate working flow of energy to the surface (an extra (and equal) input of energy next to the solar heat flux): If this were really the case, then why aren’t we harnessing this energy flux?”
As I’ve pointed out to Kristian before, the known 2nd law of thermodynamics deters efforts due the top post fallacy of separating out this particular energy flux to harness. There will always be a loss in trying to do so – proven by extensive test and reasoned analysis in intro. text books. All matter – gas, liquid, solid – at all temperatures radiates at all frequencies all the time. An atm. is matter so radiates at all frequencies at all temperatures all the time although in varying amounts.

August 5, 2014 8:23 am

That sort of comment does not go down well with his lordship. He called me a SL***r for expressing the same notion on this thread.

August 5, 2014 8:25 am

I refer to Chris Wright’s at 5:43.

August 5, 2014 9:40 am

Steve Oregon says:
August 4, 2014 at 4:37 pm
During all of my searches I’ve noticed that all of the government and alarmist sites avoid any mention of proportions. They speak about tonnes of CO2 etc. but not the EPA, NASA or any other I could find mention the percentages.
—————
Steve O, there is a primary “reason for their madness” that they speak about “tonnes of CO2” which has its origin in the work conducted by Charles Keeling that began producing more accurate and reliable results in 1958. And I cite the Keeling Curve Graph and/or Mauna Loa CO2 ppm measurements in conformation of the above.
In the early 1980’s the proponents of CAGW, …. and James Hansen et el specifically, needed some form of physical evidence to prove and/or justify their “junk science” claims that human emissions of their claimed “greenhouse” gas CO2 was the culprit directly responsible for their “fuzzy math” calculated increases in/of Average Surface Temperatures (Global Warming) via the highly questionable Surface Temperature Records being maintained by the NWS. See: History of the NWS – 1870 to present @ http://www.nws.noaa.gov/pa/history/evolution.php
Anyway, the Mauna Loa data has steadily and consistently shown an average 1 to 2 ppm annual increase in atmospheric CO2 from 1958 thru the 1980’s …. which was proof enough for Hansen et el that human emissions were partly or totally responsible for. (And ps, that steady and consistent increase of CO2 ppm continued on thru to present, 2014 )
And if the average mass of the atmosphere had been calculated to be about 5 quadrillion (5,000,000,000,000,000) metric tons, ….. then atmospheric CO2 in the 300+ ppm (0.03+%) range would mean that every one (1) ppm of CO2 would be equal to about 5 billion tons of CO2.
Therefore, Hansen et el now had the “magic number” of ….. 5bt/1ppm CO2 ….. for converting ….. “calculated temperature increases directly to increases in atmospheric CO2 ppm”.
And thus, …. all Hansen et el had to do then was to insure that the highly questionable estimations of total yearly human emissions of CO2 were stated in “tons of CO2” of sufficient quantities that would correspond to the aforesaid ….. average 1 to 2 ppm annual increase in atmospheric CO2 as defined by the Mauna Loa record.
But, when their “calculated temperature increases stalled” …… with “the pause”, …… but the annual average increase in atmospheric CO2 ppm” ….. did not “stall” with “the pause”, ….. then there was “
BIG trouble in River City”.

August 5, 2014 10:38 am

ferdberple says:
August 4, 2014 at 6:50 pm
there is one and only one test that will tell us the effects of CO2. And that is to observe what happens when CO2 is released PRIOR to warming. Do we see any CONSISTENT evidence of warming following natural events that release CO2?
—————–
Now I devised a cheap and simple experiment … like 10 years ago …. that will permit one to accurately measure the effects, … if any, …. that an increase in CO2 ppm will have on near surface air temperatures.
Just build two (2) identical frameworks, ……. say 20′ x 10′ x 8′ square, …. out of 1/2″ plastic pipe, …. outside in an area where each will receive the same amount of Sunshine, ……. place calibrated temperature sensors (thermocouples) inside of them that are connected to an auto-recording unit outside of said frameworks, ……… then cover them “air tight” with 4 mil clear plastic sheeting including the floors of each unit …… and when the night time temperatures in both stabilizes and reads the same degree F, …….. then at say 2 AM inject enough CO2 at the same degree F in one of them to increase the current 400+- ppm of CO2 to say 800 ppm of CO2. Thus, both units should be at the same degree F.
Then turn on the auto-recorder and record the temperatures in both structure …… and again every hour on the hour (or every half hour, or ten minutes) ……. for the next 24 hours.
And if CO2 is the global warming “greenhouse”gas that all the proponents of CAGW claims it is, then when the Sun rises in the morning and starts shining on the structures, the temperature in the structure containing 800 ppm CO2 ……. should start increasing sooner and faster and reach a greater temperature than in the other structure ….. and when the Sun starts setting the temperature inside the structure with 800 ppm CO2 should remain higher than it is in the other structure up until and past the 2 AM starting point ….. which will be confirmed by the “data log record” …. which will also include any actual “degree” increase in temperature.
And if it doesn’t, …. then the CO2 causing AGW claims are totally FUBAR.

August 5, 2014 10:48 am

TimNitOz says:
August 4, 2014 at 7:02 pm
This is a continual work in progress, but at this stage (it took a while) I’ve discovered that my entire set of truths are beliefs, and I actually do “know”, very little, or maybe, nothing.
—————–
Well now, … congratulations TimNitOz, …. me thinks you have just discovered the literal truth …. of the absolute fact that …… “You are what your environment nurtured you to be”.

August 5, 2014 10:56 am

Trick says, August 5, 2014 at 6:30 am:
“All matter – gas, liquid, solid – at all temperatures radiates at all frequencies all the time. An atm. is matter so radiates at all frequencies at all temperatures all the time although in varying amounts.”
So how much would an atmosphere at an average of say 255K consisting purely of N2 and O2 radiate to its surroundings? Since, as you say, an atmosphere is matter and all matter – gas, liquid, solid – radiates at all frequencies at all temperatures all the time …

kadaka (KD Knoebel)
August 5, 2014 10:57 am

Now, the problem with the theorem of Pythagoras is that it is unquestionably true. For those who agree with the philosopher Schopenhauer that Euclid’s demonstration is “a triumph of perversity”, here is the admirably clear proof by dissection attributed to the fifth-century Hindu mathematican Aryabhatta.
Am I the only one here willing to point out that is not admirably clear? The extra design frippery makes it much more confusing. Lord knows I’ve seen enough optical illusions to question what deception the coloration and possibly variable widths of the extraneous border pieces are hiding. For example, two opposing triangles maintain their orientation and border color cues between panes, but the other opposing set is apparently mirrored yet with different color cues when it is passed off as the same in the second pane. Why?
Interestingly, Googling for “pythagoras proof Aryabhatta” shows that graphic used in a 2010 Bob Arthy interview of Monckton. Which is packaged next to “… and Monckton’s proof by inclusion” which is also too cutesy but without consistent use of bordering, and messy, and not clear.
Further, I have seen more than enough previous claims of cherrypicking to find that it shows anything but a clever combination of lengths that just happen to work, it is not to be taken as a general proof. It lacks enough descriptive detail to show otherwise.
In addition, my spellchecker says “mathematician” was misspelled, while Wikipedia says “Aryabhata” was also misspelled, with “every astronomical text” spelling it as indicated.
And while Monckton says this proof is attributed to Aryabhata, I cannot find such. But I can find this proof by dissection (aka proof by rearrangement) attributed to Pythagoras himself, as given here, and notably without the mirrored set of triangles.
With Pythagoras living circa 570-495 BC, and Aryabhata 476-550 AD, the attribution of the proof to Pythagoras takes precedence until demonstrated otherwise.
And now that I’ve vented about that cluster-coupled frou-frou plopped in my path that prevented me from getting into the flow of piece, that artsy child’s wooden puzzle allegedly containing an “admirably clear proof”, maybe I can finally read it through, if I can find the time!

Matthew R Marler
August 5, 2014 12:08 pm

Kristian: It is not a separate, thermodynamically working flow of energy.
I’ll have to leave you there. I think you are wrong, but if you are right then I am wrong.

n.n
August 5, 2014 12:22 pm

It’s difficult for sophisticated people to acknowledge what they don’t know and especially what cannot be known. Science is distinguished from other philosophies by its intentionally constrained scope in time and space. However, as people seek to answer fundamental questions (e.g. origin of life), they callously exploit and corrupt science in order to mold a perception of knowledge, and to marginalize competing interests. Unfortunately, this has predictably (i.e. with precedent) corrupted science, sabotaging its utility and function.

kadaka (KD Knoebel)
August 5, 2014 12:40 pm

From Samuel C Cogar on August 5, 2014 at 10:38 am:

Now I devised a cheap and simple experiment … like 10 years ago …. that will permit one to accurately measure the effects, … if any, …. that an increase in CO2 ppm will have on near surface air temperatures.

Except it doesn’t. You propose filling two identical structures with differing concentrations of CO2, as if CO2 was insulation. But the CO2 that provides the greenhouse effect is ABOVE the surface, not AT the surface.
Thus your little boxes would not work. Thus all your proposed experiment can demonstrate is you don’t understand enough about how greenhouse gases work to begin to test how well they work.
Plus by the logarithmic effect of CO2, 400 to 800 ppm actually won’t show much, as the effect is already essentially saturated at 400ppm. Again, design fail. Try pure nitrogen vs pure nitrogen with 400ppm CO2, once you figure out an effective experiment design.

August 5, 2014 2:10 pm

kadaka (KD Knoebel) says, August 5, 2014 at 12:40 pm:
“Except it doesn’t. You propose filling two identical structures with differing concentrations of CO2, as if CO2 was insulation. But the CO2 that provides the greenhouse effect is ABOVE the surface, not AT the surface.
Thus your little boxes would not work. Thus all your proposed experiment can demonstrate is you don’t understand enough about how greenhouse gases work to begin to test how well they work.”

Meaning there has never been an experiment in the history of humanity really showing us what effect increased CO2 in the atmosphere will have on the surface temperature. So far it’s all been purely theoretical conjecture.
So how do we test your proposed CO2 warming mechanism in the lab? How do we begin to test how well it supposedly works?

milodonharlani
August 5, 2014 2:24 pm

It’s easy to show that the effect of doubling CO2 from 300 to 600 ppm in dry air would negligible at best.
Consider first the case of the moist tropics. There you have about 40,000 ppm of the GHG water vapor. Adding 300 ppm of CO2 means the total of the two far & away most important GHGs would grow from 40,300 to 40,600 ppm, ie practically no increase, especially when considering the logarithmic nature of the effect.
Consider now the dry high latitudes, especially polar. There water vapor concentration might be of about the same magnitude as CO2, ie three or four molecules per 10,000 dry air molecules. So, starting from three H2O & three CO2 molecules for a total of six per 10,000, jumping to seven still means practically no effect, due to initial thinness rather than abundance of water.
Same goes for all the cases in between, in the temperate zone & dry tropics. The global average for water vapor level is probably around 30,000 ppm. CACA should have been laughed out of the room long ago.

milodonharlani
August 5, 2014 2:27 pm

PS: The Team knows this, which is why their GIGO models make such absurd, unfounded assumptions about water vapor feedback.

Trick
August 5, 2014 2:41 pm

Kristian 10:56am: “So how much would an atmosphere at an average of say 255K consisting purely of N2 and O2 radiate to its surroundings?”
Good question, see Christopher’s top post discussion concerning “Stacey” adopting a sound, scientific approach. For a detail discussion, I would refer you to the link I posted for Konrad 6:12am section S.3 and Fig. S5 for reasoned analysis of IR optical depths at 1 bar in hypothetical but plausible planetary atmospheres compared to observed. N2 & H2 et. al. are discussed but not O2, so for your answer you are going to have to do some line by line radiative transfer integration of your own given the cited ref. work methods.
Eqn. S19 will be helpful in your approach to an answer also – using the gas mixing ratios of your choice, mass extinction coefficient (kappa) for each gas species and some reasoned analysis.
Back of the envelope with some “unknowns are known” (Christopher term) parameters to play with: as a surface 1bar atm. in such as Earth’s orbit was reduced toward primarily N2, O2 from current composition, the emissivity measured looking up from the surface would substantially reduce (optical depth tau become less thick) from global 0.8 to a much lower number and the surface Tmean approach 255K by simple 1st law, Planck distribution, Fourier conduction and fluid convection reasoned analysis based on lab experiment.
At measured N2, O2 global atm. emissivity of say 0.1 (from the line-by-line integration surface to TOA) looking up from surface for example, the atm. component of DWLWIR radiating to surroundings would be about 13 W/m^2 at a surface global Tmean of about 258K once approx. long term equilibrium was established with the sun & albedo as is in the current epoch.
******
Kristian 2:10pm: Concur. See Christopher in top post already coming to your conclusion:
…the honest answer to Stacey’s question about what climate sensitivity is (to CO2 forcing) indicated by the temperature record of the past 30 years is, “We do not know”.
To answer your new questions though: Looking over centuries of thermometry, there is some basic physics since Callendar’s 1938 paper estimating & predicting what the added CO2 ppm sensitivity should be from lab test and it came out reasonably close to observed global experiment correcting for the actual observed ppm CO2. The prediction is near the error bars of the thermometry record so observe the number of resulting blog posts – that alone should tell you something.
******
milodonharlani 2:24pm: ”..practically no effect, due to initial thinness rather than abundance of water….”
The emissivity of Earth atm. composition measured looking up from surface in the moist tropics is ~0.95, in the drier polar regions it is ~0.7 so there is real effect of atm. water vapor content in a column, not so much from well mixed CO2 – but not quite negligible as Callendar 1938 paper demonstrates from lab test and reasoned analysis.

Monckton of Brenchley
August 5, 2014 3:30 pm

Mr Knoebel sneeringly and futilely quibbles about the spelling of Aryabhatta (the older transliteration, still widely used in India). He prefers Aryabhata. If he wanted to be successful in being pathetically picky and pusillanimously pedantic he should really have spelt it Arya-Bhata.
Then he says he can’t understand Aryabhatta’s proof and doesn’t think it rigorous. Well, since his idea of doing homework is to trawl the internet, where he seems to be saying he was unable to find the spelling Aryabhatta, cites Creepymedia (the most unreliable source of information on the planet) and says he was unable to find the proof by dissection attributed to Aryabhatta, let him try a little harder and look up the proof for himself, and think about it before shooting his mouth off.
He also implies that I had said Pythagoras’ proof was in fact first demonstrated by Aryabhatta, when I had manifestly said no such thing. If he had wanted to be successfully picky rather than merely inadequate, he would have mentioned that the theorem was certainly known to the Babylonians long before Pythagoras. Perhaps he would have preferred it to be called the Babylonian Theorem. Then he could have quibbled and driveled about the spelling of “Babylonian” as well,
And let him not, in future, sneer to the effect that something he does not understand is likely to be incorrect. On his past form here, it is more likely that he is simply not bright enough to grasp what children of 11 have not the slightest difficulty in understanding. More adult standards of conduct and of intellect are expected here. And, in future, for Heaven’s sake stick to the main point, which is that – whether Mr Knoebel likes it or understands it or not – the theorem of Pythagoras is true.

milodonharlani
August 5, 2014 3:37 pm

Trick says:
August 5, 2014 at 2:41 pm
I expressed that part of my argument poorly. There is some effect even from a small amount of water vapor in the Arctic & Antarctic. The addition of one more molecule of CO2 per 10,000 dry air molecules to this thin envelope of GHGs however would indeed have little effect, especially when considering the logarithmic nature of the effect.

milodonharlani
August 5, 2014 3:41 pm

Or three more molecules, for the doubling from three to six.

Monckton of Brenchley
August 5, 2014 3:42 pm

I note that “Konrad”, who seems confused, is unable to provide a mean emission temperature for the Earth other than the 255 K that I did not – as he alleges – assume or accept. I went back to first principles and calculated it for myself, assuming emissivity of unity: for, with respect to the blackbody radiation we are dealing with, the Earth’s surface (after allowance for albedo) has an emissivity close enough to unity to avoid significant error. I asked him to provide and justify an emissivity other than unity, and he responded with a vague comment about the Earth’s surface temperature. I asked him whether the experiments for which he had been unable to give me even a result had been peer-reviewed, and I take it from his reply that they have not. I repeat that I do not yet see any reason to take his suggestions seriously.

Monckton of Brenchley
August 5, 2014 4:15 pm

Mr Wright thinks climate sensitivity to CO2 is “one of the great scientific questions, a bit like the age of the universe”. Not really. If climate sensitivity is small it is unimportant. If it is large, it may or may not be important. Since so many unknowns are present, it is not at present definitively determinable. Yet Mr Wright says he thinks climate sensitivity is likely to be immeasurably small.
His arguments are as follows:
1. “Nearly half of the modern warming occurred before there was sufficient CO2, so clearly something else is at work”. That is the argument from false cause. Merely because natural warming can be shown to exist, one should not assume that CO2-driven warming does not exist.
2. “There is no empirical scientific data that shows a change in CO2 followed by a corresponding change in temperature. Surely, if the sensitivity is one degree, it should show up repeatedly in the ice core data? Of course, the CO2 follows the temperature, and not the other way around.” That is the post-hoc-ergo-propter-hoc or Wafter, therefore because” fallacy. It is possible that when some astronomical or volcanic event shifted the temperature regime on Earth, the uptake or outgassing of CO2 in the oceans provided feedbacks amplifying the change, and there is no method of disproving (or, for that matter, proving) that assertion of the true-believers.
3. In the Holocene, each climate maximum – Old Kingdom, Minoan, Roman, Medieval, modern – was less warm than its predecessors. That is the statistical fallacy of assuming that trends are predictive. The usual suspects may well be right when they say we are liberating CO2 to the atmosphere from which it came so fast that its concentration is unprecedented in 800,000 years (indeed, perhaps even in 20 million years). Since elementary measurements and theory would lead us to expect some warming from CO2, that is what I expect.
4. “AGW theory makes a number of specific forecasts: the equatorial hot spot, forecast warming, and the trend of Earth’s infra red emissions. The emissions are predicted by AGW to be getting smaller, but Lindzen showed that, not only is the size of the trend wrong, it has the wrong sign.” Many of the forecasts are wrong, and I calculate that the forecasts of CO2-driven warming are exaggerated. But the fact that many forecasts are wrong does not imply that all are wrong: that is the inappropriate argument from the particular to the general that is the fallacy of accident. And Professor Lindzen did not show that the trend in infrared emissions from Earth is the wrong size and the wrong sign: the measurements from the ERBE and CERES satellites are in the right ballpark, and they show a decline in outgoing radiation that is particularly evident at the wavelengths corresponding to the principal absorption bands of CO2. He found that the sign of the temperature feedback sum was likely to be somewhat negative rather than strongly positive, concluding that climate sensitivity was about 0.7 K per CO2 doubling – in other words, in much the same ballpark as my own “less than 1 K” from back in 2008.
5. “Much of climate science is corrupt almost beyond belief.” That is not really fair to climate science. Some of it is undeniably corrupt, and if the scientists who knew it to be corrupt had arranged for one or two of the worst offenders to be prosecuted there would no longer be a climate scare. However, to assume that because much of climate science is corrupt all of it is corrupt is to perpetrate yet again the fallacy of accident.
All five of Mr Wright’s arguments for zero sensitivity are instances of common fallacies.
Mr Wright also asks what evidence there is to support a climate sensitivity as high as 1 K per CO2 doubling. One answer is that the surface temperature of the Earth is some 34 degrees higher than its emission temperature. Greenhouse gases account for the difference, in approximately the proportions set out in various scientific papers over the past couple of decades. Adding more greenhouse gas will cause more warming. How much, or how little, is the big question. I calculate there will be some modest warming, but not enough to worry about.

August 5, 2014 7:18 pm

“One answer is that the surface temperature of the Earth is some 34 degrees higher than its emission temperature. Greenhouse gases account for the difference,..”
That would be true if it were not for the massive thermal reservoir of the oceans. It is not long-wave radiation that makes the global surface mean T of the oceans 7.5°C higher than the global land surface mean T.

kadaka (KD Knoebel)
August 5, 2014 7:59 pm

From Monckton of Brenchley on August 5, 2014 at 3:30 pm:

Then he says he can’t understand Aryabhatta’s proof and doesn’t think it rigorous.

The proof shown by the graphic is indeed rigorous, especially when properly notated and drawn, and is more properly attributed to Pythagoras. If you had indeed read my words then you shouldn’t be so obviously misrepresenting what I had said.

He also implies that I had said Pythagoras’ proof was in fact first demonstrated by Aryabhatta, when I had manifestly said no such thing.

I noted an apparent error in attribution, with reference. Such attribution errors can be mundane matters given the fragmentary histories.
But at a depth clearly greater than what he bothered to comprehend of my writings, as such a proof is normally attributed to the first one to present it, by attributing that proof to Aryabhata he did indeed claim it as first demonstrated by Aryabhata, rather than make the better attribution to Pythagoras.

And let him not, in future, sneer to the effect that something he does not understand is likely to be incorrect.

Meanwhile Monckton has clearly done his sneering without understanding what I wrote. Apparently his time is so valuable he finds it efficacious to spew falsehoods based on a quick glance rather than attempt comprehension.
Since Monckton did not provide countering evidence that the proof is better attributed to Aryabhata than Pythagoras, the attribution thus stands as better belonging to Pythagoras.
Of course, indicative of the negligible attention he gave to my comment, he did say:

Well, since his idea of doing homework is to trawl the internet, where he seems to be saying he was unable to find the spelling Aryabhatta…

When I clearly stated in my comment, presented here with bold added to make it obvious:

Interestingly, Googling for “pythagoras proof Aryabhatta” shows that graphic used in a 2010 Bob Arthy interview of Monckton.

I show something I found on the internet using his spelling, he thinks I’m saying I was unable to find that spelling on the internet.
And indeed, I found many things using his spelling, as it is a common misspelling, thus such findings are expected.
This is saddening. I had expected better of Christopher Monckton, now a decidedly ex-journalist.

Dr. Strangelove
August 5, 2014 8:14 pm

Silly those who doubt the ocean is nearly a blackbody. Do they doubt the ocean is water? Or they doubt all the experiments that measured 0.95 the emissivity of water?
Those who doubt the existence of downward longwave radiation (DLR). This has been measured all over the world at around 300 W/m^2. Read and learn.
http://scienceofdoom.com/2010/07/17/the-amazing-case-of-back-radiation/
“…”DWLWIR (‘atmospheric back radiation’) to be a real, separate working flow of energy to the surface (an extra (and equal) input of energy next to the solar heat flux): If this were really the case, then why aren’t we harnessing this energy flux?”
Actually we are harnessing it. Without DLR the sea surface will freeze. Ocean Thermal Energy Conversion (OTEC) harnesses the temperature difference between the warm sea surface and cool deep water to generate power. Tokyo Electric Power Co. produced 120 kW of electricity in 1981 using OTEC technology.

Konrad
August 5, 2014 9:23 pm

Monckton of Brenchley says:
August 5, 2014 at 3:42 pm
—————————————
Viscount Monckton,
In re-reading this thread I see some of my comments can be view as overly antagonistic. I should clarify that I appreciate much of what you write and have enjoyed attending some of your talks when you toured overseas. From those talks I would accept that you would have the experience to fairly level the charge of “bombastic” against me 😉
As to “bluff”, not so much. Anyone is free to repeat my experiments and check my claims.
As to my “confusion”, let me clarify. I understand the basic radiative physics you are using. I have shown may times the refined design of an early empirical experiment into the two shell radiative model –
http://i44.tinypic.com/2n0q72w.jpg
http://i43.tinypic.com/33dwg2g.jpg
http://i43.tinypic.com/2wrlris.jpg
– it works, and the target plate in chamber 1 will reach the higher temperature. But it all goes pear shaped if you don’t use the vacuum pump. What if you replaced the target plates with a selective surface (emissivity lower than absorptivity) and allowed conductive gas coupling between the target plate and black foil in chamber one? Now the target plate in chamber 2 would reach the higher temperature. This is the situation on Planet ocean. The ocean surface is conductively, evaporatively and radiately coupled to a radiative atmosphere that is a better radiator than the ocean surface.
You say –
“ I went back to first principles and calculated it for myself, assuming emissivity of unity: for, with respect to the blackbody radiation we are dealing with, the Earth’s surface (after allowance for albedo) has an emissivity close enough to unity to avoid significant error.”
It is not just the assumption of unity (ratio of emissivity and absorbivity 1:1) I am challenging. I show that there are dual selective surface (or more accurately selective material) effects in play with regard our oceans.
“I asked him to provide and justify an emissivity other than unity, and he responded with a vague comment about the Earth’s surface temperature.”
There have been many experiments to determine the emissivity of the oceans. One of the results that was a big read flag was the apparent emissivity changing with viewing angle and surface roughness. A near blackbody does not respond in this manner. Ie: while and emissivity setting of 0.95 is fine for calibrating an IR instrument to counter Hohlrumn/cavity effect when measuring water under a radiative sky, this figure should not be used for radiative balance calculations. You are correct to claim I do not have an exact figure for the effective (not apparent) emissivity of water, but I do not have the funds to drop background IR to 3K over a sample. I did however show you a simple experiment in which background IR was reduced under a -40C “sky” –
http://i61.tinypic.com/24ozslk.jpg
– and I did say the result for a 40C water sample was apparent emissivity dropping below 0.8. That result alone should have been enough (peer-reviewed or not) to indicate that there was something very wrong with the assumption of emissivity of unity for the oceans.
“I asked him whether the experiments for which he had been unable to give me even a result had been peer-reviewed, and I take it from his reply that they have not.”
I did mention that these experiments-
http://oi62.tinypic.com/zn7a4y.jpg
http://oi61.tinypic.com/or5rv9.jpg
-were covering peer reviewed results from 1965. This is the relevant paper –
“Harris, W. B., Davison, R. R., and Hood, D. W. (1965) ‘Design and operating characteristics of an experimental solar water heater’ Solar Energy, 9(4), pp. 193-196.”
What they covered was design issues with evaporation constrained (NOT salt gradient) solar ponds –
http://oi62.tinypic.com/1ekg8o.jpg
(Ignore the use of the the word glass on the old image, consider only IR transparent LDPE film) They found a surprising result. When layer 2 was made matt black, despite now absorbing more SW, the pond did not heat as well. Surface temperatures were around 30C higher than water just millimetres below. Average temperature was far higher if layer 2 was clear and layer 3 black.
I demonstrate this effect more clearly in “selective surface experiment 1” linked above. Here two blocks have equal SW absorption and IR emissivity, yet have a dramatic difference in temperature when exposed to SW due solely to the differing depth of SW absorption. Full sun for 3 hours and block A will have a 20C higher average temperature, and a dramatic 40C base temperature differential will develop between the target blocks.
This empirically demonstrable effect is clearly missing from the base assumption that ~240 w/m2 can only heat our oceans to 255K.
“I repeat that I do not yet see any reason to take his suggestions seriously.”
If you build and run the experiments for yourself you would see the reason. Even better, you could repeat the Texas A&M experiments using IR transparent film in deep ponds at 6000m in the Atacama desert. The air is dry and there is little DWLWIR. Try one pond(A) with layer 2 matt black and pond(B) layer 2 clear and layer 3 black. Pond A will have extreme surface temp variation but an average temperature close to freezing (ie: close to the prediction of climastrologists). Pond B would be destroyed by steam overpressure in a dramatic demonstration that the assumption that the “oceans are a near blackbody” is utterly incorrect 😉

Monckton of Brenchley
August 6, 2014 12:18 am

Mr Knoebel continues to do what he does best – pick nits. I have already made it plain that, since he came here with a sneering tone he must do his own homework to find the attribution of the proof without words to Aryabhatta (that spelling is used in India with more frequency than the spelling he prefers on the dubious ground that Creepymedia recommends it, and is also common on the internet, where Mr Knoebel seems to do his primary research). Let him go into a real library and read some proper books. It will not take him long to find the attribution to Aryabhatta. And his assertion that because Pythagoras predated Aryabhatta the attribution of the proof to the former is preferable is an interesting variant of the post-hoc-ergo-propter-hoc subspecies of the fallacy of false cause. His quibble about the fact that my simple diagram exhibits symmetry about one of the diagonal axes, unlike the images he has seen on the internet, demonstrates how little he understands the proof, for my rearrangement of the “puzzle pieces” makes not the slightest difference to its efficacy. He now whines – for he is a serial whiner – that the proof should have been annotated: however, since the children of 11 to whom I have shown this proof are capable of grasping it instantly, and since the proof was merely an incidental ornament to a head posting on another topic and most of the readers here have a mental age >11, I had no need to annotate it. In Mr Knoebel’s first futile posting on this side issue plainly doubted that the proof was rigorous – with one of his characteristic sneers. Now he accepts that it is rigorous after all. That is some progress, I suppose.
“Konrad” continues to be confused. He says the ocean is a “selective surface”. No, it isn’t. Then he says maybe the coupled ocean-atmosphere system is a selective surface. No, it isn’t: the mean emission altitude of the coupled system is many miles above the ocean surface. He says the emissivity of the ocean is not close to unity, as all recent measurements suggest it is, but continues to fail to provide an alternative value, let alone any credible reasons for it. He says he thinks water at 40 C has an emissivity of 0.8, but what relevance that has to an ocean that exhibits such temperatures almost nowhere on Earth is not explained. I can see increasing reason not to regard his case as credible. Let him draw it up in a fashion less unsystematic and better verified than what he has presented here, and let him send it to a journal for peer review. As far as I know, not a single paper following that which he cited finds ocean emissivity to be anything other than the usual value in the region of 0.95.

QuixoteNexus
August 6, 2014 1:51 am

http://journals.ametsoc.org/doi/full/10.1175/2011JCLI4210.1
All is not well in the World of DWLDIR.
H/T Griss-JoNova

QuixoteNexus
August 6, 2014 1:52 am

Duh ,,DWLWIR

Dr. Strangelove
August 6, 2014 1:53 am

“One of the results that was a big read flag was the apparent emissivity changing with viewing angle and surface roughness. A near blackbody does not respond in this manner.”
Measurements of emissivity of materials are done with direct radiation, the rays are perpendicular to the surface. Changing the angle of incidence will change the radiative flux on the surface. The results are no longer comparable. A light source emits 100 W/m^2 directly on a blackbody surface. The body emits 100 W/m^2. Change the angle of incidence. The body emits 80 W/m^2. Aha! The emissivity is only 0.8
This is apple and orange. The direct radiation changed to 80 W/m^2 though the light source still emits 100 W/m^2. The blackbody is still black.

Konrad
August 6, 2014 2:58 am

Monckton of Brenchley says:
August 6, 2014 at 12:18 am
—————————————–
Mr Knoebel chose to get into a maths fight with a provably accomplished mathematician. Not dissimilar to making close visual inspection of the intake and/or exhaust of a running Trent 800 turbofan engine. The results are predictable and nothing to crow about.
But as to the rest…
“Konrad continues to be confused.”
Only in so far as to why you don’t pay attention to empirical evidence place right under your nose.
“He says the ocean is a “selective surface”. No, it isn’t.”
Is. Isn’t. Is. Isn’t. Play as long as you like. I still win. Emissivity less than absorptivity. UV/SW absorption at depth not surface. Slow speed of internal non-radiative transport. That’s a selective surface. No way out of that.
“Then he says maybe the coupled ocean-atmosphere system is a selective surface. No, it isn’t:”
Well, no I didn’t. Coupled ocean atmosphere emits near theoretical blackbody, around 255K. Please don’t ever try and mis-quote me.*
“the mean emission altitude of the coupled system is many miles above the ocean surface.”
Oh do please buy yourself some IR measurement gear as I have done. ERL? Garbage! Get outside. Measure the sky! The strongest emission is from cloud (see if you can catch the IR pulse during condensation). ERL is a mathematical fiction.
“He says the emissivity of the ocean is not close to unity, as all recent measurements suggest it is, but continues to fail to provide an alternative value, let alone any credible reasons for it. He says he thinks water at 40 C has an emissivity of 0.8, but what relevance that has to an ocean that exhibits such temperatures almost nowhere on Earth is not explained.”
Come now Christopher, here you are just struggling. I have empirically demonstrated the difference between apparent and effective emissivity. “all recent measurements”?! Got any where water was measured as a thin film less than 100 microns thick under a 3K “sky”. Of course you don’t. (side note – I suspect someone smarter than me may have attempted this as 0.67 appears in many old tables ands texts.)
“I can see increasing reason not to regard his case as credible. Let him draw it up in a fashion less unsystematic and better verified than what he has presented here, and let him send it to a journal for peer review. As far as I know, not a single paper following that which he cited finds ocean emissivity to be anything other than the usual value in the region of 0.95.”
Sadly for you claims I have been very systematic. First I empirically checked whether incident LWIR slows the cooling rate of water that is free to evaporatively cool. (it doesn’t, so game over for AGW). But I am systematic. I didn’t let it rest. I went further. I found how the sun alone is enough to heat the oceans far beyond 15C if it were not for atmospheric cooling.
And emissivity of 0.95? That’s apparent not effective emissivity. If you don’t understand the difference, then perhaps you should not have brought maths to a physics fight 😉
*As to the mis-quoting thing…here, so you have no further excuse in this area, I have a Oh-So-Easy cut and paste list of 22 basic claims of “Konrad” you can refer to in future (all in the interest of scientific accuracy 😉 )
1. DWLWIR cannot heat nor slow the cooling rate of the oceans.
2. Radiative gases play a critical role in radiative subsidence of air masses in vertical tropospheric convective circulation in the Hadley, Ferrel and Polar cells.
3. The power of radiative gases to cool our atmosphere is double their power to warm it.
4. Without radiative gases our atmosphere would super heat.
5. The surface is far more effective at conductively heating the atmosphere than it is at cooling it.
6. The oceans are a UV/SW selective surface not a “near blackbody”
7. The effective IR emissivity of the oceans is far below their apparent IR emissivity.
8. Solar variation drives climate change by variance in UV heating below the ocean thermocline.
9. Radiative gases act to cool our atmosphere at all concentrations above 0.0ppm.
10. Adding radiative gases to the atmosphere will not reduce the atmosphere’s radiative cooling ability.
11. The net effect of our radiative atmosphere, excepting pressure, over the oceans is cooling of the oceans.
12. Regardless of DWLWIR, without atmospheric cooling our oceans would heat to 80C or beyond.
13. Radiative gases are the atmosphere’s only effective cooling mechanism.
14. The sun heats the oceans.
15. The atmosphere cools the oceans.
16. Radiative gases cool the atmosphere.
17. 97% of climastrologists are assclowns.
18. 3% of climastrologists provably knew they were lying.
19. Global warming was in effect a global IQ test with results permanently recorded on the Internet.
20. Sceptics will never forgive and the Internet will never forget.
21. The corpse of the global warming hoax cannot be re-animated nor can it be hidden.
22. Every activist, journalist or politician who ever vilified sceptics is about to have their public face, metaphorically speaking, punched to custard. No excuses, no exceptions.

Chris Wright
August 6, 2014 3:34 am

Christopher,
I thought you would be able to classify my arguments as fallacies, and I wasn’t disappointed!
Just to clarify, I don’t mean to suggest that each point of the five is a proof in its own right. What I suggest is that, as a whole, they suggest a natural explanation for the warming. For example, it is odd that the modern warming is probably no greater than the previous ones, despite the claimed extra warming due to CO2.
I’ll concentrate on your final remarks, as it’s the most important. I asked for examples from historical data that shows a temperature change that followed a change in CO2. I note that you could not give a single example.
Instead, you cite the probable fact that the Earth is warmer due to greenhouse gases (of course, CO2 provides a tiny amount, about 95% of greenhouse warming comes from water vapour).
So, we are comparing two cases: zero greenhouse gases and the present. How many doublings does this represent?
Of course, it represents an infinite number of doublings. This is so different to what we’re talking about (an increase of just 50%), so I don’t think it helps. Curiously, if you ascend to a height on Venus where the atmosphere is Earth normal pressure, the temperature is also roughly Earth-normal – despite the massive amounts of CO2 above. The temperature on Venus rises dramatically only as you descend into the denser atmosphere. The same happens on Earth but – fortunately – it is far less extreme. This strongly suggests that atmospheric density is also a strong factor, and coud conceivably be far more significant than CO2.
If you consider Venus, Earth and Mars, the planetary temperature correlates with atmospheric density and distance from the Sun. But it does not correlate with the amount of CO2 (although the Martian atmosphere is very thin, it contains more CO2 per volume than Earth)..
Nature has conducted an amazing experiment over the last million years, with the climate going up and down like a yo-yo, and it carefully recorded the results in the ice.
If the ice cores show absolutely no evidence of a one degree sensitivity, then the only rational conclusion is that the sensitivity is far less, and is probably unmeasurable.
Best regards,
Chris

Konrad
August 6, 2014 3:45 am

Dr. Strangelove says:
August 6, 2014 at 1:53 am
———————————
Sad, just sad.
A true blackbody does not change apparent emissivity with viewing angle.
But I can’t expect any more from you.
Remember what you tried at Dr. Spencers site? Remember when you lied and claimed you had conducted one of my experiments? Remember when, not I, but another commenter replicated the experiment and shot your lies down before I could. The Internet remembers. Forever.
You don’t even rate as a “sleeper”. Poptech popped. Slick Nik slipped up. Kanada karked. And they were all better than you by far. Want to be good? Look to the “Hoff”. He’s real good. Shot down, kept quite and put many solid hours into re-building “cred”. Now that’s dedication to the “cause”. Compared to that you are just not in the running…

August 6, 2014 3:56 am

Dr. Strangelove says, August 5, 2014 at 8:14 pm:
“Those who doubt the existence of downward longwave radiation (DLR). This has been measured all over the world at around 300 W/m^2. Read and learn.
http://scienceofdoom.com/2010/07/17/the-amazing-case-of-back-radiation/

Sigh. How many times must this be explained? We are NOT ‘measuring’ DLR unless it comes in as HEAT, an actual, detectable transfer of energy. ‘Back radiation’ in a heat transfer situation is NOT detectable. It cannot be extracted (isolated) from the radiation field as a whole. People who think so don’t know what a radiation field is and how it works. We are calculating the DLR from the outgoing heat and the temp of the sensor (pyrgeometer) or estimating it from the incoming heat from the atmosphere to a severely cooled sensor (interferometer) (in which case the radiation could no longer be considered ‘back radiation’, the sensor doesn’t heat the atmosphere). Science of Doom is a guy who thinks ‘heat’ flows both ways in a thermal radiative exchange and mocks people who know better and try to point it out to him. He’s an embarrassment to himself.
“Actually we are harnessing it. Without DLR the sea surface will freeze. Ocean Thermal Energy Conversion (OTEC) harnesses the temperature difference between the warm sea surface and cool deep water to generate power. Tokyo Electric Power Co. produced 120 kW of electricity in 1981 using OTEC technology.”
Right.

Monckton of Brenchley
August 6, 2014 7:43 am

Mr Wright seems to consider that five logical fallacies, taken together, constitute a valid argument. In logic, though, five wrongs don’t make a right.
As for his final argument about the ice-core record, there is a statable case that after temperature changes had been triggered they caused CO2 concentration to change, which then amplified the temperature change. As far as I know, there were no events in the past 800,000 years that added enough CO2 to the atmosphere to cause warming – except the increases in temperature, which caused CO2 to outgas and hence may have caused some additional warming.
We must all be careful not to adopt a scientific position based on wishful thinking. By trying to pretend there is no greenhouse effect, or to pretend it is insignificant, we are unlikely to convince anyone with scientific training. If instead we point out that the temperature feedbacks in response to CO2-driven warming are likely to be small, then we have some chance of opening some closed minds.

August 6, 2014 8:33 am

kadaka (KD Knoebel) says:
August 5, 2014 at 12:40 pm
Except it doesn’t. You propose filling two identical structures with differing concentrations of CO2, as if CO2 was insulation”.
—————
BUT, BUT, BUT, …. the proponents of CAGW have always claimed that the insulating properties of atmospheric CO2 is the PRIMARY cause of Global Warming Climate Change. To wit:
Greenhouse gases
Greenhouse gases act as an insulation layer to trap some of the sun’s energy in the earth’s atmosphere, between the earth’s surface and this insulation layer. This is similar to the situation in a greenhouse – the glass walls allow heat and sunlight in during the day and trap it there so it can warm the plants that are growing inside.
” Ref link, to wit: http://www.landlearnnsw.org.au/sustainability/climate-change/what-is-it/greenhouse-gases
————–
These steady additions have begun to tip a delicate balance, significantly increasing the amount of greenhouse gases in the atmosphere, and enhancing their insulating effect”. Ref link, to wit: http://clinton5.nara.gov/Initiatives/Climate/greenhouse.html
————–
kadaka, iffen you are intent on “supporting a cause” ….. you really should familiarize yourself with the particulars of what you are supporting.
==============
But the CO2 that provides the greenhouse effect is ABOVE the surface, not AT the surface”.
———————
kadaka, I would sure be interested in knowing what physical property of CO2 that you are referring to that permits said CO2 molecule to determine its vertical “height/location” above the surface of the earth …. so that it can start absorbing and emitting thermal (heat) energy?
Anyway kadaka, iffen the 1st one of my above cited CAGW biased “quotes” about “greenhouse” gases CONFIRMS the fact that my stated “experiment” is scientifically sound then it is not silly for you to criticize it.

Trick
August 6, 2014 9:20 am

Kristian 8:14pm: ”Science of Doom is a guy who thinks ‘heat’ flows both ways in a thermal radiative exchange…”
It does! It is proven. In this case what you write has absolute proof even though it was very hard to come by & took some time – a lot of effort by many pioneers was required as top post says: ”In the physical sciences, absolute proof – which mathematicians call demonstration – is very seldom available. In mathematics, the art that is the language and the queen of the sciences..”
Think I’ve pointed out before you just remain behind in your reading of the absolute proof of your statement by lab test & math written down in several languages & accents. SOD was preceded in that discussion by a Scottish farm boy who developed lab tests & then provided the theory in support of an English book binder apprentice’s more extensive prior lab experiments (incentivized by a couple Frenchmen) showing the clip herein that you wrote to be true. A German then provided the lab experiment proving the Scot’s theory and an Austrian showed why. True multi-national collaboration story. Text books today still follow these guys. Catching up on your reading will enlighten you. Fill in their names for me; prove you did the research work.

August 6, 2014 9:40 am

Konrad says:
“1. DWLWIR cannot heat nor slow the cooling rate of the oceans.”
While short-wave radiation will warm both surface and sub-surface layers, long-wave radiation will cause a cooling of the surface depending on the temperature and humidity of the air:
http://onlinelibrary.wiley.com/doi/10.1111/j.2153-3490.1963.tb01399.x/pdf

August 6, 2014 10:01 am

“The central climate fallacy is that the unknowns are known”
The central fallacy is that natural variability is internal to the climate. Take the warm AMO since 1995, that is a negative feedback to declines in solar plasma forcing, which caused at least 0.2°C warming of the global surface mean T just by increasing poleward ocean transport.
“We cannot predict the behavior of the oceans – indeed, we cannot even measure changes in their heat content with sufficient resolution to give a meaningful result.
We cannot predict el Niño and la Niña events.”
The AMO and ENSO can both be forecast if you understand that they are both amplified negative feedbacks to the short term solar signal, and that you can forecast that signal. I forecast (last Autumn) for weak El Nino conditions for the first half of 2014, but weakening strongly from late July, accompanied by a rise in Arctic sea ice extent from late July.

Monckton of Brenchley
August 6, 2014 11:13 am

I am most interested in Mr Lyons’ statement that there is a link between solar events and el Nino, and would be most grateful if he were to email me further details.

August 6, 2014 11:35 am

Monckton of Brenchley says:
August 6, 2014 at 11:13 am
From my discussion of solar influences on climate in comments on the BOMBSHELL post, a paper on solar modulation of an ENSO-related climatic phenomenon:
http://onlinelibrary.wiley.com/doi/10.1002/jgrd.50453/abstract
Solar cycle modulation of the ENSO impact on the winter climate of East Asia
Qun Zhou1, Wen Chen1,* and Wen Zhou2
This study examines how the East Asian winter climate response to the El Niño-Southern Oscillation (ENSO) varies with the 11-year solar cycle. The results indicate that the ENSO and East Asian climate relationship is robust and significant during winters with low solar (LS) activity, with evident warming in the lower troposphere over East Asia, which can be closely linked to the decreased pressure gradient between the cold Eurasian continent and the warm Pacific. Moreover, during the LS and El Niño winters, there is a typical rainfall response in Southeast Asia, with wet conditions over South China and dry conditions over the Philippines, Borneo, Celebes, and Sulawesi, which can be explained by the anticyclone over the western North Pacific (WNP). However, during high solar activity winters, both the surface temperature and rainfall anomalies are much less closely associated with the ENSO. The possible mechanism for this solar modulation of the ENSO-related East Asian climate anomalies may be the change in the tropospheric circulation with the ENSO in both tropical and extratropical regions. Particularly, in the LS cases, an anomalous WNP anticyclone is intensified and a noticeable cyclone occupies northern Northeast Asia, resulting from the changing location and strength of the large-scale Walker circulation induced by the more pronounced sea surface temperature anomalies associated with the ENSO. Further investigation with long historic data confirms that the relationship between the ENSO and the East Asian winter climate anomalies depends on the phases of 11 year solar cycle, with enhanced East Asian climate variation during the LS winters.
There are lots of other papers with similar conclusions, including effects upon ENSO itself, such as the fact that El Niños are more common during the warm phase (warmer water in the East Pacific and cooler in the West) of the PDO, which oscillation is itself influenced by solar activity, and La Niñas during the cold phase.

August 6, 2014 11:55 am

milodonharlani says:
August 5, 2014 at 2:24 pm
Consider first the case of the moist tropics. There you have about 40,000 ppm of the GHG water vapor. Adding 300 ppm of CO2 means the total of the two far & away most important GHGs would grow from 40,300 to 40,600 ppm, ie practically no increase, especially when considering the logarithmic nature of the effect”.
—————-
milodonharlani, me thinks your above comment pretty much defines the “root problem” that exists in most every commentary/conversation concerning and/or related to the per se “greenhouse” gasses and their contribution to the “warming” of earth’s atmosphere.
It is of my opinion that a large majority of the aforesaid commentators … can not see the forest for the trees, …. and conversely, … can not see the trees for the forest. In other words, if their eyeballs and mind are “focused” on the trees …. then the forest is non-existent. And if their eyeballs and mind are “focused” on the forest …. then the trees are non-existent. But the literal fact is said forest and trees are mutually conjoined and thus inseparable.
And likewise, the per se “greenhouse” gasses are mutually conjoined via their similar/same physical properties associated with both the “conduction” of thermal (heat) energy …. and the absorption & emission of “radiated” Infrared thermal (heat) energy and thus are inseparable.
In other words, ….. “What’s good for the goose is good for the gander”.
Now, given the fact that most everyone agrees that both atmospheric CO2 and H2O vapor are per se “greenhouse” gasses …. and that the H2O vapor molecule is a far greater absorber/emitter of thermal (heat) energy than is the CO2 molecule …. and the fact that there is far, far, far greater quantities (up to 40,000+ ppm) of atmospheric H2O vapor than there is of CO2 (400 ppm) …… then “What’s good for the goose is good for the gander” still applies.
And with a “400 to 40,000 ratio” means that H20 vapor (humidity) will cause 100+ times more “warming” of the atmosphere than the CO2.
But like the “forest and the trees”, …. most everyone averts their eyes and their mind to the “warming” effects of the H20 vapor (humidity) while they are touting the “warming” effects of the CO2.
And I SPECIFICALLY stated H20 vapor (humidity) in the above to distinguish it from atmospheric H2O droplets that exist in the form of clouds, fogs and mists which have additional physical properties that permit them to “reflect” the energy in Visible Light as well as to act as a “bi-directional buffer” to/for the “radiated” Infrared thermal (heat) energy.
Now I did a rough calculation and decided that iffen 300 ppm of CO2 had the “warming” potential of causing a 1 degree F INCREASE in average temperatures …… then 40,000 ppm of H20 vapor (humidity) had the “warming” potential of causing ….. your socks to melt off your feet while standing knee-deep in water.
And iffen the “logarithmic nature of CO2” is as is described herein, to wit: http://knowledgedrift.wordpress.com/2011/09/07/co2-is-logarithmic-explained-3/ … then that is just more “junk science”.

August 6, 2014 12:17 pm

@Monckton of Brenchley
August 6, 2014 at 11:13 am
My LinkedIn profile has my contact details, if you can mail me first as I cannot find a contact for you, thanks.

Trick
August 6, 2014 1:02 pm

Ulric 9:40am: “…long-wave radiation will cause a cooling of the surface depending on the temperature and humidity of the air…”
Note in the top post the population ”…who have been sedulously deluded into believing that “the science is certain”… I had to look up “sedulously”. This issue you note is a good example.
The paper you link does not show what you write since it deals in T differences. It shows in Fig. 1 the temperature difference between surface air (assumed = surface water skin) and the water 0.5m below surface for increasing heat used for evaporation computed from wind speed & humidity. Shows the difference gets smaller for increasing evaporation. Tests show nothing about absolute T.
Again in Fig. 2 discussion, the paper plots this difference from others but concludes the bulk of work shows effects of “evaporation and radiation failed to be conclusive.”
The readings were recorded only to the nearest 0.01C so did not have the precision required for modern absolute sea surface skin temperature vs. DWLWIR readings. Modern ocean in-situ measurements run by U. Miami researchers with much more precise interferometry based instrumentation have reportedly indicated a lot of scatter (due in part waves radiating at each other & spray) for roughly .004K increase in ocean skin T per 2 W/m^2 increase DWLWIR. AFAIK this work is unpublished, you would have to contact the researchers for more up to date information.
http://modis.gsfc.nasa.gov/sci_team/meetings/200503/posters/ocean/minnett1.pdf

kadaka (KD Knoebel)
August 6, 2014 1:15 pm

@ Monckton of Brenchley on August 6, 2014 at 12:18 am:
Heh. You protect you fanciful little drawing like Mann protects his Hockey Stick, Mann using flipped-over sediments and you with flipped-over triangles. You two also share a tendency to threaten legal action against those who question you.

Let him go into a real library and read some proper books.

There are good university libraries within driving distance. Who then shall watch my elderly mother? As with my father before but not as bad this time, it is quite worrying to leave the house for errands. While she will not suddenly wander off on the way outdoors without warning, an annoying occurrence with him that has trained me to never undress when daring to pause for sleep, she might collapse or otherwise fall down. If I repeatedly cannot reach her on her cellphone that she should be carrying, I quickly head for the checkout and home unless she finally calls back.
I accept it is difficult in your privileged life to imagine the horror of having discovered your mother laid dying because you dared to travel to a real library to counter some internet yahoo who copped a “Do your own research, you ignorant stupid moron!” attitude like seen from a (C)AGW-pushing Team member. You lack equivalent reference points. Compared to that, being continually looked down upon in such places for my permanent sensible uniform of work pants, work shoes, and durable shirt over an obviously not new T-shirt, is nothing.

His quibble about the fact that my simple diagram exhibits symmetry about one of the diagonal axes…

My “quibble” was you took a simple diagram and exerted “artistic license” to make it less simple and more confusing. When I questioned the appearance of your banged-up unannotated mess by viewing it skeptically, you choose to misrepresent that as my not understanding Pythagoras’ proof, which you continue to misattribute to Aryabhata without any evidence by essentially stating “I said it’s true, find the evidence yourself it is true!”
Monckton, among climate skeptics you have truly become your own Mann.
Very well then. Note I had Googled for “aryabhata pythagoras proof” but it came up as “Showing results for aryabhatta pythagoras proof”.
First result has your spelling, at an abandoned blog:
http://akshayunleashed.blogspot.com/2009/05/easiest-proof-to-pythagoras-theorem.html

Now wasnt that as simple as 1,2,3…?I ‘m not sure who gave this proof ,Pythagoras himself,Bhaskaracharya or Aryabhatta .It was Patrick who told me about it and I am grateful…

http://faculty.oxy.edu/ron/math/395/10/ws/15.pdf

Brahmagupta and Bhaskara are two of the most famous Indian mathematicians. They both flourished in the 7th century CE. There was a second mathematician with the name Bhaskara later
Bhaskara I’s Proofs of Pythagoras’ Theorem
Bhaskara gave a pictoral “proof” of the Pythagorean theorem (which had clearly already been known for hundreds of years in India at the time because it appeared in older Indian writings called the Sulbasutras. He gave the following pictures and simply wrote “Behold!” (Eves, 1990).

Not exactly what you presented, this is clear and easy to follow. It’s “square centered in box, not touching outer sides”. The result is the pile of the center square with the two rectangles made from the four triangles, do the simple algebra from adding up the areas.
Aryabhata is mentioned, as having shown he could apply the quadratic formula.
From Drexel University:
http://mathforum.org/isaac/problems/pythagthm.html

Nonetheless, a rigorous, general proof of the theorem requires the development of deductive geometry, and thus it is thought that Pythagoras probably supplied the first proof. Most math historians credit him with a proof by dissection, which relies on the use of two squares, one inscribed inside the other. The Indian astronomer Bhaskara (1114-1185) developed this proof:

It then shows similar to your first pane as the starting point, except simple and clear, “square centered in box, touching outer sides”. No mention of Aryabhata or Aryabhatta.
Another blog, this one decidedly Indian, this post showing how ancient Indian mathematicians knew about Pythagoras’ Theorem long before Pythagoras lived.
http://factsfootprint.blogspot.com/2012/09/baudhayana-pythagoras-theorem-indian.html

Also, another ancient Indian mathematician called Bhaskara later provided a unique geometrical proof as well as numerical which is known for the fact that it’s truly generalized and works for all sorts of triangles and is not incongruent (not just isosceles as in some older proofs).

And there’s that simple drawing again, “not touching”.
No mention of Aryabhata or Aryabhatta here. However the blog does have another post using your spelling:
http://factsfootprint.blogspot.com/2012/09/aryabhattathe-indian-mathematician.html
The author mentions Arybhatta as “the earliest known author on Algebra”, but mostly comments on the astronomy work. He also notes an alternative spelling thusly:

The chief doctrines which Aryabhatta (Aarya-Bhatt) professed were the following:

On August 5, 2014 at 3:30 pm you gave the alternative spelling as “Arya-Bhata”, in variance to this native speaker. Of course, as this author has some curious posts up on crystals and 9/11 and doesn’t know some other common info about Aryabhata, I doubt his spelling is authoritative.
Next up:
http://jwilson.coe.uga.edu/EMT668/emt668.student.folders/HeadAngela/essay1/Pythagorean.html
Quick summary, the simple and clear original version of your drawing is attributed to Pythagoras, “touching” with the other pane drawn without a flipped set. Bhaskara is “not touching” with pile. No mention of Aryabhata or Aryabhatta.
http://pbraun.de/cgi-data/weblog_basic/uploads/2006/09/HoM2.pdf
Aryabhata on algebra, specifically for the formula for the sum of triangular numbers.
http://www.ms.uky.edu/~sohum/aak/pdf%20files/aryabhatta.pdf
An 11Mb download. While the file name has your spelling, the document uses the correct one. This is actually a photocopy of “The Mathematics of Aryabhata” by A. A. Krishnaswami Ayyangar, an old paper. I was able to track down the link source, an informative page about his work by his son, which also lists other authoritative works by AAK, 1896-1953, a tenured mathematics professor.
http://www.ms.uky.edu/~sohum/aak/prelude.htm
The paper appeared in Quarterly Journal of the Mythic Society, Vol. 16, 1926. Aryabhata is noted for his extensive algebraic and astronomical contributions. It is mentioned Aryabhata used the “Pythagorean rule” for a sundial problem. Aryabhata is not noted for a proof of the Pythagorean Theorem, which includes no graphical proof.
An Indian tech blog post:
http://www.tharunpkarun.com/press/edu/famous-indian-mathematicians-profile-and-contributions/
A somewhat confused presentation, “Aryabhatta” appears three times, the correct spelling five times. He is credited with algebra and trigonometry contributions, but not for a proof of Pythagoras’ Theorem.
http://www-groups.dcs.st-and.ac.uk/history/Projects/Pearce/Chapters/Ch8_2.html
Aryabhata credited with algebra, trigonometry, and some geometry contributions. No Pythagoras.
So in summary, by the sources available to me without needlessly endangering my mother, the artistically-interpreted “touching” two-pane version you are showing is attributed to Pythagoras although a one-pane “touching” was attributed to an Indian mathematician, while “not touching” with or without pile is attributed to an Indian mathematician, but the attributed Indian mathematician is Bhaskara.
All this time, since at least the 2010 interview, and you didn’t even have the right mathematician.
And two blogs use your spelling with one blog using both. All academic-related sources used “Aryabhata”.
I realize you are having quite an enjoyable time continually spurning this insolent impoverished peasant with your jackboot, but as it is admirably clear you are wrong about even the actual name of the Indian mathematician, and refuse to provide even a single shred of evidence otherwise which appears quite deceptive, I shall now abandon you to being yourself while I scrounge up a nutritious dinner for my mother and me.

August 6, 2014 1:40 pm

Trick says, August 6, 2014 at 9:20 am:
“Kristian 8:14pm: ”Science of Doom is a guy who thinks ‘heat’ flows both ways in a thermal radiative exchange…”
It does! It is proven. In this case what you write has absolute proof even though it was very hard to come by & took some time – a lot of effort by many pioneers was required as top post says: ”In the physical sciences, absolute proof – which mathematicians call demonstration – is very seldom available. In mathematics, the art that is the language and the queen of the sciences..”
Think I’ve pointed out before you just remain behind in your reading of the absolute proof of your statement by lab test & math written down in several languages & accents. SOD was preceded in that discussion by a Scottish farm boy who developed lab tests & then provided the theory in support of an English book binder apprentice’s more extensive prior lab experiments (incentivized by a couple Frenchmen) showing the clip herein that you wrote to be true. A German then provided the lab experiment proving the Scot’s theory and an Austrian showed why. True multi-national collaboration story. Text books today still follow these guys. Catching up on your reading will enlighten you. Fill in their names for me; prove you did the research work.”

Trick,
It’s been well-established for a long time that you haven’t got the slightest clue what HEAT is and represents in physics. This comment of yours simply acts to underline this recognition.
‘Heat’, trick. Not electromagnetic energy. ‘Heat.’

Monckton of Brenchley
August 6, 2014 3:33 pm

Mr Knoebel seems to suggest that I have threatened him with legal action. I have not done so. I quietly pursue one or two cases a year against those who persist in libels, and do not usually lose the cases. He appears not to understand that my disposition of the triangles in Aryabhatta’s diagram does not alter the validity or rigor of the proof in any way. If he does understand that, then he has been picking nits pointlessly. He has no doubt by now discovered that Aryabhatta may be spelled as I have spelled it, and is frequently spelled that way in India. And his impolite attempt to divert this thread into inconsequentialities was fortunately too late to disrupt what has been (except for his pompous contributions) an uncommonly interesting thread. I should be grateful if he would refrain from attempting to disrupt these threads in future unless he has serious points to make about the principal subject-matter of the head posting. Even then, he must make his points politely, and abandon his habitual sneering tone. He is out of his depth here.

Konrad
August 6, 2014 4:07 pm

Ulric Lyons says:
August 6, 2014 at 9:40 am
———————————-
Ulric,
I have run across that paper before. The idea that in some circumstances DWLWIR could actually cause slight surface cooling is not impossible. However the paper does not cover methodology and the measurements suffer from being environmental rather than lab, the same problem as the Minnett study* Trick tried to counter with.
My own simple experiments –
http://i47.tinypic.com/694203.jpg
http://i42.tinypic.com/2h6rsoz.jpg
– are sufficient to demonstrate that DWLWIR cannot be raising ocean temperatures anywhere close to the 33C claims of climastrologists. However they do not have the resolution to determine very slight cooling or warming. Theoretically, depending on air temperature, humidity, wind speed and water temperature, DWLWIR should be able to induce slight warming or cooling, but the experiment to quantify this would be complex.
*The Minnett “study” was a series of noisy environmental measurements. The idea was to measure sea surface temperature response to varying DWLWIR from passing cloud. This seems reasonable at first blush, until you realise they never tried a simple clean lab experiment first. The study was confounded by sea following thermometer not measuring skin temperature, IR measurement off vertical, no windspeed measurement within millimetres of the surface following thermometer and almost all measurements taken during the day when SW scattering at low angles of incidence was occurring. In short it was garbage. Worse the temperature response they claimed to have found was an order of magnitude lower than what would be required for the “basic physics” of the “settled science” to be correct. Little wonder it was never submitted for peer review. further, to have conducted such an experiment with out a simple lab check speaks to motive.
It also speaks volumes to me that when challenged, no AGW believer can ever provide a simple , repeatable lab experiment comparable to mine demonstrating incident LWIR heating or slowing the cooling rate of liquid water that is free to evaporatively cool. Not a single credible challenge in three years.

Trick
August 6, 2014 4:20 pm

Kristian 1:40pm: “Trick, It’s been well-established for a long time that you haven’t got the slightest clue what HEAT is and represents in physics.”
Concur. I frequently hear that “heat rises”. I have only slight clue what that really means. Energy rises? Nonsense. I also hear about “heat radiation”. Huh? Energy radiation? No. I hear hot bodies radiate heat, the implication being cold bodies do not. What? This misuse of heat term is called word jazz using the words of an eminent meteorologist (ret.). “Heat” term is found lacking in the literature even in the early ‘50s; it is not just me.
I also hear the oceans contain “heat”. For example, Christopher in top post:
“We cannot predict the behavior of the oceans – indeed, we cannot even measure changes in their heat content with sufficient resolution to give a meaningful result.”
I was able to understand Christopher because I parsed his word “heat” into “energy” and it was used correctly; there is plenty of energy content in the oceans but zero “heat”.
Caloric theory of heat and slide rules had their uses and should have some respect. They had their day, did what was asked of them, but have been superseded. They were buried with ceremony & should remain below ground to reduce imprecise confusion in narrative. Calculators and “energy” term are much more precise now, no need to slip back Kristian. Move the scientific method ever forward as in top post.
Work is defined as force thru distance; heat has no such tidy definition. You are forced to resort to paranormal arguments that are sometimes incorrect, a sure road to befuddlement.
Specific heat?? Parse it into specific enthalpy. So on. You got me going (again) and is on topic of top post to move science method forward using history.

kadaka (KD Knoebel)
August 6, 2014 4:33 pm

Re Monckton of Brenchley on August 6, 2014 at 3:33 pm:
And once again Monckton has demonstrated it is beneath his standing to listen carefully enough to the words of the lowly peons to know what was actually said.
Just as well. He has become his punishment in this life, and perhaps the reason for it in the next.

Dr. Strangelove
August 6, 2014 6:44 pm

Konrad is a Dragon Slayer like Doug Cotton and hopeless liar. I did not replicate your experiment because it is flawed. But your limited intelligence cannot comprehend that. I did a different experiment which of course you rejected because it contradicted your false claim.

Dr. Strangelove
August 6, 2014 7:01 pm

Konrad your experiment is flawed. I did not replicate your experiment as you claim. I did a different experiment. Submit your paper to peer-review physics journals stating your conclusion that LWIR cannot heat water. If the reviewers are kind enough, they will not just reject your paper, they will also explain to you why your experiment and conclusion are wrong. No harm trying if you’re really interested to learn.

Dr. Strangelove
August 6, 2014 7:43 pm

Kristian
“We are NOT ‘measuring’ DLR unless it comes in as HEAT, an actual, detectable transfer of energy.”
This is funny because by the same logic we are NOT ‘measuring’ HEAT in a mercury thermometer unless it comes in as VOLUME EXPANSION of the mercury, an actual, detectable movement of fluid in the glass tube. Medieval philosophers will agree with you but scientists and engineers will have a good laugh.
“‘Back radiation’ in a heat transfer situation is NOT detectable.”
This is just hand waving in the face of hundreds of actual measurements of ‘back radiation.’
“It cannot be extracted (isolated) from the radiation field as a whole.”
Of course LWIR can be isolated from the radiation field. The device is called spectrometer. It splits the radiation into different wavelengths. You can see some waves are long, some are short. The long ones are called LWIR. (I suspect the medieval philosophers will again insist it’s not radiation but tiny electric current)

milodonharlani
August 6, 2014 8:02 pm

Samuel C Cogar says:
August 6, 2014 at 11:55 am
Not to mention that there is significant overlap in the absorption spectra of H2O & CO2. Regardless of which GHG is more potent molecule for molecule, the huge preponderance of water vapor over carbon dioxide across the vast majority of the globe, in the latitudes with the most energetic insolation, shows H2O to be on the order of two orders of magnitude more important.
I would hope that most readers here understand the difference between water vapor, which is invisible, & the much larger condensed water droplets, which can be seen in clouds & fog. Or dew, for that matter.

Trick
August 6, 2014 8:05 pm

Konrad 4:07pm: “It also speaks volumes to me that when challenged, no AGW believer can ever provide a simple , repeatable lab experiment comparable to mine demonstrating incident LWIR heating or slowing the cooling rate of liquid water that is free to evaporatively cool. Not a single credible challenge in three years.”
Konrad was recently provided the household equipment list, experimental set-up, results and citations on how to demonstrate “incident LWIR…slowing the cooling rate of liquid water that is free to evaporatively cool”. Apparently he chooses not to replicate and learn from the simple test, speaks volumes.
To reduce fallacy & turn the unknown to known, interested readers can find it and some interesting mod.s described in “Clouds in a Glass of Beer – Simple Experiments in Atmospheric Physics” C. Bohren 2001, sec. 8, p. 53. Originally published in mid-1980s, Konrad has had ample time to research. And in Bohren’s updated “What Light Thru Yonder Window Breaks – More Experiments in Atmospheric Physics” Sec. 7, 2006. Both paperback or Kindle, going for under $11 new. Or at your library.

stas peterson
August 7, 2014 12:23 am

It is well known that inbred royalty results in the likely production of dimwitted imbeciles.
The mental midget and serial abuser of Affirmative Action, used the fear of his professors being labeled ‘racist’, to skate through college. He did this while seldom attending class, and when occasionally visiting to likely be stoned out of his gourd, as he freely admits. He used it all the way to a Law degree from Harvard.
He never practiced and committed himself to rabble rouser, and political extortionist, until selected by the malevolent, marxist, media, manipulators to carry a banner into office. He hid his grades. He out racist the Clintons, guilt tripped the electorate, and now finds himself totally in over his head. The consequences are clear.
What ever made you think he is not another moron, or a near imbecile, as well? What else would you expect from such a pair?

Dr. Strangelove
August 7, 2014 12:31 am

Here’s a simple experiment you can do at home to determine once and for all if longwave IR can heat water. (Of course we already know the answer but for the sake of those who refuse to believe)
Get a thermos with top opening 5 cm diameter. Put 100 ml of cold water at 5 C. With cap open, leave the thermos in a dark room with ambient temperature of 25 C. Close the door and windows so no moving air inside the room. Turn off light and cover windows with thick curtain so no light radiation inside the room. After 3 hours, measure the temperature of water with a thermometer.
Hypothesis: There is only convective heat transfer to the water. Longwave IR does not heat water. Therefore no radiative heat transfer to the water.
CALCULATION
Convective heat transfer equation
Q/t = hc A (Ta – Tw)
Q = heat; t = time; hc = convective heat transfer coefficient of air; A = area exposed to air (thermos top opening); Ta = air temperature; Tw = water temperature
Q/t = 5 W/m^2-C (pi/4 x 0.05^2 m^2) (25C – 5C) = 0.196 W
At t = 3 hrs. = 10,800 sec.
Q = 0.196 W (10,800 s) = 2120 J
Specific heat equation
Q = Cp m (T2 – T1)
Cp = specific heat of water; m = mass of water; T1 = intial water temperature; T2 = final water temperature
Q = 4.18 J/g-C (100 g) (T2 – 5C)
Solving for T2, we get
T2 = 10 C
TEST THE HYPOTHESIS
If after 3 hours, the measured temperature of water is less than or equal to 10 C, the hypothesis is true. No radiative heat transfer.
If the measured temperature of water is greater than 10 C, the hypothesis is false. The water was heated by convection plus IR radiation.
Publish your result at WUWT.

Konrad
August 7, 2014 3:12 am

Dr. Strangelove says:
August 6, 2014 at 6:44 pm
Konrad is a Dragon Slayer like Doug Cotton and hopeless liar. I did not replicate your experiment because it is flawed. But your limited intelligence cannot comprehend that. I did a different experiment which of course you rejected because it contradicted your false claim.
———————————————————————————————————-
I have no association with any “slayers” or “sky dragons”, so again you have just lied. But that is your usual form.
As to Doug Cotton, the permanent record of the internet shows me debating his false claims on numerous occasions. Again you fail epically.
“my limited intelligence”? Get over yourself. Has any of your work, like mine, passed rigorous engineering peer review, won engineering awards and been exhibited in technology museums? No, you just did Al Gore’s “how to be a climate warrior from your mums basement” course. You are a sad joke.
You now claim that you did a “different” experiment, superior to the the three variants I published. You were challenged to provide photos and build diagrams as I always do so others could replicate and check. At Dr. Spencer’s site you fled the challenge. Squealing.
Please provide photos so we may all sneer and laugh at the worst “sleeper” to ever pretend to be a sceptic 😉

Konrad
August 7, 2014 3:33 am

Trick says:
August 6, 2014 at 8:05 pm
“Konrad was recently provided the household equipment list, experimental set-up, results and citations on how to demonstrate “incident LWIR…slowing the cooling rate of liquid water that is free to evaporatively cool”.”
————————————————-
Is that all you have left, lying?
I provide repeatable experiments for others to replicate that show incident LWIR DOES NOT slow the cooling rate of water that is free to evaporatively cool.
You have to mis-quote me? What level of squealing bitchosisty have you descended to?
Then you try citing Bohren. Oh please Trick, not again….
If you want to keep playing could we at least see some new tricks? For entertainments sake..;-)

August 7, 2014 4:14 am

Dr. Strangelove says, August 7, 2014 at 12:31 am:
Another person all confused. ‘Heat transfer’ by IR and ‘radiant emittance’ of IR is not the same thing.
If the air is warmer than the water, heat will be transferred from the air to the water. Also by IR.
If the air is not warmer than the water, heat will not be transferred from the air to the water, via convection/conduction or via IR.
In both cases, though, the bulk air will thermally emit electromagnetic radiation in all directions.
We’ll see if he gets the difference this time …

Trick
August 7, 2014 7:58 am

Konrad 3:33am:
More arm waving, extreme frantic mode, fails to provide counter cites, no reasoned analysis science debate. Comment content 100% bluff and bombast as per Christopher 6:27am: “There is plenty of that in the climate science debate, but I am not impressed by it, and nor should anyone else be.”
If you want me to explain, again, the reasoned science demonstrated by Bohren’s Fourier law of cooling water experiment that shows LWIR slowing the cooling rate of liquid water free to evaporatively cool you requested 9:40am, just ask politely as you demonstrate misunderstanding of it the 1st time. Reducing your unknowns to knowns is on topic.
******
Kristian 4:14am: Correct after parsing: If the air is warmer than the water, energy will be transferred from the air to the water. Also by IR.
Incorrect after parsing since energy transfer is two way, this demonstrates your confusion again:
If the air is not warmer than the water, energy will not be transferred from the air to the water, via convection/conduction or via IR.
Corrected for macro laws:
If the air is not warmer than the water, net energy will not be transferred from the air to the water, via convection/conduction or via IR.
Because as you correctly write incident terrestrial radiation on the water skin from the air will be absorbed, reflected and transmitted by the water skin: In both cases, though, the bulk air will thermally emit electromagnetic radiation in all directions.

August 7, 2014 8:53 am

Monckton of Brenchley@ Aug 6 ,7:43 says:
You conjecture that increased CO2 acted as a positive feedback and further increased warming. The climate record of the last 70 years or so show no such effect. So by empirical data, there is no basis for such conjecture. The “climate sensitivity” remains a theoretical construct which is contradicted by observation.

August 7, 2014 11:58 am

milodonharlani says:
August 6, 2014 at 8:02 pm
I would hope that most readers here understand the difference between water vapor, which is invisible, & the much larger condensed water droplets, which can be seen in clouds & fog. Or dew, for that matter”.
———————
Most all people do understand the difference if they are asked directly …. but they don’t always consider that difference when they are thinking about “greenhouse” gasses and thermal energy absorption and emissions.
Ana a quick Google search provided a PRIME example of what I am speaking of and one can find thousands of other commentaries that are almost identical in context. So, read the following and note my boldfaced text, to wit:
——————-
Water vapour: feedback or forcing?
Any mainstream scientist present will trot out the standard response that water vapour is indeed an important greenhouse gas, it is included in all climate models, but it is a feedback and not a forcing.
The overlaps complicate things, but it’s clear that water vapour is the single most important absorber (between 36% and 66% of the greenhouse effect), and together with clouds makes up between 66% and 85%. CO2 alone makes up between 9 and 26%,
While water vapour is indeed the most important greenhouse gas, the issue that makes it a feedback (rather than a forcing) is the relatively short residence time for water in the atmosphere (around 10 days)
”.
Ref: http://www.realclimate.org/index.php?p=142
————————–
People are actually convinced that the H2O vapor (humidity) is the “forcing” feedbacker of thermal energy to the CO2 …… and the CO2 is the “feedback” forcer of thermal energy that is driving the INCREASE in average near-surface temperatures. They literally believe that to be “true”, a fact of science.
And all because they believe that the 30,000 to 40,000 ppm of H2O vapor (humidity) only remains resident in the atmosphere for a maximum of 10 days. One seldom, if ever, hears or reads a local or regional “weather report” that doesn’t mention the RESIDENT “humidity”.
HA, H2O vapor (droplets) in the form of clouds, fogs and mist might only hang around for a maximum of 10 days ….. but that RESIDENT H2O vapor (humidity) from the tropics thru most all the temperate zones ….. hangs around Winter, Summer, Spring and Fall …. and does almost twice (2X) the forcing that the CO2 does ….. and “quantity wise” ….. nigh onto 100 times that of the total atmospheric CO2.
And if you note in the above the author only made mention of the H2O vapor (droplets) that form clouds. And nowhere have I read where a climate scientist and/or a proponent of CAGW has made mention of the H2O vapor (droplets) that form fogs.
And “DUH”, it is of my learned opinion that iffen the …. total annual fog coverage of the earth’s surface was accounted for each year …. then it would probably be far GREATER THAN the total annual cloud coverage.
River fogs, lake fogs, seacoast fogs, ocean fogs, valley fogs, after rain fogs, lowland fogs, city fogs, mountain fogs, etc. Ya know, The Great Smokey Mountains did not get their name because of forest fires. Thus, fogs and clouds are “two peas in a pod”.
Cheers

Konrad
August 7, 2014 3:43 pm

Trick says:
August 7, 2014 at 7:58 am
————————————
Trick, I owe you an apology. I mis-read your previous post and thought you were mis-quoting me and I accused you of lying by omission. I was wrong.
You were actually claiming to have provided a simple repeatable experiment that showed incident LWIR at the surface of a water sample free to evaporatively cool slowing its cooling rate. You were actually lying by commission.
Shall I link to the Talkshop post where you claimed wrapping al-foil around IR opaque glass flasks of water demonstrated the effects of LWIR on the surface of water? Some may find it entertaining 😉
Or perhaps I should link to the time you tried to dismiss the role of radiative subsidence in atmospheric circulation and argued black and blue that I couldn’t drive convective circulation in a fluid column by removing energy from the top of the column?
Or perhaps the time, when faced with the truth that without atmospheric cooling our oceans would become an evaporation constrained solar pond with temperatures far above 255K, you panicked and tried to claim that the 255k figure was for the rock under the oceans? That was a Trick classic!
Or perhaps I should ask you for the 50th (?) time to provide a simple repeatable empirical experiment others can try showing LWIR slowing the cooling rate of liquid water that is free to evaporatively cool with air flow across the surface comparable to Beaufort scale 4. No hand waving, texts or “cites”. Build diagrams and photos of your work is all you need. It shouldn’t be too hard should it? It would be a bit sad if something so fundamental to the “basic physics” of the “settled science” couldn’t be demonstrated with a simple lab experiment….

Monckton of Brenchley
August 7, 2014 4:39 pm

Mr Painter is confused. He says the climate record of the last 70 years or so shows no warming that might be attributable to the CO2 feedback. Well, the record shows warming. The record also shows CO2 increasing. One could make an estimate of what part of the CO2 increase is attributable to outgassing if we were able to measure ocean heat content accurately, but we are not able to do so. One could estimate what part of the temperature increase was caused by CO2 forcing, what part by CO2 feedback, and what part by natural causes, if the available information were sufficient.
But the information is insufficient. It is also insufficient to rule out warming as a result of CO2 forcing or feedback. Mr Painter should learn that there is a large element of uncertainty in climatology, Though he did not quote me correctly, I said there was a statable case (not a certainty) that the CO2 feedback had amplified naturally-caused temperature changes in the paleoclimate (and not in the past 70 years).

Trick
August 7, 2014 5:19 pm

Konrad 3:43pm – Go ahead, make my day, link away. Turn the unknown to known. Like Willis writes: “quote my words.”
“Build diagrams and photos of your work is all you need.”
Concur but need the data & reasoned analysis thereof too. I have already provided these in the source material 8:05pm, no hand waving, none, nada, zilch just experiments & data plotted w/cooling lines with reasoned analysis – you know: the scientific method. Black & white pix of the experimental apparatus (basically a lab glass flask of tap water, thermometer held in center of water by paper clips, and aluminum foil) will have to do but this is not a lesson in color.
Or you can use Dr. Strangelove’s 12:31am experiment & being the ace experimentalist, be the 1st to publish results right here on WUWT to double check “demonstrating incident LWIR….slowing the cooling rate of liquid water that is free to evaporatively cool.” You now have TWO recent credible experimental answers to your original challenge! Impressive site.
Now you offer a modified challenge 3:43pm & the answers to this one are also in the bag:
“…provide a simple repeatable empirical experiment others can try showing LWIR slowing the cooling rate of liquid water that is free to evaporatively cool with air flow across the surface comparable to Beaufort scale 4.”
Nature’s answer for global climate windiness is beyond calm kitchen table lab test & and your data taking equipment is not precise enough to capture the effects of ocean waves radiating to themselves and spray and clouds so there is lotsa’ scatter but a signal. Have to go out on the ocean in situ on a research vessel with modern properly calibrated precision equipment for that. I posted the reported reduced results 1:02pm above from data taken day & night over about a month in all natural wind conditions near New Zealand. You will simply have to contact the researchers for “Build diagrams and photos” of their work. I linked their contact info. Descriptions & photos of the equipment Marine-Atmospheric Emitted Radiance Interferometer (M-AERI) are published for you.

Dr. Strangelove
August 7, 2014 7:36 pm

Konrad
Sorry I do not wish to engage in ad hominem though you started it. I withdrew my insulting comment but it still got posted. No more insults from you and me. You can chose to do the experiment to prove or disprove your claim. You are the one making an extraordinary claim: LWIR cannot heat water. The burden of proof is on your side.

Dr. Strangelove
August 7, 2014 7:58 pm

Kristian
“Another person all confused. ‘Heat transfer’ by IR and ‘radiant emittance’ of IR is not the same thing.”
You’re the one confused because I never said radiant emittance is heat transfer by IR. Where did you get that?
“If the air is warmer than the water, heat will be transferred from the air to the water. Also by IR.”
Good, you understood that one.
“If the air is not warmer than the water, heat will not be transferred from the air to the water, via convection/conduction or via IR.”
I’m sure you’re not talking about my proposed experiment where air is at 25 C and water at 5 C. See 25 > 5
“In both cases, though, the bulk air will thermally emit electromagnetic radiation in all directions.”
Yes of course. I hope you see ‘all directions’ include downward to the water surface.
“We’ll see if he gets the difference this time …”
Did you get it now?

August 7, 2014 8:31 pm

No, I am not confused. The late warming trend circa 1977-97 has been shown to be due to increased in insolation, not any enhanced greenhouse effect (CO2). The great failing of climate science is the persistence of theory which has been falsified by observations. Thank you for replying.

Monckton of Brenchley
August 8, 2014 12:43 am

Mr Painter remains confused. Though the reduction in cloud cover from 1983-1998 did indeed allow more sunlight to reach the surface, and probably had a greater warming effect than CO2, it remains impossible to rule out some warming contribution from CO2 (see my paper on this very point in the World Federation of Scientists’ Annual Proceedings for 2011). A false balance is abomination to the Lord: but a just weight is His delight (Proverbs, XI:1). We do no favors to science by trying to push the truth too hard in one direction or another. Disentangling the relative contributions of natural and anthropogenic warming remains difficult and we should not try to pretend otherwise.

August 8, 2014 1:30 am

Dr. Strangelove says, August 7, 2014 at 7:58 pm:
I do see it. And my apologies for my rushed misinterpretation of what you were trying to say. Agreed. Put a glass (or a thermos) of 5 degree water on a table inside a room at 25 degrees and eventually the water will have reached 25 degrees also. From heat transfer from the air to the water. It will be mostly through convection > conduction, but I don’t see why radiation shouldn’t also make a (small) contribution.
However, I don’t see how this is relevant to the real-world ocean surface/atmosphere situation, where globally, the ocean surface is always warmer than the air/atmosphere above. DWLWIR from the atmosphere can not directly heat the surface, meaning increase its ‘internal energy’, raising its temperature. Because it isn’t ‘heat’.

August 8, 2014 7:26 am

Proverbs?
You cite Proverbs?
You are right, I am confused- Proverbs on WUWT! And what shall I see next- Isaiah?
Oh I shall be leaving now, I have done my best but I have trumped by- Proverbs!

August 9, 2014 5:24 am

Will this experiment prove or disprove the “greenhouse” gas theory?
Items required for conducting experiment.
1. A “point” source of high-intensity visible light
2. A 1 inch diameter reflecting mirror.
3. A 10 gal (2,310 in3) “round” (spherical) glass bottle with 1” diameter neck portal.
4. 10 gal of a “clear” no-reflective liquid.
5. A precision Lux meter.
6. 5,000 ¼“ diameter spheres w/highly reflective surfaces and same density as above liquid.
Conduct experiment
1. Fill glass bottle with liquid and darken room.
2. Turn on light source and use mirror to direct “light beam” at center of glass bottle.
Make measurements using Lux meter
1. First measure and record the intensity of the reflected “light beam” at the location it strikes the glass bottle.
2. Next measure and record the intensity of the reflected “light beam” at the exterior surface of the glass bottle, directly opposite of where it strikes the glass bottle.
3.Next measure and record the intensity of any “scattering” (re-reflected) of the reflected “light beam” at various surface areas around the circumference of the glass bottle
Modify experiment parameters
1. Add 4,000 of the ¼“ diameter spheres to the liquid in the glass bottle. (4,000 simulates 10% of 40,000 ppm of atmospheric H2O vapor (humidity)
2. Stir the liquid to disperse the ¼“ diameter spheres throughout the liquid.
3. Repeat measurement instructions #2 and #3 above.
Modify experiment parameters again
1. Add 40 more of the ¼“ diameter spheres to the liquid in the glass bottle. (40 simulates 10% of current 400 ppm of atmospheric CO2.
2. Repeat measurement instructions #2 and #3 above.
Modify experiment parameters again
1. Add 40 more of the ¼“ diameter spheres to the liquid in the glass bottle. (adding 40 simulates a doubling of the current 400 ppm of atmospheric CO2 to a total 800 ppm.
2. Repeat measurement instructions #2 and #3 above.
Experiment completed
1. Analyze the recorded results of all Lux meter measurements, for each of the three (3) different parameters with specific emphasis on the intensity of the “scattering” effect of the re-reflected light that “exits” the glass bottle.
2. Analyze and compare the three (3) different results obtained in #1.
3. Write summery report.

Konrad
August 9, 2014 9:04 pm

Dr. Strangelove says:
August 7, 2014 at 7:36 pm
“ You are the one making an extraordinary claim: LWIR cannot heat water. The burden of proof is on your side.”
————————————–
Actually my claim and supporting experiments are not extraordinary. Process engineers, particularly in food processing are well aware of the problem. They would love sterile non-contact IR heating elements for liquids, but it just doesn’t work for materials free to evaporatively cool.
The experiment you offered tells you nothing. You don’t know if conduction or radiation heated the liquid. You would need to do as I have done, and use two identical water samples with identical air flow over the surface (remember – average wind speed over the oceans Beaufort scale 4) with the only difference between samples being the strength of LWIR incident on the surface.
No supporter of the idea of a net radiative GHE has ever been able to produce evidence of such an experiment.

Konrad
August 9, 2014 9:42 pm

Trick says:
August 7, 2014 at 5:19 pm
———————————–
The sadness continues…
“Concur but need the data & reasoned analysis thereof too. I have already provided these in the source material 8:05pm, no hand waving, none, nada, zilch just experiments & data plotted w/cooling lines with reasoned analysis – you know: the scientific method. Black & white pix of the experimental apparatus (basically a lab glass flask of tap water, thermometer held in center of water by paper clips, and aluminum foil) will have to do but this is not a lesson in color.”
Let me guess, nothing at all to do with incident LWIR on the surface of water that is free to evaporatively cool? Go on, scan the photo and post it. You have nothing do you? You’ve had three years and you still can’t counter my experiments 😉
“Or you can use Dr. Strangelove’s 12:31am experiment & being the ace experimentalist, be the 1st to publish results right here on WUWT to double check “demonstrating incident LWIR….slowing the cooling rate of liquid water that is free to evaporatively cool.” You now have TWO recent credible experimental answers to your original challenge! Impressive site.”
Dr. Strangelove’s attempt is no use. It doesn’t answer the relevant question. And I bet your Bohren goose chase will be al-foil over IR opaque glass just like your sorry efforts…
And this? –
“Nature’s answer for global climate windiness is beyond calm kitchen table [that pathetic old smear?] lab test & and your data taking equipment is not precise enough to capture the effects of ocean waves radiating to themselves and spray and clouds so there is lotsa’ scatter but a signal. Have to go out on the ocean in situ on a research vessel with modern properly calibrated precision equipment for that. I posted the reported reduced results 1:02pm above from data taken day & night over about a month in all natural wind conditions near New Zealand. You will simply have to contact the researchers for “Build diagrams and photos” of their work. I linked their contact info. Descriptions & photos of the equipment Marine-Atmospheric Emitted Radiance Interferometer (M-AERI) are published for you.”
– you’re just trying to run back to Minnett’s tripe. Why bother? If they could effectively demonstrate incident LWIR slowing the cooling rate of water that is free to evaporatively cool they would have done it in the lab first. They never did. Worse they couldn’t even get close to the extent of the supposed effect in their “results” from actual oceans. Minnett did not demonstrate that incident LWIR can raise the temperatures of our oceans 33C, nor have you or any other AGW believer.
So trick, for the 51st time I challenge. Show a repeatable empirical experiment that others can replicate demonstrating incident LWIR at the water/air interface heating or slowing the cooling rate of water that is free to evaporatively cool. You still can’t do it can you?

Trick
August 9, 2014 10:32 pm

Konrad 9:42pm: “Show a repeatable empirical experiment that others can replicate demonstrating incident LWIR at the water/air interface heating or slowing the cooling rate of water that is free to evaporatively cool. You still can’t do it can you? “
Yes. I have. More frantic hand waving by Konrad, offering only bluff & bombast in response. No science.
The experiments are self evident. One is performed, analyzed, pictured, results documented Konrad. You have now been shown at least twice empirically & replicable that “incident LWIR at the water/air interface …slowing the cooling rate of water that is free to evaporatively cool.” Even tougher for you, a third one performed out in the wild in all wind conditions.
This challenge of Konrad’s was a simple one to document, already accomplished almost 30 years ago and Konrad did not even know. Konrad offers no defense. None, zilch, nada. Nature’s results stand; again, are self evident.

Trick
August 9, 2014 10:41 pm

The problem for Konrad remains, boils down to his experiments being purported to prove either satellites measuring Tmean = 255K or thermometers measuring Tmean = 288K are wrong. To date, Konrad’s efforts have not achieved even a modicum of generally accepted success in that endeavor despite his experiments. Nothing published by Konrad. No cites. The satellites and thermometer field results stand in the face of Konrad’s experiments. The root cause is Konrad’s inaccurate interpretation of his experiments to suit Konrad’s view.

Konrad
August 10, 2014 12:33 am

Trick says:
August 9, 2014 at 10:32 pm
—————————————
Go on Trick, post the photos of the experiment you claim demonstrates incident LWIR at the water/air interface heating or slowing the cooling rate of water that is free to evaporatively cool. What’s the hold up? You obviously have a computer. What is so hard?
You have nothing do you?

Konrad
August 10, 2014 12:56 am

Trick says:
August 9, 2014 at 10:41 pm
————————————-
Ooooh, Two posts!
“The problem for Konrad remains, boils down to his experiments being purported to prove either satellites measuring Tmean = 255K or thermometers measuring Tmean = 288K are wrong.”
What are you blithering about?! Both these figures are close to correct. Nothing I am empirically demonstrating contradicts this! A planet with a selective surface and a highly radiative atmosphere, that is absorbing around 240 w/m2 will appear to be radiating at 255K. What is so hard to understand?
“To date, Konrad’s efforts have not achieved even a modicum of generally accepted success in that endeavor despite his experiments.”
Actually more than I could ever imagined a single individual could ever have achieved! Go Internet!
“Nothing published by Konrad. No cites.”
Funny thing, someone else cut and pasted my work and got it peer reviewed and into an engineering journal. You fail again. (although on this one I admit to being slightly confused…)
“The satellites and thermometer field results stand in the face of Konrad’s experiments.”
Nope, no conflict there. Running back to environmental readings because you have no actual repeatable lab experiments was it Trick?
“The root cause is Konrad’s inaccurate interpretation of his experiments to suit Konrad’s view”
And back to the Alinsky method of isolate and vilify…sigh. “Konrad’s view” wins engineering awards. Your view, in contrast, just leaves a permanent record on the interwebs utterly discrediting every acolyte of the Professional Left from now until the heat death of the universe 😉
Come on Trick, pony up! Where is your repeatable lab experiment showing incident LWIR slowing the cooling rate of water that is free to evaporatively cool. Links to photos not “Kindal” if you would be so good. You wouldn’t want to be rated with the Viscount Monckton “but, but, but the maths says it must but I have no empirical evidence” crowd now would you? (yes, yes I know, I’m doing “group and vilify”, but Christopher sooooooo deserves this after his “slayer” comments 😉 )

Trick
August 10, 2014 8:24 am

Konrad 10:32pm, 12:56am: “Funny thing, someone else cut and pasted my work and got it peer reviewed and into an engineering journal.”
Link of the funny thing? Citation? Nothing really? Zilch reasoned science, only bluster, bombast. The bulk of generally accepted science stands opposed to Konrad fallacious interpretation of experiment.
“…post the photos of the experiment you claim demonstrates incident LWIR at the water/air interface…..slowing the cooling rate of water that is free to evaporatively cool….Where is your repeatable lab experiment showing incident LWIR slowing the cooling rate of water that is free to evaporatively cool.”
Done, posted. Konrad has a computer & library access, let me post pictures for 52nd time see right here for tests & photos of experimental set-up 1st published almost 30 years ago demonstrates incident LWIR at the water/air interface slowing the cooling rate of water that is free to evaporatively cool:
http://wattsupwiththat.com/2014/08/03/the-central-climate-fallacy-is-that-the-unknowns-are-known/#comment-1703419
Do this test, take a picture, post the results:
http://wattsupwiththat.com/2014/08/03/the-central-climate-fallacy-is-that-the-unknowns-are-known/#comment-1703504
Contact these researchers, get photos their equipment & in situ ocean pictures & latest research on the subject:
http://www.emetsoc.org/fileadmin/ems/dokumente/awards/young_scientist_travel_awards/poster_esasummerschool2012_wong.pdf
See brightness temperature emissivity experiment 9-3, repeat it, published 1987, very easy:
http://fas.org/irp/imint/docs/rst/Sect9/Sect9_2.html
See Henderson et. al. 2003, Konda et. al. 1994, Niclòs et. al. 2005 all demonstrate incident LWIR at the water/air interface slowing the cooling rate of ocean water that is free to evaporatively cool:
http://public.lanl.gov/jt/Papers/polemiss.pdf
http://www.terrapub.co.jp/journals/JO/pdf/5001/50010017.pdf
http://www.researchgate.net/publication/223747223_In_situ_angular_measurements_of_thermal_infrared_sea_surface_emissivityValidation_of_models
After that you will see the 255K and 288K Tmeans are affirmed in generally accepted physics, oh wait:
“Both these figures (optically thin atm. 255K, optically thick atm. 288K) are close to correct. Nothing I am empirically demonstrating contradicts this!”
Good! Konrad finally sees the light & correctly affirms all the testing. This demonstrates success in reading the above; Konrad finally gains the generally accepted reasoned physics knowledge and concurs the results show incident LWIR at the water/air interface…..slowing the cooling rate of water that is free to evaporatively cool and the oceans are a spectral selective surface with emissivity around 0.96 depending slightly on weather conditions due none of Konrad tests “contradict this”.
No backsliding Konrad. Stick to the Tmean = 255K thin atm. at/from satellite orbit, and Tmean = 288K surface in optically thick (tau) Earth atmosphere from thermometry demonstration are close to correct as you write. You will go far.
Bookmark this post’ link Konrad (I will), and hit this button right here…

Konrad
August 11, 2014 7:08 pm

Trick says:
August 10, 2014 at 8:24 am
————————————-
“Link of the funny thing? Citation? Nothing really? Zilch reasoned science, only bluster, bombast.”
Where has accusing me of lying ever got you?
“The bulk of generally accepted science stands opposed to Konrad fallacious interpretation of experiment.”
The bulk of what? Cliamastrology? Process engineers don’t try to heat water with LWIR. Only climastrologists believe that works, no one else.
“Done, posted. Konrad has a computer & library access, let me post pictures for 52nd time …”
The only thing you have achieved for the 52nd time is to utterly fail to produce an empirical experiment demonstrating incident LWIR on the surface of liquid water that is free to evaporatively cool heating it for others to replicate. You say there is a photo somewhere in a library? Scan it and post it using TinyPic then. How hard is that? Again you have nothing do you? Posting links to texts that have to be paid for instead of direct links to images speaks to motive.
“Contact these researchers, get photos their equipment & in situ ocean pictures & latest research on the subject:”
Sigh. Minnetts tripe again? Have you nothing else?
“See Henderson et. al. 2003, Konda et. al. 1994, Niclòs et. al. 2005 all demonstrate incident LWIR at the water/air interface slowing the cooling rate of ocean water that is free to evaporatively cool:”
Ooh, maybe. Links to more papers! This could be good…but no, each and every one of this deals with apparent not EFFECTIVE IR emissivity of water. Nothing there about empirical tests of the effective emissivity of water with background IR and Hohlrumn/cavity effect minimised. Nothing about the effect of LWIR on the cooling rate of water free to evaporatively cool. Nothing at all. And how many times? Those measurements are for apparent emissivity settings for IR measurement of water in situ. They cannot be used in radiative balance equations.
K – “Both these figures are close to correct. Nothing I am empirically demonstrating contradicts this!”
T – “Good! Konrad finally sees the light & correctly affirms all the testing. This demonstrates success in reading the above; Konrad finally gains the generally accepted reasoned physics knowledge and concurs the results show incident LWIR at the water/air interface…..slowing the cooling rate of water that is free to evaporatively cool and the oceans are a spectral selective surface with emissivity around 0.96 depending slightly on weather conditions due none of Konrad tests “contradict this”.”
Pathetic, truly pathetic. First, you never get to condescend to me. You claimed I couldn’t drive convective circulation in a fluid column by removing energy from the top of the column. Your physics is woeful.
Secondly about that “finally”…. to try that you would need to show where on the web I have ever claimed that a planet absorbing around 240 w/m2 would be radiating anywhere other than 255K if in near radiative balance. That would be something else you can’t do, because I have never claimed otherwise.
“No backsliding Konrad. Stick to the Tmean = 255K thin atm. at/from satellite orbit, and Tmean = 288K surface in optically thick (tau) Earth atmosphere from thermometry demonstration are close to correct as you write. You will go far.”
Oh please. A planet absorbing ~240 w/m2 will have an apparent radiative temperature viewed from space of ~255K. This is news? To know anything about near surface temperatures you would need to know surface and atmospheric properties, and my empirical experiments prove beyond a shadow of a doubt that climastrologists got the oceans wrong. Not a little bit wrong. Totally and utterly wrong.
“you will go far”??! I am currently sorting a few mil of mechanical engineering and design for the Samsung corporation and you are not. How much further would you like me to go 😉

Brian H
August 11, 2014 9:07 pm

Lord Moncton’s assumptions are in the way of setting boundaries, as in “let’s take the worst case, for example”. Even given all such extremes, the numbers crank out unalarming results. And the assumptions are highly leveraged; dial them back only a little and the purported outcomes shrink rapidly to negligible levels.
As to the recent cover story for the Pause, that somehow natural variance has freakishly hidden the AGW effect, it is logically necessarily true that if it can do that once, without warning, it can do it whenever it wants, and the sign can go either way. In other words, it has the potential to dominate just lurking around, and is thus in charge of whatever occurs at all times.